Sei sulla pagina 1di 68

a

+.
,''' r'"
B^
"i'
X. a. l

iJ' \: {,4}r
..'.;- ,.r :
,-]
\ 'i,+1.
).4-
',i.':.-!
,.

,:/i,'*
-*-
./:
,tj,t
'
.L i. t
it .lr'

; \\
,-,..:jii' f tt * ka *

3
'*r.-i ) - d,."-
''-^@l' rr*, *GE R NTERNATT'NAL
dJ
r

5.E #;x$

* *lr .r*
.:ji

.!t
G)
t-
m
T
o

courtesy of Universit)

GeodesicDomeby R. Buckminster Fuller

IF ALL BUCKMINSTER FULLER EVER DID WAS so monumental. The photo below shows Buckminster
I invent the geodesic dome, his place in history would have Ful1er and his wife, Anne Hewlett Fuller, in their geode-
been assured. It is considered by some the most significant sic home in Carbondale, Illinois. Turn to page 8 for a look
structural innovation of the 20th century. It encloses more at even smaller geodesic structures: hollow carbon mol-
space with less material than any alternative form. "When ecules called "buckyballs" in honor of this wide-ranging,
I invented and developed my first clear-span, all-weather geo- forward-looking thinker.
desic dome," Fuller wrote/ "the two largest domes in the
world were both in Rome and were each about 50 meters in
fiameter. They are St. Peter's, built around e.o. 1500, and
the Pantheon, built around a.o. 1. Each weighs approxi-
mately 15,000 tons. In contrast/ my {irst 50-meter-diam-
eter geodesic dome installed in Hawaii weighs only 15
tons-one thousandth the weight of its masonry counter-
part. An earthquake would tumble both the Roman domes,
but it would leave the geodesic unharmed."
Fuller coined the word "tensegrity" for the continuous
tension/discontinuous compression structural system he
developed from an idea he learned from a sculptor. He soon
reahzed that, because of the greater efficiency of tension
compared to compression, very large domes could be built
with his tensegrity trusses. He calculated that a 3-km
dome would weigh only 4,000 tons.
In the photograph above, Fuller is standing in front of
the76-rn dome that housed the US pavilion at the world's
fair in Montreal in 1967. Not all geodesic domes are quite r$
Rrl.rr
fltur,'lr1 lhorograplu. \crr,.. Lnrrrre\) ,il rrt\e.\ l\
un- sn rthrrr lll iu . Lt iver-ir) .rr ( t ond rl. \lU(
n
MAY/JUNE 1994
TU VOLUME 4, NUMBER 5

FEATU R ES
4 Prime Properties
Ihree lalhs lo ]tlll. terlnal-[uler
by Vladimir Tikhomirov

I MolecuiarArchitecture
tollow lhe lounrinu luulylall
by Sergey Tikhodeyev

10 Mathematics and Psychology


A sll'an[e Elnler'or' altd a $traflUe 0e[sl'al
by lgor Akulich

C ov er art by Leonid Tishkov


2ll Large-scale Phenomena
Some of us can recall, with a twinge T[e loundilt$ lnair
of shame, a time when it was oh-so- by lvan Vorobyov
easy to make a bit of money. A11 you
had to do was strike a little deal with
a younger kid: "Here, I'11 trade you
this nice, big nickel for that little dime
of yours." It was a nice scam-for a
DEPARTMENTS
while, at least. Either the kid got wise,
or someone older intervened-with 2 Puhlishel''$ Paus 38 Physics Conlesl
dire consequences for the clever cur- Laser levitation
rency trader. I 5 Bnainteasel's
Who can blame the four-year-old
42 0uanlum $miles
for thinking a nickel is worth more 1g llow 0o You tiUune? A mathematical handbook
with no figures
than a dime? Only an adult would
think of making a bigger coin less 20 tulathematical Sul'[rises
40 Al lho Blackloal'd ll
valuable than a smaller one. It all but Six challenging dissection
Nine solutions to one
invites fraud in the seven-year-old ta sks
problem
mind. Of course/ grown-ups aren't im-
mune to psychological mishaps when 28 ttllath lnuesli0atiolt$ 50 ]laIpeninUs
it comes to money. In [act, the noble Happy birthday, Uncle
Paul! A tale of one city ...
Roman on our cover has fallen prey to Bulletin Board
a cunning emperor. |ust how cunning?
You be the judge. The story begins on
30 ln Youn ]lead 53 Cnissu'oss Science
page 16. Geometric summation

!2 l(aleidosrope 54 An$wens, ]linls & Sululions


Horseflies and flying horses
02 Tuy Slone
35 Al Ihs Blarkhoand I The rolling cubes
A little lens talk

0 U A !l T l,l lil / C0 il I t li I $
PUBLISHER'S
PAGE

lllumlel,s in our UsrE$


The role of quantification in molecular biology

I N A RECENT EDITORIAL IN viral genes. A second gene, called


journal Nature {Vol. Cro, will bind to the same DNA site
associated with molecular biology.
I the British One is the use of quantity, svmbols,
1368, 10 March 19941, |ohn Mad- if there is no repressor protein and equations; the other is their ap-
I dox makes an important point present and repress the activity of plication in fundamental larvs of
regarding the present descriptive the repressor. This is called a science, such as thermodl-namics
character of molecular biology. switching mechanism. In examin- and kinetic theory.
Maddox observes that in the human ing this situation, researchers have I have noticed a tendency to
genome proiects, the main goal is to found that there are only about i00 downplay the "hard sci.ences" on
list the genes and "to specify their or so free molecules in the cell un- the grounds that the real1r exciting
nucleotide sequences. " The projects der consideration. This means that areas of research are in molecular
also try to specify "the sequences of there are fluctuations of macroscopic biology. Yet even if that is u.here the
regions of DNA that hoid the genes laws of equilibrium thermodlrramics "excitement" is, you rr.r11 come to a
together in the chromosome." that are not accounted for. Maddox dead end if you fail to utilize phys-
These activities are, first of. all, speculates that the genetic switch ics and chemistry. You can't get to
ones of identifying structure and giv- may in factbe a krnetic phenomenon, the heart of natural phenomena if
i.ng names to that structure. Second, and that "the energetic implications you don't understand the basic laws
researchers make connections be- of what appears to be equilibrium and principles of science that under-
tween these structures and other constants may be spurious." lie those phenomena.
structures or characteristics of the As Maddox points out, "The Readers oI Quantum magazine
organism that are inherited through naming of parts does not in itself continually see the unusual and ex-
the gene. Still, these processes are yield understanding." He goes on to citing ways in which such funda-
inherently descriptive. They do not list the problems yet to be resolved: mentals lead to a more profound un-
re\y on qu antit ativ e rcIationships. As "how the molecules olrepressor fold derstanding of varied phenomena in
description, we can tell what the re- into their characteristic dumbbell bioiogy as well as in other areas of
sults will be, and we can give names shape, why dimers are so much science. Recall, for instance, "Math-
to genes and sequences of regions of more stable than monomers, and ematics in Living Organisms" (No-
DNA. But we cannot make quantita- how an alpha helix in the amino- vember/December 1992), where cats
tive predictions. Furthermore, quali- component interacts with DNA at are shown to be handy with loga-
tative predictions that fail to consider the binding sites. To be more pre- rithms; or "Trees Worthy of Paul
underlying quantitative variables cise, what happens has been deter- Bunyan" (November/December
may well be wrong. mined by elegant genetic experi- 19931, where physical processes af-
Most importarLtt regardless of ment; why that, not something else, fecting plant growth are expiored.
how well descriptive molecular bi- happens remains to be discovered." Regardless of your interests-but es-
ology may tellwhathappens, it does In this brief overview I haven't pecially if you intend to go into the
not te1l why it happens. This can done justice to Maddox's editorial. If Iife sciences-continue to study
only occur if we can apply laws of you have read some molecular bioi- physics and chemistry and acquire
science that are quantitative to the ogy, I recommend that you read the the mathematical tools needed for
situation. Maddox uses as an ex- essay in its entirety. Here, I simply research in all the sciences. We hope
ample a virus that infects E. coli, wanted to give a sketch of his ideas Quantum will help keep you on
called bacteriophage 1,, and areprcs- to suggest the importance of two that productive path.
sor protein produced by one of the aspects of science that aren't often Aldridge
-Bi1lG.
lllY/Jutlt I 904
OUANTUM
Be a lacton in lfle

OUANTUM
Eqllalim! THE MAGAZINE OF MATH AND SCIENCE
A publication of the National Science Teachers Association (NSTA)
Have you written an article that O Quantum Bureau of the Russian Academy of Sciences
you think belongs it Quantum! in conjunction with
Do you have an unusual topic the Ameilcan Association of Physics Teacherc (AAPT)
that students would find fun and O the National Council of Teacherc of Mathematics (NCTM)
challenging? Do you know o{ The Natioaal Science Teachers Associatian is an otganizatton of sctence educatian prcfessionals
anyone who would make a great and has as its ltupose the stimDldtion, improventenL and coordhtation of science teaching and leaming.

Quantum author? Write to us


and we'll send you the editorial Publisher
guidelines {or prospective Quan- Bill G. Aldridge, Executive Director, NSTA
tum contributors. Scientists and Associate Publisher
Sergey Krotov, Di1ectol, Quantum Bureau,
teachers in any country are in- Professor of Physics, Moscow State University
vited to submit material, but it Founding Editors
must be written in colloquial Yuti Ossipyan, President, Quantum Bureau
English and at a level appropriate Sheldon Lee Glashow, Nobel Laureate (physicsl, Harvard University
William P.. Thurston, Fields Medalist (mathematics), University of Caliiomia, Berkeley
for Quantum's predominantly
student readership. Field Editors for Physics
Larry D. Kirkpatrick, Pro{essor of Physics, Montana State University, MT
Albert L. Stasenko, Professor o{ Physics, Moscow Institute of Physics and Technology
Send your inquiries to:
Field Editors for Mathematics
Managing Editor Mark E. Saul, Computer Consultant/Coordinator, Bronxville School, NY
Vladimir Dubrovsky, Associate Professor of Mathematics, Moscow State University
Quantum
Managing Editor
1840 Wilson Boulevard Timothy Weber
Arlington, VA 22201-3000 Staff Attist
Sergey Ivanov
Editorial Consultants
Alexander Buzdin, Professor of Physics, Moscow State University
Yuly Danilov, Senior Researcher, Kurchatov Institute
Ulhatb [appening? Larissa Panyushkina, Managing Editor, Quantum Bureau

Summer study ... competitions ... new Inl ern ational C onsuhant
books ,,. ongoing activities clubs and as-
... Edward Lozansky
sociations ... free samples .,, contests ,., Advertising Managers
whatever ii is, if you think it's of interest to Paul Kuntzlet (Washington office)
Bob Vrooman (New York office)
Quantum readers, let us know about itl
Help us fill Happenings and the Bulletin Advisory Board
Bernatd V. Khoury, Executive Of{icer, AAPT
Board with short news items, firsthand re-
|ames D. Gates, Executive Director, NCTM
poris, and announcements of upcoming George Berzsenyi, Professor of Mathematics, Rose-Hulman Institute of Technoiogy, IN
events. Arthur Eisenkraft, Science Department Chair, Fox Lane High School, NY
Karen fohnston, Professor of Physics, North Carolina State University, NC
Margaret J, Kenney, Professor of Mathematics, Boston Col1ege, MA
ltllfiatt olt your lniltd? Thomas D. Rossing, Professor o{ Physics, Northern Illinois University, IL
dexander Soi{er, Prolessor of Mathematics, University of Colorado-Colorado Springs, CO
Write to us! We want to know what you Barbara I. Stott, Mathematics Teacher, Riverdale High School, LA
lhink ot Quantum. What do you like the Catol-ann Tripp, Physics Teacher, Providence Country Day Schooi, RI
most? What would you like to see more
of? And, yes-what don'tyou like about Quantum IISSN 1048-8820) is published bimonthlyby the Subscription lnformation: North America
Quantunl? We want to make it even bet- National Science Teachers Association in cooperation with
Springer-Verlag New York Inc. Volume 4 {6 issues) will be Student late: $15; Personal rate (nonstudentl: $20, Insti
ter, butwe need your help. published in 1993-1994. Qumtum cortains authorized En- tutional rate: $34; Single Issue Pdce: $5. Rates include
glish-language translations ltom Kvant, a physics and postage mdhmdling. (Caa&an customers plmse add 7%
mathematics magzine published by Qumtum Bureau of GST to subscdption pdce. Springer-Verlag GST registra-
tttlhat$ ottr addre$s? the Russian Academy of Sciences, as well as origlnal ma- tion numberis 123394918.) Subscriptionsbegin with next
terial in English. Editorial olfices: NSTA, 1840 Wilson published lssue (backstarts may be requested). Bulk rates
Boulevad, Arlington, VA 22201, telephone (703) 243-7100. Ior students are available. Send orders to Quantum,
Quantum Production offices: Springer-Verlag New York, Inc., 175 Springer-Verlag New York, Inc., P.O. Box2485, Secaucus,
Fifth Avenue, New York, M 10010-7858. Supervising Pro- NI 07096-2485; or call 1-8O0-SPRINGER {ir New York,
National Science Teachers Assoc. ducrion Editor: Madeline R. Ituuer. call [201] 348-40331.
1840 Wilson Boulevard All Countries Outside Nonh America
Advertising:
Arlington, VA 22201-3000 Subscription rates in U.s. curency as above (a1l rates
Advertising Representatives: lWashinSon) PauJ Kmtzler calculated in DM at the exchange rate current at the
1202) 328-5800; lNew Yorkl Patrick Ferenczl2L2) 460-1575t time of purchase) plus postage and handling. SAL (Sur-
and G. Probst, Springer-Verlag GmbH & Co. KG, D-14191 face Airmail Listed) is mandatory for ]apan,
Be a lactor in the Berlir, FRG, telephone {0) 30-82 07-1, telex 185 41 1. Tndia, Australia, and New Zealand. Cus- Oz
tomers should ask {or the appropriate price

OUANTUM Second class postage paid at New York, NY, and additional
ma.iling ofices. Postmmter: send address changes to: Quan-
rum Springer-Verlag New York, Inc., Jownal Fulfillment
Seryices Depdtment, P. O. Box 2485, Secaucus, Nrl 07096-
list. Air mail delivery to all other countries
is available upon request. Orders may be
placed through your bookseller or directly
through Springer-Verlag, Postfach 31 13 40,
eqllalioll! 2485. Coplright O 1994 NSTA. Printed in U.S.A. D-10643 Berlin, FRG.

OUAr'lTU]ll
.:":::,, .,r:e . ':;l i :
'j..:; ,-fr1.'lj;.''
';? :i: :, :a- .1: .r,..:j.

i,...:.::iiN l
:,:

...:: .l,,.," *,.


''..;-nir'.:'
..::! it . .. ,'..
'-:r.$"j:.*{.. r:::' :Ii:'
rit$
i{i\
$\$;
-ii

'' l::.,,:,;;:,+l::l
:r'a jL t-i,',.I j,li}irI!

$ffi
rlrr-jj:ri**.
1i:i:.ili&,}
l
C
t,:ia*tr
()
g
(c
$il
c
<

-s
Thl'ee palh$ Io [Ull. Fermal-Euler

Let Lagrange, Zagier, and Minkowski be your guides

by Vladimir Tikhomirov

OOK AT THE FIRST FEW have celebrated the 350th anniver- Starting from the assumption that the
prime numbers greater than 2: sary of this remarkable result. On conclusion of the theorem is not valid
3,5,7, rL, 13,17,19,... . The Christmas Day in 1540, the great for a certain prime of the form 4n + l,
numbers 5, 13, and l7 canbe rep- Pierre Fermat (1601-1565) wrote a he proves that it must be wrong for
resentedas the sums of two squares- letter to the renowned Mersenne, a some smaller number and proceeds
faithful friend of Descartes and the allway down to the number 5, thus
5=12+22
main intermediary in the correspon- arriving at a contradiction (since the
13 :22 + 3i
dence of scientists of that time. He theorem is true for 5).
17=12*42'
informed Mersenne that "any prime The first complete proofs were
the other four numbers (3, number that yields a remainder of found by Leonhard Euler (1707-
-while
7 17, 19) cannot. (Check it yourself one when divided by four is 1 783 ) betwe en 17 42 and 17 47 . Erler
, !)
Is there afly way to tell one sort of uniquely representable as the sum of held Fermat in the highest esteem
number from the other without a two squares."l At that time there and, ceding priority to his predeces-
brute-force search through all pos- were no mathematical iournals, so sor, created a proof that elaborated
sible breakdowns? And how can this mathematicians exchanged infor- the idea in Fermat's letter. Giving
difference be explained? The answer mation by mail. In general they sim- credit to both great scholars, we now
is given by the foiiowing theorem. ply announced their results and call this statement the Fermat-Euler
THsoR.sr{. A prime number didn't include any proofs. Theorem,
greatu than 2 is representable as However, almost 20 years after There is a feature inherent in al-
the sum of two squares if and only writing to Mersenne, Fermat de- most any beautiful mathematical
if its remainder upon division by scribed his plan of attack in proving result (as weli as almost any beauti-
four is one. the theorem presented above. In a ful but forbidding summit): many
(Indeed, 5 : 4. l+ 1, 13 = 4. 3 + l, letter to Carcavy2 sent in August paths lead to it. We can approach it
L7 :4.4 + l, whereas 3 = 4.0 + 3,7 1659, he writes that his proof is based from different sides, and all the
=4.1+3, 11 =4.2 *3,....) on the method of infinite descent. paths give sheer delight to those
Problem 1. Prove the "only if" 1In this article we who aren't afraid to take them.
won't touch on
part of the theorem: any prime (ex- uniqueness, which was established The Fermat-Euler Theorem viv-
o cept 2) equal to the sum of two idly displays this wonderful feature,
long before Fermat. See the solution to
o squares can be written as 4n + | Ior problem Ml15 in this issue.-Ed. and I'm going to demonstrate this
(o 2After his death, Mersenne's role as
o some integer n. below.
a scientific intermediary was played
e Who first discovered this math- We'll ascend to this peak, discov-
by the Royal Librarian, an amateur
!o ematical phenomenon? There is evi- mathematician and Fermat's friend ered in the 17th century/ in three
=
o
dence that not long ago we could Pierre de Carcavy (d. 1684).-Ed. different ways. One of them was

OUIiITllIil/IIITUflI
{ound in the l8th century, another than . The largest of the numbers this reasoning is to consider the
in the 19th century, and the third ^1pis greater than.Jr
k (orm) - 1, so the transformation / of the set S of all
only recently, in our own century. number of such pairs is geater than positive integer triples satisfying
lq p - 1) + 112 : p. Therefore (by the our equation that swaps y and z-
Lagrange$ pl'ool pigeonhole principle applied to the Ik, y, zl : k, z, yl-and note that,
The first proof (with certain pairs as pigeons and the remainders first, it's an involutioa-that is,
modifications) is given in almost modulo p as pigeonholes),3 there are when applied twice it takes us back
every textbook on number theory. at least two different pairs (k, mrl arrd to the starti second, its fixed points
It's based on the following lemma. (k, m2l such that the remainders of k, (*, y, ,l = I&, y, z) supply the re-
WrrsoN's Lrume. For any prime p + Nm, andkr+ Nm, when divided by quired decomposition of p (since
the number (p - 1)! + 1 is divisible p are the same. Then a + Nb (where they are characterized byy : zl; and
bv p. a: kr- k2, b - mt 4zl is divisible third, the number of points that
In order not to digress {or the proof byp. Note that lal . ",lp andlbl . I p . aren't fixed is even, because they
of this auxiliary (but very usefill fact, Now, a2 - : la - Nbl(a + Nbl can be arranged in pairs such that
I'1I demonstrate its main idea using is divisible by p, arrd since M = -l
^Pbz either element of each pair is the
the prime number 13 as an example. (mod p), a2 + bz is also divisible by image of the other element. Of
For any integer from 2 through 11, p-that is, a2 + bz : rpfor some posi- course/ the last statement holds for
let's find the factor whose product tive integer r (r + 0, because other- any involation of any finite set.
with this integer gives a remainder wise the two pairs above would be And now let's consider the trans-
of I when divided by 13, and collect the same). It remains to note that az formation B of triples (x, y, zl defined
such pairs of factors in the factorrza- +b2 <2p, which means thatr = 1 and as follows: B(x, y, zl: ({,1/, y'l,where
tion of (13 - 1)! together: a2 + b2: p, completing the proof.
(1)for x<y-z:d:x+22,
(13-11! =12! t' =2,y, =y_x_z;
agiel'suoo
= 12 . 7N3. e)(4 . 10)(s . 8)(5 . 11) . 12
Another proof, which is due to l2l for y - z < x < 2y { : 2y - x,
(where 2. 7 = 14=13 + l, 3. 9 = 27 = the contemporary mathematician t':y,2,=x_y+Z;
2. 13 + L, 4. 5. 8 : 40 =3. 13 + 1,
I0: D. Zagier, completely stunned me:
(3)forx>2y:d=x-2y,
6 . 11 : 66:5. 13 + 1. It follows that the result seems to emerge miracu- 5/=x-Y+2,y':Y-
the remainder of. l2l upon division lously, out of thin air. Like /, this transformation con-
by 13 is l}-thatis,l2l + 1 is divis- Our goal in reproducingZagpe{s sidered on the set S is also an invo-
ible by 13. The general case is proof will be to show that for any lution of S. First of all, it maps the
treated likewise. prime p of the form p = 4n + I the set into itself, because it preserves
S
Ptoblem 2. Prove that for any equation in positive integers the valuex2 + 4yz.lndeed, take case
prime p the integers 2,3, ..., p - 2 1, for instance:
can be paired so that the product of
*+4yz=p
the numbers in each pair la, bl gives has a solution (x, y, zl with y = z.
{2 + 4fz' : (x + 2,212 + 4z(y - x - zl
: * + 4xz + 4zz + 4zy - 4zx - 422
a remainder of 1 when divided byp. This would yield the representation :*+4y2.
(This is written as ab = 1 (mod p).) p : * + 4J? : * * (2ylz,which proves
Use this fiact to prove Wilson's the theorem. Verification in the other two
Iemma in the general case. We'll prove the existence of such cases is iust as straightforward. Fur-
From Wilson's lemma we derive a solution in a rutherbizarre way: by ther, if (X', y', z'l = B(x, y, zl, t};'en
a corollary. proving that the (obviously finite) B(1, y', z'l = (x, y, zl. This is also
Conorrenv. If the number p: 4n number of solutions to the above verified by direct calculation. For in-
+ f. is prime, then [(2n)!]2 + 1 is di- equation is odd. How is the oddness stance, if x < y - zt we must appiy
visible by p. connected to the existence o{ the the equations in case 1: they yield:1
To prove it, we rewrite (p - 1)! + 1 solutions we need? All the solutions : x + Zz > 2z = 25/, so (f,, y,, 2,,1 =
as (4nl!+ 1 = 1 . 2. .... 2n. (2n + Il. withy + z can be arranged in pairs by Bld, y', z'l rrust be computed using
... .(anl + 1 = 1 .2. ... .2n.(p -2n1. swapping y arrd z; if.lx, y, z) is a so- the equations in case 3, and we get
(p -2n- 1). ... . (p
=r) * 1 : (znlt. lution, then (x, z, yl is a solution as
d'=d -zt':x+22-22=x,
(-L12"(2nll + I =|2nltlz + I (modp), well. So the number of these solu-
and note that the left side is divisible tions should be even, and the totai
y''=1-t'*z'=x+22*z+
bv p. number can be odd only if there's a
(v-x-zl=Y,
_r, _J
Denote (znllby N. Then our cor- solution withy: z.
olIary means that M: -l (mod p). The modern way to articulate E*r*irrrtion of the other two
Now we have to overcome the ma- 3For an explanation of the cases isleft to the reader. After such
jor difficulty. verification, we conclude that B is
pigeonhole (or Dirichlet) principle, see
Consider all the pairs (k, m) of non-
"Pigeons in Every Pigeonhole" in the an involution of S.
negative integers that are no larger lanuary 1990 issue o{ Quantum.-Ed. What about the fixed points of B?

I'llY/JUilt lSS4
Looking at the definition, we see
thatincases 1 and3,1 >xord <x,
respectively/ so a {ixed point can
arise only in case 2, which yields x
=/ =2y-x1 otX= jt.
Conversely/ you can see at once
that any triple of the form (x, x, zlis
preserved under B. But only one of
these triples belongs to the set S. of
positive integer solutions to our
equation: If p = * + 4xz : x(x + 4zl,
then x = I (since p is a primel and z
= n (recall thatp = 4n + 1). Thus, the
involution B of the set S has a
unique fixed point (I, l, nl, and
therefore, as we've seen, S consists
of an odd number of triples, which
is what we set out to prove. Figure 2
Iulinfiowslri$ pl'ool
The (slightly modified) proof by of the distance OP, P = (x, yl, canbe
Hermann Minkowski (1864-190911, foundfrom the triangle OQP, where ! oo. o, .
= ! nG-li
which I'm going to present now/ Q = (x, 0): in this triangle, OQ = 22 "rn(ze) "in(ze)
staggers the imagination perhaps lxlJa, QP = lyllc , and the angle at
even more. Q is o or 180' - cr, depending on the =;![41- "-"-
Minkowski's proof begins with a signs of 5 and y. However, no mat-
result that doesn't seem to have any- ter what these signs are, the Cosine
thing to do with the Fermat-Euler Law always yields =!.tr"_a,
Theorem. 2
Tsponrm. Let a, b, and c be any oPL : oQ2 -zoQ.oP cos(zQl+oPz
integers satisfying a, 0 and ac -b2 = a* + 2bxy + c1fl. =1
2
= L. Thenthe equation ax2 + 2bxy +
cf = t has aninteger solution (x, y). The points with integer coordinates So ndzlS < lf 2, or & . +ln < 2. Since
Proof . The expressional,2 +2bxy form the integer grid with respect to & an
is integer (dz : amz + 2bmn +
+ ct' :1 can be viewed as the square our coordinate system (fig. 2), and cnzl, we get d = which proves Min-
1,
of the distance from the origin O to we have to prove that there is a node kowski's theorem.
the point P with coordinates (x, y) in in the grid at a unit distance from But what relationship does this
a certain coordinate system (not the origin. marvelous theorem have to Fermat
necessarily rectangular). To con- Let d be the smallest distance from and Euler? The most directl
struct such a system/ draw the axes at the origin O to another node, and let By the corol1ary of Wilson's lemma
the angle a defined by cos a: b I (m, nlbe a node at a distance d from proved above, we know that thenum-
^Fac
(this is possible because ac > 0 and O. Since the distance from (x, y) to berM + 1, whereN= [(p - 1]/21!, is di-
lbl Ja"l < 1, since ac = b2 + I > b2l. (x1, y1l is equal to the distance from visible byp, don't we? Well, now let's
Choose the units of scale on the x- (0, 0) to (*r- *, yt- yl, the distance apply Minkowski's theorem to the
and y-axes equal to 1E and ,lZ, rc- between any two nodes is no numbers a=p,b =^/,c =(b2+llla.
spectively (fig. 1). Then the square smaller than d. Therefore, the The theorem says that for certain in-
circles of radius dfL centered at all tegers marrdn
the nodes of our grid do not overlap:
l=amz+2bmn+cnz,
if two such circles, with centers A
andB, have a common interiorpoint so
C, thenAB < AC + CB < dlz + dl2 =
p = a = a2m2 + 2abmn + {b2 + Llnz
d. As is clear from figure 2, the area
= (am + bnl2 + n2
covered by these circles in the tri-
(1,0) 180'- o angle with vertices O(0, 01, A(1,0l1, p
is, is the sum of two
and B(1, 1)is half the area of one
-that
squares. And, once again, the theo-
circle-that is,nd2f 8. And this is only remisproved! O
a part of the triangle's area, which ANSWERS, HINTS & SOLUTIONS
Figure 1 equals ON PAGE 60

0lJlilIUl'l/ttATUBt
tollotnl the houncinu huclryhall
On fullerenes and other carbon structures

by Sergey Tikhodeyev

VERYBODY KNOWS THAT gen and two carbon atoms. Here we'11 make use oi : --.r's
carbon is one of the most com- Another example of atomic struc- theorem: ior any conves : -.:c-
mon elements. But did you tures where each carbon atom has dron, where C is the numbe r - :.rr-
know that carbon atoms are a three neighbors is graphite, the sec- ners, F is the number of iaces =:d E
first-rate building material for con- ond natural form of carbon. Graph- is the number of edges,
structing a wide variety of crystals ite is a layered substance whose
and molecules? The record in the structure is based on planes in C+F-E:2. 1)

hardness department belongs to dia- which the atoms sit at the corners of For more complicated polr-h. ::.rns
mond, which is one of the crystal- regular hexagons, forming a kind of equation {1) must be modrr-.: by
line forms of carbon. The complex honeycomb. Actually, tro other introducing a new concept hai':tr to
organic molecules known as pro- structure is possible when each car- do with the number of "hani-t. in
teins-whose atomic "skeletons" bon atom forms valence bonds with a polyhedron. For a torus _i = 1,
are atoms of carbon and nitrogen- only three neighbors and all the at- which means that it has one hanJle,
form the basis of all living things. oms are arranged in the same way. while for a convex polyheir.--n g
The great variety of atomic struc- Fortunately there is a mutual atttac- equals zero.
tures made of carbon is due to the tion between adjacent planes, which The generalized Euler theorerl
fact that carbon-an element of connects the carbon layers to form yields
group IV of the periodic table-has a crystal of graphite. These atttac-
four electrons in its outer valence tive forces (known as van der Waals
C-F-E=2-2g. ,2'
shell and can form valence bonds forces, which decrease with distance It's surprisingly easy to prove
with four, three, or two neighboring as r7l are much weaker than the in- ecluation {2). We need but note that
atoms. If a carbon atom has four teraction between adjacent carbon each handle of a polyhedron and the
close neighbors, the resulting struc- atoms in the same layer. Thus, polyhedron with its handles cut off
ture is three-dimensional. One ex- graphite isn't strong mechanically, satisfy equation (11. When we paste
ample of such a structure is the dia- and so it can be used to make pen- each handle back onto the polyhe-
mond crystal, in which each carbon cil lead. The carbon planes them- dron, the four glued faces disappear,
atom sits in the center of a regular selves, however, are as strong as dia- while the difference C - E doesn't
tetrahedron whose corners are the mond. change.
neighboring carbon atoms. The question arises: can we make Now we have all we need to de-
If there are only two adiacent at- something more interesting from duce the architectural rules for
oms/ a one-dimensional linear struc- carbon atoms than just a flatlayer in constructing polyhedrons out of
ture appears-long polymer mol- a graphite crystal-say, a polyhe- carbon atoms. Suppose we want to Y
ecules are examples of this type. a
dron? Since each carbon atom must construct a closed polyhedron with j-
a
When there are three neighbors, the have exactly three neighbors, the hexagonal faces on1y, and that there 0)
atomic structures include flat re- {oliowing geometrical problem ate n(1 of them. Since three faces .C
O
gions. For example, in the flat mol- arises: how to construct a polyhe- come together at each corner, and o)

ecule of benzene CrHu each carbon dron in which exactly three edges because each edge belongs to two (E
a
atom forms bonds with one hydro- come together at each corner? faces simultaneously, we get -a

ilAY/Jlltllt 1gg4
r ii-:'l
r :u.
",

:
t

i< i
a J

l .i
I
C
0)
c
O
a)
CO
o-
n
i
5n. F 6n- ecules have not been found either,
=
-i, = 176, E - -----L. (3) though the term "matryoshka" has
" aheady been coined for them (after
Substituting this equation into the Russian nesting dolls). It's also
equation (2) results in I = 1. Thus, curious that almost 20 years before
using hexagons only we can con- the discovery of hollow molecules,
struct a polyhedron that is topologi- their existence was hlpothesized by
cally equivalent to a torus. David E. H. |ones, who for many
If we want to construct more di- years wrote the famous Daedalus
verse structures, hexagonal "graph- column in the journal New Scien-
itelike" faces won't suffice. Sup- tist.3
pose, in addition ton. hexagons/ we At present the dimensions of
have nu pentagons and n, hepta- the buckyball are well known: its
gons. Repeating the reasoning Figure 1
radius is 0.3512 nrn, the length of
above, we get Thr ee- dimensionaT structur e of the short bond (which separates
buckminsterfuller ene C (buckyb all).
the hexagons) is 0.1388 nm, and
ns- nt = 612 -zsl (41
polyhedron.
^n
Carbon atoms sit at the-corners of the
that of the long bond (the same ior
So if we're interested in convex poly- hexagons and pentagons ) is 0 . 1 +3 3
hedrons (S = 0) only, we can do with- ized by a laser beam in a stream of nm. The numbers are very similar
out heptagons, but in that case we helium. The most stable was the Cuo to those for graphite. Quantum-
must add precisely 12 pentagons. molecule, and the discoverers mechanical calculations show
(Incidentally, it was Euler himself thought that it had the form of a that the valence electrons must be
who first noticed and proved this hollow soccer ball (fig. 1). In honor distributed more or less homoge-
fact.) To construct more compli- of "Bucky" Fuller the researchers neously in the spherical sheil rvith
cated structures/ we need heptagons called their molecrle a buckyball a width of approximatelv S a.u. (1
as well. (which soon evolved into the more a.u. [atomic unit] = 0.0519 nm, the
Thus, we have deduced the basic stately buckminste{ullerenel, ard Bohr radius). An electron-ire e cav-
rules for constructing complicated they named the whole class of Cuo- ity about 2 a.u. in radius is iormed
three-dimensional structures from Iike molecules fullercnes. in the center of the buckvball. So
carbon atoms. Large structures simi- It turned out that fullerenes the Cuo molecule re se mbles a little
lar to these exist in nature. For ex- arert't all that rare: there are plenty empty cage.
ample, the skeletons of rafiolarians- of them in lampblack, gas soot, The existence oi the cavit)- rnside
the simplest organisms among and other substances resulting the buckyball appears to hare been
plankton-as well as many viruses from incomplete combustion. The proved experimentally b1, muon
are constructed in just this way.r problem is isolating a pure sample analysis. (The sensor in this method
These structures are also familiar in of such molecules-that is, obtain- is muonium, which is something Like
architecture. The geodesic domes of ing a substance that contains al- the hydrogen atom, but instead of a
R. Buckminster Fuller spring to most nothing else. proton it has a muon-an elementary
mind.2 To drive home the fact that In the molecule buckminster- particle with a charge o{ +e and a
such structures aren't rarities, pick fullerene C.o, the comers are carbon mass of 200 m". The properties of
up an ordinary soccer ball, which is atoms and the edges are theirvalence muonium depend strongly on the
stitched together from 20 hexagons bonds. There are no free valences in electron density at its location. The
and 12 pentagons. But the question Con, which explains its high chemicai researchers managed to place muo-
remains: is it possible to build such and physical stability. The Cuo mol- nium inside a "fullerene cage" and
a structure out of carbon atoms? ecule is the most symmetrical and show that the properties of free and
stable among fullerenes. The next (in captive muonium are virtually the
Fullel'enes and fulleriles order of increasing numbers of carbon same.
In1985 H. W. Kroto (Great Brit- atoms) stable molecule Crn has 25 Free buckyball molecules attract
ain), |. Heath, S. O'Brien, R. Curl, hexagons and the same 12 pentagons. one another with the same weak
and R. Smalley (United States)found It is formed less often than C.o. The van der Waals forces that appear be-
that fairly stable molecules consist- highly symmetrical molecules Cro6, tween the carbon layers in graphite.
ing of a large 132-9Ol and always Cuoo, and Cruo, which are thought to Because of this attraction, bucky-
even number o{ carbon atoms were be stable as well, have not been found
3The ideas expressed by "Daedalus"
formed when graphite was vapor- yet.
rn 1966 are not so much {antasy as
lSee Growth and FormbyW. It's curious that there is nothing scientific prediction that has come
to prevent Cseomolecule from hav- true. See The Inventions of Daedalus
D'Arcy Thompson or the book cited in ^
{ootnote 3.-Ed. ing a Cuoo molecule (or other fuller- by David E. H. fones (San Francisco: W.
2See Gallery Q in this issue.-Ed. ene)inside it. Such compound mol- H. Freeman 8r Company, 1982).-Ed.

10 illY/Jtlilr roo4
called cluster molecules (consisting these crystals produced the first re-
of a small number of identical at- liable information about the shape
oms) had reached an advanced state and properties of fullerenes.
of development. Usually the num- Later this method was perfected.
ber of atoms in such clusters is It turned out that in order to obtain
rather arbttrary. However, the very the fullerene-rich carbon soot, it was
first experiments with carbon pro- convenient to use an electric arc be-
duced a surprise: large carbon clus- tween carbon electrodes. When the
ters with an odd number o{ atoms monomolecular fractions were ex-
were never formedl At first this fact tracted from the fullerene solution,
was explained by the formation of a purity of 99.99"/. was achieved.
polymer chains of the type [-C =C-]. Yet the basic production stages re-
Figure 2 H. W. Kroto and his colleagues were mained the same: evaporating
Sftucture of pure fullerite-the crystal the first to provide a correct interpre- graphite electrodes in helium, then
fu o m buckminst e{ull ercne s.
f or m e d tation, though they couidn't per- dissoiving the soot in an organic sol-
One elementary cell of the crystal is form a reliable structural analysis- vent. So the price of the final prod-
shown. The buckyballs are situated in
the corners and in the middle of the
they had too few fullerenes. Their uct in this improved production pro-
cube faces. explanation remained a hypothesis, cess depended only on the cost of the
and the fullerene an exotic toy for electricity consumed: about 5 cents
balls crystallize at room tempera- theoreticians, until the summer of per gram of fullerenes!
ture into a fragile yellow-red crystal 1990, when a revolutionary event How are fullerenes formed when
with a face-centered cubic lattice. occurred: a method of large-scale graphite is evaporated, and why is an
The new crystal was named production of fullerenes was pro- atmosphere of helium essential? As
fullerite. The distance between ad- posed. was mentioned above, graphite
jacent buckyballs in this crystal at The solution was found rather consists of flat layers of carbon
room temperature is 1.00 nm. Pure surprisingly by a group of American hexagons. Fairly small carbon
fullerite that contains nothing but astrophysicists-speciaiists in the clusters seem to be formed ini-
buckyballs is a dielectric. area of cosmic dust: W. Kretschmer, tially during graphite vaporization
Figure 2 shows an elementary cell D. Huffman, and their students L. in the electric arc. They are linear
of fullerite. Buckyballs play the Lamb and C. Fostiropoulos. As {ar and have plenty of free bonds. In
same role in a fullerite crystal as at- back as 1983 Kretschmer and Huff- the cooling atmosphere of helium
oms in an ordinary crystal. Many man had tried to experimental\y re- these clusters form graphite "fish
characteristics of fullerites (for ex- produce the natural conditions scales" resembling scraps of graph-
ample, the electron spectrum) can needed for the formation of cosmic ite planes (see figure 3 on the next
be calculated with great accuracyby dust. To this end they evaporated page).From the energy standpoint
treating the buckybalis as if they graphite samples heated by an elec- it's advantageous for these fish
were atoms and applying traditional tric current in gaseous helium. A{ter scales to change their shape (be-
methods of calculating the proper- the discovery of buckminster- cause they have free bonds at the
ties of crystals. fullerene, the researchers decided to edges, which disadvanta-
The new carbon molecules repeat their old experiments. To ex- geous)-they form ^reseveral penta-
(fullerenes) and the crystals made tract the spherical molecules they gons instead of hexagons and bend
from them (fullerites) are the third expected to produce, Kretschmer because of it (the ends of the free
form of naturally occurring car- and Huffman took advantage of the bonds come together and thus
bon-or in scientific terms, the old chemical rule: dissolve a sub- lower their energy). Since it is en-
third allotrope of carbon. The first stance in a similar one. They dis- ergetically disadvantageous for
two ailotropes-diamond and solved the lampblack (formed by two pentagons to be next to one
graphite-have free bonds that carbon vaporization) in benzene, another, the open ends must come
grab stray atoms (for instance, hy- which also consists of closed mo1- together in the course of this evo-
drogen atoms). Such is not the case ecules. A yellowish or reddish liquid lution, the structure that emerges
with fullerenes and fullerites, since was produced whose color depended automatically is-the soccer ball!
they don't have any free bonds, so on its concentration. Soon it was (The buckyball is a minimal
among carbon allotropes, they are clear that the dissolved fraction of fullerene, in which the pentagons
the purest. Iampblack was composed of mol- have no common edges.)Thus, if a
ecules of Cuo 175%1, Crol23%), and fullerene grows slowly enough, it
llow l0 huild a fuller8ll0 even larger fullerenes (2%1. Nter the must necessarily become a
By the mid-1980s, when fuller- benzene evaporated ol{, small buckminsterfullerene. Under actual
enes were being discovered, meth- fullerite crystals remained on the conditions, of course, the shell can
ods for experimentally producing so- bottom of the cuvettel Analysis of close up before the ideal soccer-ball

OUA[ITUIiIi IIATtlRI 1 1
Figure 3
Consecutive stages of formation of buckminsterfullerene C60 during graphite the new technology: not micro-, but
vaporization in a helium a.tmospherc. nanoelectronics. The characteristic
lengths of the elements are nano-
structure is formed, and then other soccer bal1, it's the smallest and meters. In nanoelectronics the most
structures arise. strongest ball bearing as well. interesting objects from the view-
It's not accidental that inert he- The chemical stability and hol- point of possible applications are
lium serves as the cooling bath low structure of fullerenes suggest quantum dots-microcrystals or
when fullerenes are grown. ("Bath" ways in which they might be used in other formations incorporated into a
here is a term of art, not figurative chemistry, microbiology, and medi- nanoelectronic circuit-that can re-
language.)This is due to the fact that cine. For example, fullerenes seem tain (localize) electrons. Such dots
helium doesn't saturate the free car- to have no match as a packing ma- have a number of unique optical prop-
bon bonds, which lets the carbon terial for individuai atoms. Scien- erties that make it possible to use
fish scales close in on themselves. If tists have learned how to pack them either as control elements in
there were hydrogen atoms/ for in- fullerenes even with such heavy at- fiber-optic communications or as the
stance, in the combustion atmo- oms as lanthanum and uranium. basic processor elements in the opti-
sphere, they could saturate some of Fullerenes filled with such atoms cal supercomputer cuffently being
the free bonds and destroy the sym- open unexpected possibilities for designed. Fullerenes are in many re-
metry of the curling fish scale. The chemists. For instance, fullerenes spects ideal quantum dots. Adding to
opposite sides wouldn't be able to can be used to pack and transport our list of records, we can say that
come together. As a result, struc- not only atoms but entire molecules fullerenes have a good chance o{ be-
tures that resemble shells would to the required destination. Not a coming the smallest microchip in a
grow instead. It's interesting that bad idea for pharmacists and micro- computer nanoprocessor.
this very process underlies the for- biologists! So the world's smallest And, last but not least, high-tem-
mation of carbon soot during incom- soccer ball is also the world's small- perature superconductivity.4 Fol-
plete combustion in ordinary air. est packing box-or should I say, lowing the discovery of high-tem-
pillbox. perature oxide superconductors in
T]te smalle$tfii$, lhe $lnalls$lfial . . . Nowadays molecular biologists 1986 by Bednorz and Mriller, new
There are many proposed appli- engaged in genetic engineering use substances have continually been
cations for fullerenes. For example, viruses (many of which, by the way, tested for possible superconductiv-
they might be used as the basis for are shaped like buckyballs). If scien- ity. A pure fullerite, of course, was
producing unique lubricants. As tists manage to use fullerenes to an unlikely candidate for supercon-
was mentioned above, the Con mol- transport the necessary organic mol- ductivity, since it's a dielectric (as
ecule is very strong both chemicaliy ecule to a particular site in a protein, was mentioned above). But everyone
and mechanically. Its mechanical it would mean the creation of an knows how to turn a dielectric into
strength was tested as follows: a artifrcial, specialized virus, and a conductor: you dope it. Atoms of
flux of buckyballs was accelerated again-the smallest one (for the ben- a suitable impurity can, for ex-
to a velocity of 30,000 km/h (about efit o{ life on Earth, we all hope). ample, be donors of the electrons
orbital velocity) and then sent Now let's talk about microelec- needed to conduct electric current.
crashing into a steel waI1. The tronics. It's well known that the pro- It was doping that produced the first
fullerenes bounced off, and were cess of miniaturization of electronic high-temperature superconductor
none the worse for the wearl Such chips has recently reached its natural Lar_,Sr,CuOo (here x = 0.1-0.2 is
strength is just what one wants in a limits-that is, molecular and atomic 4See
"Meeting No Resistance" in
lubricant. So not only is the dimensions. As a matter ol f.act, arr- the September/October 1991 issue of
fullerene Cuo the world's smallest other term is used more and more for Quantum.-Ed.

12 llAY/Jtlilr 1 gs4
Energg Sources
ond Natural Fuels

Figure 4 Figure 5
Cry stal structur e of sup erconducting Elementary cell of a schwartzite-an
fullerite K.C^n. The little balls corre- infinite surface made of carbon
spond to atoms of the doping impurity atom* (Six of its 24 heptagons arc
(potassium), completely visible her e. )
the concentration of the impurity- But what about heptagons? Such
strontium, in this case). structures have not yet been obtained
In the beginning of. l99l a new experimentally, but theoreticians are
discovery grabbed the scientific abeady modeling their properties on
bg Bill Aldridge, Linda Crow,
headlines: A. Hebard and his col- computers. It f.act, heptagons offer
and Russell Aiuto
leagues discovered that a fullerite even more possibilities than fuller-
This book is a vivid exploration of
doped with potassium-K.Cuo- enes. For example, a carbon "sponge"
energy, photosynthesis, and the
became a superconductor at 18 K has been found whose complex sur-
(-255'C). This temperature wasn't formation of fossil fuels. Energg
face consists of hexagons and hepta-
Sources and Natural Fuels
a record, but when rubidium was gons that separate three-dimensional
follows the historical unraveling
substituted for potassium, the su- space into two subspaces.
of our understanding of photo-
perconductivity transition tem- These structures were named
synthesis from the 1600s to the
perature jumped 28-29 K. Before schwartzites af.ter the German
early part of this century. Fifty-
the race began in 1986 to find mathematician who was the first
one full-color illustrations woven
high-temperature superconduc- to study such surfaces at the end of
into innovative page layouts
tors/ no one had found a material the last century. Figure 5 shows
bring the subject to life. The
that was superconducting above but one elementary schwartzite illustrations are by artists who
24 K. Now materials that are su- celi. The entire crystal is obtained
work with the Russian Academy
perconducting at 126 K have been by an infinite repetition of such
of Science. The American
found, and there are reports of cells. Schwartzite has the same type
Petroleum Institute provided a
even higher temperatures. of crystal lattice as the cubic face- grant to bring scientists, engi-
Superconductors based on C.o centered fullerite in figure 2. The el-
neers, and NSTA educators to
molecules appear to enjoy superior ementary cell has 215 corners,24
create the publication. This
stability due to the strength of these heptagons, 80 hexagons, and 3 group worked together to
molecules. This is what makes handles. Note that in this periodic
develop the student activities and
them stand out from the oxide high- structure the handles connect adja-
to find ways to translate indus-
temperature superconductors. The cent crystal cells; in figure 5 each
trial test and measurement
crystal structure of a superconduct- handle is cut in two and only half of
methods into techniques
ing fullerite is shown in figure 4. it is shown. appropriate for school labs.
The doping impurity occupies posi- Again, such stryctures have not (grades 9-10)
tions in the crystal between the yet been observed experimentally.
#PB-104, 1993, 67 pp. US$12.95
fullerenes. But if researchers manage to synthe-
size this new allotrope of carbon,
they would obtain a substance with
To Order, Call
$till annilher lol,m olcailolt:
unique mechanical, physical, and
sclttum[ile chemical properties. o r-800-722-NSTA
So, we see that cellular structures
made of carbon pentagons and hexa- The author is grateful to Grigory
gons have been discovered and are
now the subject of intense research.
Kopelevich, who prepared the computer-
generated illustrations for this article.
@HT:'
OUA[IIUill/TIATllRI t3
Seeing is
Believing

LEt NSTAS
Project Earth
Science
series open
your eyes to
the skies.

The hands-on, teacher-tested activitie in Astronomy and Meteorologlr-the first wo books in the National
s
Science Teachers Association and BP America's Project Earth Science series-bring the sometimes daunting
concepts of astronomy and meteorology down to Earth. Background information, supplementary readings, and
suggestions for integrating other disciplines provide a framework for launching a successful introduction to both
subjects.
ln Project Earth Science: Astronomy students will discover Earth's uniqueness by examining it as a part of the
whole Solar System. How did the planets form? Are we seeing a star's present or past?'Why is Earth's distance
from the Sun so important?
Learn to read weather maps and do forecasts; model the water cycle on a tabletop; and use real data to track
Hurricane Andrew as it moves oyer ocean and land in Project Earth Science: Meteorologt. Other activities explain
why the equator heats more quickly than the poles and why dust is needed for cloud formation. Both books
provide supplemental readings for teachers (and for interested students).

Project Earth Science: Meteorology (grades 5-10, 1994,230 pp.) #PB103X $18.50
Project Earth Science: futronomy (grades 5-10, 1992,160 pp.) #PB090X $18.50

VISA, MasterCard, Discover,


To 0rder Call 1-80F722-NSTA & Purchase orders welcome
BRAINTEASERS

Jusl lol' Ihe lun ol il!


8111
Breaking even. According to a corltractt a worker is to be paid 48 francs
for each day worked and is to give up 12 francs for each day not worked.
A{ter 30 days the worker is owed nothing. How many days did the
worker work during these 30 days? (Etienne Bezout 11730-l793ll

81 12
What's wrong! Once I found a
strange notebook. A hundred
statements were written in it.
They said:
"There is exactly one wrong statement in this notebook."
"There are exact.ly two wrong statements in this notebook."

"There ,r" one hundred wrong statements in this notebook."


"*r",ly
Which of these statements is true? (A. Savin)

81 13
)Fire!" Which is more effective in extinguishi.ng a fire-cold water or
boiling water? (S. Krotov)

81 14
Arithmetic of lacing. There are
many ways to lace wrestling
shoes, as is shown in the figure,
although we can't see how the
shoelace is arranged inside the shoe. Can you tell exactly how many?
(N. Zilberberg).

81 15 =
o
In half . Cut the figure at right into -U
0)
two congruent parts. o
o
f
o
f
C
a
F
ANSWERS, H//VIS & SOLUTIOiVS O/V PAGE 58

IlJ[ilrl| [li I RAI ilTtIs tns 15


Asll'altue Blnpsror
alld a $lralluE uEllEral
A case study
with two leaps into the past

by lgor Akulich

HENTERENTIUS, ABRAVE managed to ro11 it out of the treasury question: 'Did the sand become
Roman military leader, de- with the greatest difficulty, using compressed under your foot?' " Why
cided to retire, he came to the his spear as a lever). The next coin Aberdeen in 1BB5? That was where
emperor and asked for a pay- was absolutely unmovable. So the O. Reynolds showed that the sand
ment of 5 million brasses (a "brass" total sum he received amounted to actually expands rather than con-
was a copper coin with a mass of 5 g). 252,143 brasses-that is, slightly tracts under our feet, contrary to
The emperor, however, was tight more than ll2O of whathe originally common sense.
with his money, so he decided to asked for. The emperor exulted, But let's not digress too far from
cheat the general. He said, "I while Terentius suffered miserably. our subject. It would be better to ask
wouldn't want you to be content This story, with a lot of interest- a similar question about the
with such a pitiful reward. Go to the ing details and the brilliance typical emperor's award: "Of. aLI the mil-
treasury arrdcarry out onebrass the of its author, can be found in Yakov lions of readers of Perelman's book
first day, a two-brass piece the sec- Perelman's book Mathematics and the thousands of. Quantum
ond day, then a four-brass, eight- Comes Alive.l readers, how many noticed that the
brass, sixteen-brass piece, and so on, behavior of both the emperor and
doubling the value of the coin each lltto lhe distaltl[ast the general described in the story
day. I'11 have a coin minted every More than a century ago, during was at Least strange and absolutely
day of the appropriate size. As long a lecture in Baltimore, Lord Kelvin illogical?"
as you're able to carry the coin out asked the following rhetorical clues- What was so strange and illogi-
on your own, with no help, it's tion: "Of all the two hundred billion cal? We'll soon see.
yours. But as soon as a coin is be- men/ women, and children that First, let's try to estimate some of
yond your power to carryt you'll have walked across wet sand from the values we'Il need. It's clear from
have to stop, and our agreement will the beginning of time down to the the story that a coin with a mass of
be null and void." Terentius was meeting of the British Association in 655 kg was just about at the limit of
very happy. He imagined an enor- Aberdeen in 1885, how many would Terentius's physical resources: a little
mous pile of coins, each one bigger answer anything but 'yes' to the more andhe wouldbe unable even to
than the next, that he'dcarry out of budge it. we'lI estimate this "little
lQuantum readers may already be bit" as 45 kg-that is, assume that the
the treasury.
What actually happened? Teren- acquainted with this outstanding biggest coin that would yield to
Russian popularizer of math and
tius's enrichment lasted only 1B physics. See, for instance, the
Terentius's efforts has a mass of 700
days, because the coin on the 18th Kaleidoscope o{ the November/ fu(which corresponds to a denomina-
day weighed about 555 kilos (he December 1992 iss'ge.-Ed. tion of 140,000 brasses). In addition,

l0 ilflY/Jtlrrr rss{
assume that Terentius's state of J<n+1-1 decreases "irregularly." Here are a
health will allow for daily visits to "^ k-I few values of S: S(4) = 87,381; S(5) =
the treasury and removal of new 97,658; 516l = 55,987; S(71 = t37,257;
coins for ten thousand days (about If n is large enough, we can assume s(81 = 37,499, S(10) = 111,i11, S(20)
25 yearsl. n = [1ogo140,000]= 1690140,000; then = 8,421i S(50) = r27,55ti S(i00) =
So, the emperor decided to lure 10,101 (in the last case, Terentius
his combative general into a trap - 140,000k-l would come for his reward only
that is often called the avalanche. k-1 three times!).
(Indeed, it's hard to think of a better And what happens for k = 1? Per-
name: the coins grow like at ava- ^ lSg,ggg
=14o,ooo.fr. haps in this case the sum S turns out
lanche, and this is what the miserly to be greater than for k = 2? .PJas,
and cunning emperor counted on.) that's not the case. Another factor
In this case, the multiplication fac- This means that S actually decreases comes into play here-the somber
tor is k = 2-that is, each coin is as k increases. In our case, however, fact of human mortality. We've al-
twice as massive as the previous this is not exactly true: the loga- ready estimated the time allotted to
one. rithm isn't very large, so in fact S(k) Terentius for receiving his reward as
And it is this choice of multipli-
cation factor that leads one to sus-
pect that the emperor was a strange
person, because of all positive inte-
gers k, he had chosen the one that
brought the greatest profit to
Terentius!
Consider, {or instance, the case in
which each new coin is three (and
not two) times as massive as the pre-
ceding one. How many coins would
Terentius be able to lift? The value
of the (n + 1)st coin would then be 3"
brasses. The general can lift a coin
that is equivalent to no more than
140,000 brasses. What is the largest
n such that}n < 140,000? Thisn sat-
isfies the inequalities 3'1 < 140,000 <
3n*1, or
1ogr140,000 - | < n( 1ogr140,000.

Since 1o9.140,000 = 10.7 ... , we get


n = 10. So the last coin Terentius
would be able to lift is the eleventh:
on the first day he'd receive 1 brass,
on the second day 3 brasses, on the
third 32 = 9 brasses, and so on. The
total reward would come to 1 + 3 +
32 + ... + 310 = 88,573 brasses. Re-
member, with k :2he received
252,1 43 brasses-almost three times
as many!
A similar situation would occur
for larger values ofk. In general, the
sum S that Terentius could receive
in n days given a fiactor k ecluals
o
S=1+k+k2+...+kn, ro
o
where n = [1o90140,000] (and [a] de- l
notes the greatest integer not ex-
o
ceeding al.By the formula for the =l
a
f
a-
sum of a geometric sequence/ o

0UAt'lIlll'!/ttAIURt l7
10,000 days. Consequently, in the ible the result is; and those who rely advise Terentius to respond to the
case k: 1, he wouid get simply on common sense and don't trust emperor's seemingly attractive offer:
10,000 brasses. anything that contradicts it. Both "Sire! Such a reward is too gener-
Of course, we couid adopt other attitudes are absolutely normal and ous for me. Not only that, it will
limitations instead of 700 kg and natural. lessen the treasury so rapidly that
10,000 days. Then our conclusions Which of the two categories does severe damage will he infiicted on
would have been somewhat differ- our character Terentius belong to? you and on the entire state. So I can't
ent-for instance, with 1,000 kg as On the one hand, he was very satis- agree to such a sharp growth in the
the greatest possible mass of the fied with the emperor's suggestion, coins'value. But it would be impu-
coin we'd have S(2) < S(3)-but basi- l:ecause he understood at once how dent of me to turn your offer down
cally they would remain the same. large the coins will grow even if you completely. Might I ask only one
Thus, the (supposedly) cunning start with one brass. (And this is just thing of you: let the value of the
emperor, having decided to cheat what Perelman's story says: //He coins grow, but not so rapidly. I'd be
the general using the avalanche ef- imagined an enormous pile of coins, completely satisfied if each coin
fect, chose the worst multiplication each one bigger than the next.") So would be more massive than the
factor (or at least one of the worst). we can definitely include Terentius previous by twelve hundredths of a
And this gives us grounds to con- in the first category. But then, why percent." (Note: most probably they
sider him a strange person-to put it didn't he understand that since the didn't know percentages at that
mildly. value of a coin is proportional to its time. I imagine, though, that Teren-
And what about Terentius? This mass, he'd also have to cope with an tius could have expressed his wish
is a little more complicated. It may avalanche of masses? Did he simply in some other way.)
help to take a leap . . . overlook this fact? No, that's hardly Nothing ventured, nothing gained.
possible: the emperor intentionally Maybe the emperorwouldhave swal-
lnlo fie recelttpa$l emphasized that Terentius was al- lowed the bait without noticing the
Way back when I was in elemen- lowed to take coins as long as he was hook-which would eventually lead
tary school (and, I should add, after able to lift them himself, without to the bankruptcy of the empire.
I read Perelman's book), I used to ask anybody's help. Actually, in this case a certain
my friends-the ones who didn't It looks as if Terentius simulta- difficulty arises: the values of the
particularly like math-to estimate neously did and didn't understand coins won't be expressed as integers,
how many grains of wheat you that he was going to be dealing with which probably wasn't allowed at
would need to put one grain on the an avalanche, and where it might that time. No matter-Terentius
first square of a chessboard, two lead him. I can't call this behavior could propose a magnanimous cor-
grains on the second square/ four on anything but strange. rection: rounding downto the near-
the third, and so on, doubling the est integer! This wouldn't cost him
number of grains each time. "HaLf a Unden fie lahe botlom too much, because the damage will
sack," my friend would naively re- By now, I imagine you've figured definitely be less than ten thousand
ply. Then I would happily set about out that this story has a kind of brasses, which is nothing compared
convincing my victim that this an- "false bottom," like a jewelry box. to his income.
swer wasn't just wrong, it was very But there's something interesting Of course, it's easy for us to solve
wrong-that in actual fact the num- even under the secondbottom. Let's the financial problems of the brave
ber of grains keeps growing from al1ow for non-integer values of k. general. But how would Terentius
square to square/ like an avalanche, Then what value of k will make himself respond to my advice? It's
and becomes unimaginably large. I Terentius's reward the greatest? It's not unlikely that he would find the
would present the result of calcula- clear enough that for this k the mass proposal sfiange, to say the least.
tions using the geometric series . . . of the coin he takes on the 10,000th After all, he would have to wait 20
and I'd be interrupted by a skeptical day must be exactly 700 kg-that is, long years for the bulk of his reward.
snort: "What the heck are you talk- it must have a denomination of In the first five years Terentius
ing about, 'trillions of tons'? I say 140,000 brasses, which means that would receive less than 6,600
half a sack, and it's haif a sack!" A k = 140,000tleeee = 1.0012. brasses, and during the first year and
total fiasco! And it's no surprise, Then Terentius's total income ahalf he'd have to come every day
because the human mind refuses to over more than25 years of his daily for a one-brass coin! So who of the
comprehend such enormous, "un- visits to the treasury will come to three is the strangest: the emperor/
worldly" numbers. S = (Ptoooo - llllk - ll = t2O million the general , or I?. It's up to you to
Here's what I conclude from this brassesl This is many times more decide. At any rate,Ican'thelp won-
little leap into the past. There are than the sum he requested of the dering what Yakov Perelman would
basically two kinds of people: those emperor. In truth, the real avalanche have thought of this interpretation
who believe in calculations and doesn't come crashing down-it just of his story. I'd like to think he
strict logic, no matter how incred- creeps along. So here is how I would would have been amused. O

l8 ilAY/JUilr lso4
HOW DO YOU
FIGURE?

Challgltug$ in phy$ics and lnath

D, Q. Prove that their second point ber of identical turns. When the
of intersection (the point other than spring is placed vertically inside a
tUIath K) lies on the diagonal BD. (V. Dub- tall cylinderwith a smooth wall, the
rovsky) spring is half as long as it originally
Ml11 was. Water is then poured into the
Factorials and powerc. (a) Prove the Ml 13 cylinder up to the level I/2. How
identity Playing vnth quadratlcs. The coeffi- long is the spring after the water is
cients in a quadratic equation arere- added? The density of the spring is
l.21T- 2.3! n(n+1)! placed with asterisks: atr2 * x; + x = 0. p and the density of water is po. (S.
222 2n The first player names three num- Krotov)
bers. The second one writes them
_\
{n + 2)l
t q instead of asterisks at will. Can the P1 12
-L
2n first player ensure that the resulting A charge isn't alone. A point particle
equation has distinct rational roots of mass m and charge Q is placed at
l-l --l . 'l -\
!!1. no matter how the seeond one ar- a distance I from an infinite con-
(b) Find the sum ranges the coefficients? (A. Berzins) ducting plane and then released.
How long does it take the particle to
r.3t+2.4!+...+ n(n+z)l M114 reach the plane? Neglect the effects
JJ- 3n Rollingto almost everywherc. (a)A of gravity. (Hint: use the method of
regular octagon is rolled over the images and compare it to previous
(V. zhokha) - plane by repeatedly turning it over problems that you have done with
(reflecting about) any of its sides. the same force law.) (A. Bytsko)
M1 12 Prove that the sequence of rolls can
Meeting on the diagonal. A line always be chosen in such away that P1 13
drawn through a point K in a square the octagon's center ends up inside Subkme self-rescue. According to a
ABCD intersects two opposite sides or on a given (arbitrarily small) science fiction story/ an astronaut of
AB andCD atpointsP and Q (fig. 1). circle.(b) Solve a similarproblem for mass M = 100 kg was at a distance I
Two circles are drawn: through a regular pentagon. (c) For what :100 m from her spaceship with a
points K, B, P and through points K, regular q-gons is a similar statement glass of frozen water in her hand.
true? (G. Galperin) Using the sublimation of the ice, the
astronaut returned to her ship. Is
M115 such a mode of rescue possible? De-
Composite sum of squares. Prove termine the time needed to return to
that 2352 + 9722 is a composite num- the ship. Assume that the sublima-
ber. (D. Fomin) tion of the ice occurred at a constant
temperatureT:272 K. The pressure
of saturated vapor at this tempera-
Physirs ture is P = 550 Pa. The gas constant
R : 8.3 |/mol . K). The size of the
glass and the mass of the ice can be
P111 any values you wish. (A. Stasenko)
Spring in water. A long homoge-
neous spring of length I in the re- CONTINUED ON PAGE 45
Figure 1 laxed state consists of a large num-

0UAilIU1'l/ll0Il [0 Y0lJ rlEllRt? I g


The houndinU lnailt

The forces controlling the sea swells

by lvan Vorobyov

HE HURRICANE IS HUN- the level and inclination of the sur- which means that the same mass of
dreds of miles away, the air is face, as well as the velocity V of the water passes through any cross sec-
calm hereabouts, but the walls water flow along the stationary pro- tion of a given layer per unit time.
of water roll one after another file, repeat themselves over a dis- The profiles of the different layers
as far as the eye can see. It's the sea tance of the wavelength 1,. Next to are not identical. Their amplitudes
swell-the steady surging of the the surface layer is another one just decrease gradually with depth. This
Earth's great oceans. The chain of below it, and next to that is another becomes clearer if we examine the
parallel curves stretches for tens of layer below, and so on. (Physicists flow between the boundaries of one
kilometers, and the waves go on like callthislaminar flow.l As the water layer. Because the flow is steady, the
that for hours on end. moves smoothly along, there are no layer is thicker where the velocity is
Near the Cape of Good Hope the gaps or ruptures/ and the curves of 1ess, and vice versa. Flowing down-
waves canreachg*l I mwithawave- the deepest layers remain stationary hill, the water particles gain velocity;
length of 100-300 m. Oniy the with respect to the profile of the sur- climbing upward, they slow down.
ocean's great depth (2 km) reassures facelayer, repeating with the same Therefore, the distance between
us that a particularly high wave won't distance ). (fig. 1). boundaries is larger at the crests and
expose the very floor of the ocean. smaller at the troughs (fi1.2). Because
L
The speed of these colossal waves is
quite impressive: 40-70 km/h.
What are the forces that produce
this regular movement of so much
water? What does the velocity of the
waves depend on? What is their
characteristic shape (profile)? What's v2
going on beneath the roiling surface?
I'11 try to answer these cluestions. Figure 2
But first, it will be worthwhile to
take a close look at the wave itself. of this, the lower boundary of each
layer is less inclined than the upper
lrymsdflow Figure 1 boundary. The difference in their
With waves, it's more convenient heights becomes smaller, the changes
to study them when they aren't o
Sq, our "stopped was.e" reference in the velocity during ascent and de- E
moving. Imagine we're flying in a frame-turns the movement of the scent are less pronounced, and the :.
helicopter with the velocity of the water into a steady-state flow along layers become more homogeneous in
P
wave motion c. Relative to us, the the curved layers. The stationary thickness as a result of the damping
curves of the water's surface don't boundaries are formed by waterpar- o{ the curves. aE
change, and along their unchanging ticles moving along the same traiec- At the lower limit we come upon
profile the water flows steadily. Both tory. Water doesn't leave a Layer, horizontal layers of still water. But

20 illY/JlJirt 1004
' '
-sri'-*d''

F"
-,"ff
ogg,#ffi*++*-
{
i { ff--
fts'

{ -t"
'
*'***tt

\ \
,Ftr'
,,e*'ff
+"-"

, *-.+r*+,1$+r ,. -'i;+r. ,..


::iC
',e: "

"t:

'%a-q.-**-**""-;#

f *#d"
*r**F=€
***.*..*F,r,4,-
actually the water is stationary only
relative to the ocean floor-in our
moving (stopped-wave) referenge
frame, it travels with a velocity c
directed opposite to the wave. The
ocean floor moves with the same
velocity and in the same direction.
If the water is deep enough to
damp the wave appreciably, the p+dp
ocean floor will be in still water and
).
will not affect the movement in the
upper layers. A quantitative treat-
ment of this condition will be found Figure 4
at the end of this article. But for now
it's clear why the uneven relief of sure at a given depth in still water is length eclual to the wavelength 1,
the ocean floor doesn't disturb the the same throughout. In deep watet $ig. al.
waves at the surface. it differs from the atmospheric pres- To calculate the force of the pres-
To find the velocity v of a water sure, but there are no pressure varia- sure acting on the upper curved
particle in the "moving waye" refer- tions along the wave profile in either boundary, where the pressure af afly
ence frame (in which the ocean floor the surface wave or in the deep point is p, we begin with a small
and the shore are stationaryl, we waves. In the intermediate layers inclined fragment. The force is equal
need to sum the velocity of. a par- the variations in pressure cannot be to the pressure times the area of the
ticle moving along the wave profile caused by air pressure/ since it is fragment and is directed perpendicu-
V and the velocity of this profile c uniform everywhere on the surface. lar to it. The vertical component of
(fig.3): But there can't be any heterogene- the force is equal to the pressure
ities coming from below either. This times the area oI the horizontal pro-
v=V+c. is a compelling argument in favor of jection of the fragment (fig. a). The
the pressure being identical at the common factor (pressure) is taken
boundary ofanylayer. out of the brackets when we sum
<_ c
Although the pressure is the same the vertical components, and the
at atly point in a profile, it changes sum of the areas of the projections
at the transition from one boundary gives the area )vL of the horizontal
to another. The pressure difference cross section of the wave fragment
and the force of gravity accelerate (where I is its width). Thus, the to-
the water particles. We can deter- tal force of pressure acting on the
Figure 3 mine this pressure difference by the upper boundary is pluL and is di-
following reasoning. A layer as a rected downward.
This simple equation will play an whole doesn't move up or down. For a lowerboundarywith a,pres-
important role later on. The forces are counterbalanced for surep + dp, the corresponding force
every fragment of the layer with a is directed upward and is equal to
fil'auityaltd pl'e$$l'g
The term "surge" is used when
waves are anywhere from one meter
<_c
to hundreds of meters long. For
waves that long we can neglect sur-
face tension. The fact that they
travel hundreds of kilometers with
no appreciable damping is evidence
of the small role friction plays here.
So the osean's surge is basically de-
termined by the interplay of just two
forces: gravity and pressure.
The pressure along the water's
surface is identical everywhere and
equal to the atmospheric pressure.
At a very great depth the layers are
almost horizontal and the water in
them is almost stationary. The pres- Figure 5

22 l'tAY/Jltlrr rss4
(p + dpl)"L. The difference between mass entering the fragment is Dividing the force by the mass dm
these forces is counterbalanced by equal to gives us the acceleration of the frag-
the force of gravity mg, where m is ment:
the mass of the fragment. There- . (m)
dm=l'-:: lc dt.
fore, l. r./ a=Ev.
c
dp=mg From this it follows that the force of
)LL gravity acting on the fragment is The acceleration is directed at a
/\ right angle to the velocity v.
Let's take a closer look at this e=l %1" at. This is a turning point. We've
equation for the increase in pressure.
dm
" (rl determined the acceleration that
A curve doesn't change alayer's results from the forces of gravity and
mass or the mass of a fragment. It's This force is directed downward at pressure/ and now we're ready to
the same as it was between the hori- a right angle to the wave velocity c. unravel the details one by one.
zontal boundaries of this fragment The pressure doesn't change
in still water. So in still water there along the flow in alayer, and the A Ricilm olmolion
is the same pressure difference, and sum of the forces directed at one an- An acceleration that is perpen-
the same pressure. (In every case it other at the butt-ends is zero. The dicular to the velocity does not
begins at the surface with the atmo- pressure difference at the inclined change its value. In this case the
spheric pressure.) The pressure at boundaries dp results in the {orce acceleration itself a = lglclvhas a
the curved boundary is equal to the constant absolute value. Constant
initial hydrostatic pressure acting on dp LV at=(ra)v at. acceleration at a right angle to the
its particles in still water (fig. 5). velocity clearly indicates uniform
circular rotation (fig. Bl. For a circle
Acuelenation ola wafler parlicle This force is directed perpendicular
Consider a small fragment of a to the boundary, at a right angle to
thin layer (fig. 6). Its butt-ends are the velocity of the flow V.
Both forces can be obtained from
the vectors V and c in the same way:
by rotating them 90o and multiplying 'a=@v=(s/clv
by the same factor (mglLl dt. Thus,
the sum of the forces can be obtained
from the vector sum V + c by the
same two operations: rotation by 90'
and multiplication by the aforemen- Figure 8
tioned factor (IiS. 7lr. Since V + c : v,
of radius z, the centripetal accelera-
tion is a = rPl4 and since the angu-
lar veiocity is ol = vf r, then a: @v.
lmsn"lcdt Comparing the last equation and the
(msn")vdt
formula for acceleration found pre-
Figure 6 viously, we obtain the angular veloc-
ity:
perpendicular to the velocity V of {ru- = lmsn'lvdt
the flow. During the time dt of v ot={.
passage through this fragment, the dm = lm/)"lcdt L
length of the inclined boundaries
isV dt, and their area is LV dt.In Figure 7 Now we essentially have the
order to apply Newton's second whole picture of the motion: the wa-
law and find the acceleration, we the total force can be expressed by the ter particles move with the same an-
need to know the mass of the frag- velocity of a water particle relative to gular velocity in a circle of constant
rnent dm and the sum of the forces the ocean floor (that is, in the moving- radius. For particles forming the
acting on it. wave reference frame). The force is wave's profile, the radii are the same,
In the period T = ?rlc the entire perpendicular to this velocity and is but their centers lie on ahorizontal
mass m of the wave fragment is re- equal to Iine. The particles rotate simulta-
placed, so the mass that passes neously; the angular displacement be-
through the fragment per second is
mlT = (ml\,lc.In the period dt the
Fsum =(T)"" tween particles doesn't change, but
the wave profile as a whole moves

0UAr{rllit/f rlTlJrr 2l
to c, the stronger the horizontal
"compression" of the crests.
Ourphysics intuition tel1s us
that we should expect some-
thing unusual when the natural
Iimit o{ the wave velocity is
passed. Formally, atv = c an in-
dQ = adt
finitely sharp vertical spike
appears/ and atv > c the profile
Figure 9 crosses over itself and a loop
emerges (fig. 13). For a wave in
with a velocity c (fig.9l. This occurs chosen at the center of the circle, the ocean this scenario would look
at every depth in the water-only the and zero time corresponds to the pretty strange. According to our
radius of the circular motion changes moment when the particle is at the
from layer to layer. crest.
AJter a time T:2nlaapa*iclewill For a weak wave (when y = oz is
return to its initial position and find small compared to c), the horizon-
itself in the same fragment, but in the tal velocity V*= c - 611 cos ot can
next wave (fig. 10)-the one that trav- be considered constant and equal
eled the distance )" = cT = 2ncf a dw- to c. Thenx = ct.Inthe formula for
ing this time. Since a = glc, then l, = y we replace , with xf c and obtain

T = 2nla
+ an approximate equation for the
profile:
V:C v>c
V>C

(cx) Figure 13
r"o'[ zrx\
(
y = rcosl-J=
J. theory, when y is smaller than but
Figure 10 ^ close to c, thelayers near the sharp
So it turns out that a weak wave is crests cuffie steeply, and the veloc-
2nczf g. This gives us the following sinusoidal. The "weakness" condi- ity of flow changes sharply {rom
equation for the velocity of the wave: tion mentioned in the opening sec- layer to layer. Doubts arise not
tion is ecluivalent to the inequality only about the stability of such
-z - 8)" motion, but even whether such a
2n T motion is close to being stable. So
.2n the case of large waves seems to
It's easier to analyze the wave demand a more compiicated ap-
profile in the stopped-wave refer- For moderate waves we see a proach.
ence frame. The profiie then is sharpening at the crests and aflat-
drawn by the particle itself, whose tening in the troughs (fig. 12). The [amping olumues flt deilr
movement is a combination of rota- The crests of laminar waves are
tion and translation with a velocity c located precisely one beneath the
c. This allows us to construct the other, and likewise the troughs. At
profile and find the dependence of the points of the profile that are
the coordinates on time: symmetrical relative to the crests/
- the absolute value of the flow veloc-
x=ct-rsinrot, ity is equal, and also at these points
y: / cos ot. the thickness of a layer is the same
The coordinate origin {fig. 11) is because the flow is constant, which
c+v precludes any skewing.
CV The boundaries of a layer in a
x:ct*rsinrullt stopped wave are formed by the tra-
y =r cos @t Figure 12 jectories of its particles. For each of
them the motion consists of a com-
vertical deflections from the crests bination of rotation and translation.
and troughs are equal for small time The difference is that the centers of
periods, but the horizontal deflec- the circles are located at different
tions differ: the velocity at a crest is depths and their radii aren't equal.
c - v, while the velocity in a through To obtain the dependence of the
Figure 11 is c + y. The closer the value of v is radius on depth we use the fact that

24 IIIIY/JUIIE r ss4
uest
'dh
+ r-- r Invest
a stamp
c+ v trough
Figure 14

the flow is constant throughout a Starting from the circle of ra- fiffisLol
layer. Let's consider two cross sec-
tions (fig. 14). The first is in a crest,
where both radii "look" up; the flow
velocity is c - y and is directed hori-
zontally. The second is in a trough,
dius ro at the surface and descend-
ing by small steps from its center,
we can find either graphically or
numerically the radius at ar,y
depth h (fi1.15). Readers who can
ti#:
where the radii "look" down; the integrate will be able to obtain the
flow velocity is c + v. When the dis- analytical expression
tance dh between the centers is
_ofi _zrh
small, the radii r artd { of the upper I=roe " =Ioa L

and lower circles differ only slightly.


The thickness of the layer in the (They will also have obtained the
crest is dh + t - r', and in the trough
it's dh + { - r. The equality of the
expression
difficulty.)
alc = 2nl),, with no great Save
flows in these cross sections gives us
lllc - vl = @h + I - rllc + vl.
(dh + r -
The amplitude of the waves and
the velocity of the water decrease
with depth geometrically. The num-
abundle
Therefore, we can find the in- ber e is approximately equal to 2.72 For the price of a stamp, you
crease in the radius dt : / -r, which = 100 43. At a depth of ),,12n the wave can get the latest edition of the
is negative- is damped roughly by a {actor of 3.
federal government's free
But at a depth equal to the wave-
dr =-Ldh length, it's damped by a factor of e2" ,
Consumer Information
c
which is close to 535. When the Catalog li"tirg more than 200
this is correct because the ra- depth is of the order of the wave- free or low-cost government
-and
dius decreases with depth. Since v = length, agitation from the ocean publicaflons on topics such as
or, the decrease in the radius is pro- floor is far weaker than the surface federal benefits, jobs, health,
portional to the radius itself. When agitation, which allows us to neglect housing, education, cars, and
the center of the circie is lowered by the influence of the ocean floor on muchmore. Ourbookteb will
dh, theradius decreases by the same the waves. help you save money, make
proportion: And now, a few final remarks. money, and spend it a little
Our basic assumption at the out-
4! = -g-an. set was the absence of a pressure
morewisely.
TC
drop along the curved layers. In
essence this meant that a deep So stamp out ignorance, and
wave is similar to a surface wave/ write today for the latest free
r/ro only it has a different " atmo- Catalog. Send your name and
spheric" pressure. The discovery address to:
1
of this kind of "self-similarity"
has helped lead to the solution of Consumer Information Center
complex problems in fields rang- Deparhnent SB
ing from fluid mechanics to el-
1/e Pueblq Colorado 81,009
ementary-particle physics. In our
I lez probiem this approach made it
possible to describe the motion of A public service of this publication
c/a=)y/2n and the Consumer Information
the entire mass of water by apply-
Center of the U.S. General Sewices
ing Newton's second law to a
Administation.
Figure i5 single droplet. O

0UAilIUl'l/ttAIURI 25
MATHEMATICAL
SURPRISES
r
$ilt challeltUiltu disseulion lasfts
And a visit from a close relative of Q

by Martin Gardner

ARL SCI{ERE& ACOMPUTER rotating the three trisecting lines


scientist in Auckland, New about the central point (fig. a). The
Zealand, recently posed the fol- trisecting lines need not be straight.
lowing six tasks: They can be as wiggly as you like,
1. Cut a square into three congru-
ent parts.
2. Cut a square into three similar
parts, just two of which are congru-
ent.
3. Cut a square into three similar bc
parts/ no two congruent. Figure 2 Figure 4
4. Cut an equilateral triangle into
three congruent parts. finity of positions. The one shown provided that they are identical and
5. Cut an equilateral triangle into may be the one in which line seg- do not intersect (fig. ab).
three similar parts, just two of ments have the smallest possible Scherer found an elegant solution
which are congruent. integer lengths. to the fifth task (fig. 5). It's believed
6. Cut an equilateral triangle into As mathematician Robert Wain-
three similar parts/ no two congm- wright of Plainview, New |ersey, has
ent. observed, figure 2b results when the
The solution to the first task is slanting line is orthogonal.
obvious (see figure 1). It is surely We turn now to the three equilat-
unique, though I know of no proof. eral triangle tasks.
Ian Stewart and A. Womstein have The fourth task obviously has an
shown that no rectangle can be di- infinity of solutions, obtained by
vided into three 18
congruent poly-
ominoes unless
the pieces are Figure 5
rectangles.l
Figure 2 shows to be unique. Note its similarity to
three solutions to figure 2c.
Figure 1 the second dis- 2l The sixth task is easily solved (fig. 6).
probably unique, though no proof
section task. t4 It's
Task 3 is more difficult. Scherer t'-' is known.
found the pattern shown in figure 3.
-+-
I #
!l My only con-
The solution is not unique, because tribution to the
the slanting line can assume an in- six tasks was the
tlownal of CombinatoriaT Theory, t2 rediscovery of a
Series A, Vol. 61, September 1992, pp. second solution
130-35. Figure 3 to the third task Figure 6

28 ltlAY/JUltlI ISS4
a the smallest rectangle to the ratio of high-phi:
the sides of the similar largest rect-
angle is r+-!
Jl+ 1

I _xz -x+l .
J-_r*,
x xz+l ' [r+1
*-x+l Jl+1
x3 -2x2 +x-1=0,
*+l (x'-x)(x-1)=t.
Figure 7 As Knuth write, the series con-
The decimal expansion is of x verges more rapidly than the series
(fig. 7). I later learned from Scherer 1.75487756624659275... . As Wain- for phi, giving values that arc alter-
that he had found it years earlier. wright pointed out/ the number is nately over and under the true value:
What is the value of x, assuming the closely related to phi, the golden ra- 1,2, 1.71, 1.765, 1.753, r.7554, ... .
smailer side of the smallest rect- tio. The reciprocal of phi equals phi Knuth also called attention to the
angle is 1? I thought this would be a minus one. The reciprocal of x {ollowing equality for high-phi:
simple question to answer. If x isn't equals (, - ll'.Other equalities are
rational, surely it's a recognizableir- r, I -. I
=Y+-'
rational, such as 1.732... (the square 1,-1
__.,1;f _I; vX=_. I -r I
root of 3 ), or 1 .618 . . . (the golden ra- x2 x-l
tio, often called phi), or some other Karl Scherer points out that the
well-known irrational. I propose calling this number three rectangles in my figure have
To my amazement, x turned out "high-phi." DonaId areas of x, #, and1.. And if the origi
to be an irrational number ffiI;;-*.." Knuth, Stan-
ford
nal square has a side length of 1, the
rectangies have areas of I f x, I I *, ar.Ld
I had never encoun- Uni-
tered before. ,C ' versity's Iff. This shows that I : llx+Il* +
The cubic ,4ffi 'r,,,1 noted llf , and the ratio of the largest regt-
equation re- ,:N ,'i com- angle to the rest of the scluare is JP .
Iating the ,i #i{ l'i Pu.ter Scherer suggests the terms phi-
ratio of f.'#ff , I Sclen- two, phi-thtee, and so on/ for the
the W '..ri tist, first terms of the series of solutions
sides ,"..1 sug-
ofM , f
for the equation
' ,'i gested 1
1=(x_1),.
l .d' ,,' ,i'' giving it x
.j:,,,1' the symbol
i:f i"" g, in which He conjectures that 1 is the sum of
.ri',: the little cir- the infinite series of the reciprocals
,t'.,i cle of phi is of phi-two, phi-three, phi-four, and
,' ..t raised. He point- so on. In brief,
i.::j ed out in a letter
. .=. i-'1 how close a mofi-
1', +:-1.i
")t fied fraction for
r= i, 9;r'.
,6
"'
*i!i.,
high-phi resembles
0

,- J the continued frac- Can any reader prove or refute this


A Y. ,.f tion for phi. Phi is coniecture?
.,jl the limit of Is it not surprising that such a
simple geometrical construction
-1
l+- would generate such a curious num-
1+1
.Ix fl 1+1
ber? Note that 666, the number of
the beast in the Book of Revelations,
1+1 follows its first six decimal digits.
o
C
(6 l+1 Perhaps Quantum readers know of
other properties, serious or numero-
o logical.
O)

ao Add square root signs and you get I would welcome hearing from
-o the modified continued fraction for anyone who can find other solutions
:,
ffi r-a,
1.. to any of the six tasks. O
"
v*3 -F"S
{ "e
?.h

O U A II I U ill / ]ill A I 1I I lll AT ICA I. SURPRI $I S 2t


MATH
INVESTIGATIONS

]lappy hirthdaU Uncle Paul!


Erd6s is "squared,"
and we hope he will be squared yet once more

by George Berzsenyi

eur pno6s, MATHEMATT- which is greatly responsible for the motion amongst the mathematical
cian extraordinaire, recently high level of mathematical life in centers of the world, being the focus
celebrated his 8 lst birthday and that small country. Following the of mathematical activities wherever
referred to the occasion as being completion of his formal studies for he goes. Just as a bumblebee goes
"squared," since 81 :92.We from flower to flower carry-
hope he will celebrate many ing its load of pollen, he
more birthdays, write yet an- goes from mathematical
other 1,500+ papers, inspire center to mathematical cen-
many more mathematicians ter with his problems and
throughout the worid to de- information, thereby being
crease their "Erd5s number" an agent of mathem atical
to l, and pose many more cross fertilization." The
wonderful problems.l In this problems beiow are but a
column I'11 share some of his few samples of his many in-
problems with my readers in teresting queries.
the hope that they will assist Problem l:Let P be an ar-
Uncle Paul, as he is affection- bitrary point interior to a tri-
ately called by his many angle, and denoteby a, a, a,
friends and admirers, in his the distances from P to the
constant quest for the math- triangle's vertices, and by x,
ematical unknown. y, zbe the distances from P to
Paul Erd5s is a native of the three sides of the triangie.
Hungary. His unique math- Determine the minimum of
ematical talents were recog- lar+ar+arlllx+y+zl.
nizedvery early and were en- Problem 2: Let n points
couraged by his parents, both be given in a plane, with no
of whom were excellent three of them on a line.
mathematics teachers. His Maximally how many pairs
development as a mathema- of points can be unit dis-
tician was much enhanced by Paul tud6s in 1983. tance apayt?
the Kozdpi skolai Matem a- Problem 3: Prove that
tikai Lapok, Hungary's 1 O0-year-old a doctorate in mathematics, Erd6s
high school mathematics journal, became the only truly universal >;h is irrational.
Problem 4: In a convex n-gon, let
university professor of mathemat-
IIf you coauthor a paper with Erd6s, s17 s2, ... denote the multiplicity of
ics. In the words of one of his admir-
your Erd6s number is 1; if you the occurrence of the distances be-
coauthor one with someone who
ers, |. W. S. Cassels of Trinity Co1-
tween the vertices of the n-gon.
coauthored one with Erd6s, your Erd6s lege, Cambridge University: "He /.\
number is 2; and so on. has executed an almost Brownian (Note that )s, = ) Prove that
[;,
28 IuAY/JUIJI 1ss4
there exists a constant c such that ample, while f.orn = 6,7, and 8 Ilona questions: How many fistinct points
Palasti did so. (To whet your appe- of intersection can there be? And
\sl < cnT.
tite, Pomerance's example is illus- what is the minimum of the number
Problem 5: Prove thataconvexn- trated below, with equal distances of intersection points? Moreover, for
gon always has a vertex that is not bearing the same markings.) Erd6s 2n points, can the number of intersec-
equidistant from any four of the offers $10 for an example for n:9, tion points be smaller than that for
other vertices. $25 for a proof that for large n such regular 2n-gons?
The problems above were posed in an example does not exist, and 950 As the above problems illustrate,
the centennial issue of Kozdpiskolai for a proof or disproof that for large the questions posed by Paul Erd6s
Matematikai Lapok; they aie all n the points determine at leastn dis- constantly probe the frontiers of the
open. Problem 6 beiow was his first tinct distances. known mathematical universe. His
serious problem, posed in 1931; it, discoveries cover most branches of
too, is still unresolved. Problems 7 mathematics from number theory
through 10 are reproduced here from to combinatorics, from foundations
recent letters of Pali B6csi (Hungarian to analysis, from geomgtry to prob-
for "IJncle Paul") to the author. The ability, and many new areas which
prizes offered by him for the solution
were initiated by his own investiga-
of his problems are also typical, ex- tions. It is not unusual that he is si-
cept for the fact that most of them are multaneously working on several
in the thousands. It has been esti- papers with different mathemati-
mated that he couldn't possibly cover cians.
his promises if all of his problems
"Pali B6csi" loves to work with
were solved at the same time. How- young people, many of whom have
ever, since most of them are very been inspired by him to great ac-
deep and difficult, he seems to be safe
complishments. He was also one of
from ever going bankrupt. Problem 9:Letflnlbe the largest the founders of the famous Budapest
Problem 6: Let ar < az < aB < ... < integer for which there are integers Semesters in Mathematics program,
ao be distinct positive integers such a, b torwhichnllfulb!) is an integer, which is briefly described in the
that the 2k sums, 2{.rr,where e, = and a + b : n +l(n). Prove that there Bulletin Board in this issue (page
0 or 1, are all distinct. Estimate or exist cl andcrsuch thatl(n) < c, log szl.
determine the value of min ao. n for alln, and try to prove that f(nl Paul ErdSs is a member of the
Problem 7: Let a, < az < ... be the > c, 1og n for infinitely many a. Hungarian Academy of Sciences and
set of integers of the form 2d3 9. Prove Moreover, refer to n as a champion was also elected to membership in
that everyn can be written as the sum if f(nl > f(ml for all m < n. For ex- the Academies of the Netherlands,
of ar's, no one of which divides any ample, 10 is a champion, since Australia, India, and England. He
other. Is it in fact true that if n is suf- l0! l6!71 = L, 6 + 7 = l0 + 3, andf(ml has also been the recipient of nu-
ficiently large, then there exist such . 3 = fl10) for all.m < 10. Try to de- merous honorary degrees. Most re-
ar's with termine all champions.Letgnlal =b cently, the World Federation of Na-
be the largest b for which nllalb! is tional Mathematics Competitions
A.+A.+...+A.=n
lt 12 .rft an integer, andletfnful = a + g,lal -n. honored him by creating the Erd6s
Determine or estimate the value of Prize, to be awarded to mathemati-
and cians whose efforts in the popular-
+>!:f"@). ization of mathematical. competi-
Problem 10: Let X, x2, ..., xnbe tions have resulted in an increased
the vertices of a convexn-gon in the awareness of the important role of
Problem 8: Let x' Xz, ..., xnbe n plane. Construct all of its diagonals; mathematics. O
points in the plane in general posi-
tion-that is, no three on a line and there willO. (;) - n ofthem. Con- The purpose o{ this column is to direct
the attention oI Quantum's readers to
no four on a circle. ErdSs believes sider the interior points of intersec- interesting problems in the literature
that for largen the points determine tion thereof. If we assume that no that deserve to be generalized and
at least n distinct distances. For three of them go through a point, could lead to independent research
smalla this is certainly false. In fact, then it is trivially true that there are and/or science proiects in mathemat-
for n < 9 it can happen that one dis- ics. Students who succeed in unravel-

tance occurs n - I times, one n - 2 [l)


\+/ ",
them, since every choice of 4 ing the phenomena presented are en-
couraged to communicate their results
times, and so on. For n = 4 an isos- points yieldsa point of intersection. to the author either directly or through
celes triangle and its center consti- Hence let's not assume anything Quantum, which will distribute
tute an example. For n = 5 Carl about the number of diagonals meet- among them valuable book prizes andf
or {ree subscriptions.
Pomerance constructed such an ex- ing at a point. Then there are two

0 lJ I ll T l,l I'l / il A I ll I ll U t STI t AT I 0 il S 2g


IN YOUR HEAD

Geomelric $ttlnlnaliun
I nfi n ite al geb rai c ti I i ng s

by M.Apresyan

IGH SCHOOL STUDENTS area lf 4, and so on. This process cre-


know that for lql < l, the sum ates a sequence of rectangles whose (n-l)*'-l
nn'
*+-.
of an infinite geometric se- areas are equal to l,lf2,lf 4, LfB, ...,
quencel+q+q2*q3+... (tl\
Ll2n, .... The union of these rect- =in-1)l 1- '
equals llll - ql, and study an arta- angles coincides with the initial rect- \nn')-+- |

lyticai proof of this formula.For q = angle'(without one comer point). So


lf n, wherc n e N, this sum can be the sum of the areas of all these rect- Thus,
computed geometrically. You might angles is equal to the xea of the ini-
have seen similar geometric proofs tial rectangle-that is, to two. Thus, I
lln
r-=-r..
of simple algebraic identities-for l+l12+Il4+ll8+...:2. n n' n-l r-rl n
example, la + blz = a2 + Zab + b2- Now let's find the sum 1 + lf n +
that make use of a device called al- lf n2 + lf n3 ... . Start with a rectangle The same idea works for the sum
gebraic tiling. of arean and cut it inton rectangles of an infinite geometric sequence
We'llbegin with the case n:2. of area 1 (fig. 2). Next, cut one of with an arbitrary rational ratio q,0 <
Let's find the sum these rectangles into n rectangles of q < 1. hdeed, let's take the last con-
equal area (Ilnl, do the same with struction in a rectangle of area n, but
111
I+-+-+-+ .... one of these smaller rectangles, and each time let's subdivide m (m < nl
248 so on. This process yields rectangles rectangles of the n obtained in the
of areas I, I f n, I 1n2, ..., I f nk, ... (n - | previous step (fig. 3), leaving intact
Take an arbitrary rectangle o{ area rectangles of each of these areas). The the remaining n - mrectangles. Do
2. Cut it into two rectangles of unit union of al1 these rectangles is again you think you can take it from here?
area lfig. 1). Cut one of these rect- the initial rectangle with one corner Give it atry, arrd derive the formula
angles again into two equal halves of point deleted. So the area of the forl+mfn+lmlnl2+....
area lf2. Do the same with one of union is n; on the other hand, it's This method can be appiied to
them to get two small rectangles of equal to some other infinite sums as well.

I
16
ilt
I I
4 ,i
n
1
n
1 I I I I I
n
1 I
) n
t
n

Figure 1 Figure 2

30 It/lAY/JlJilt 1 SS4
n-m
$ ,li

1 1 1 . ir,i;,r,1;, jf t,

.*,,..i,t
1 .

.'il n-m ,r",r:",;i-.,' .


,i,'l:

i
ri,ltliiiil,,i,i:il .
,

1 l i, rtl lrl',,'llli:l,l,t'll:"

n
Figure 3 Figure 4

For instance, let's prove that now we cut one of the last two is 1 + 1/2! + 213! + 314! +..., which
rectangles intothree parts equal in is equal to 2-the area of the whole
-123 n area (the areaof each of these parts rectangle. O
l+-+-+-+...
2! 3! 4!
r
'(n+1)!
-', "' is l12 + 3 = llSl = ll5l. One of the
rectangles of. area 1/6 is now cut E
.-:,. . ..
1ir'i.:ii us a line at Quantum,
- into four equal parts, one of these 1840 Wilson Blvd.,
Again we'li take a rectangle of arca parts is cut into five parts, and so Arlington vA 2220 1 -3000
2. The first two steps are the same on. This yields one rectangle of area o1
as in the first example: we cut the 1, one of arealf 2, two rectangles of
rectangle into two unit rectangles, area lf}!, three of area lf4l, ..., n E ": ', us an electron or two at
203O.3 | 62@ compuserve. com.
and then one of them into two rectangles of. areallfu + 1)1, and so 7

rectangles of area l12lfi1.4). But on. So the sum arising in this case

The BURLEIGH INSTRUCTIONAI STII['"


Thansforms Atomic Theory Into Atomic Reality
High Perforrnance, Student Proof E:rperiential
tearning System Allows First Time Users to Vieur
Atoms and Measure Them... Quickly and Easily
Comprehensive Scanning Tunneling Microscope
System designed specifically lor classroom use.
2.45 A
- Easy to operate even for first time users - supplied
with step-by-step Quick Start instructions.
- Laboratory workbook provides predetined experi ments
along with theory and history of microscopy -
prequalilied sample set enclosed.
Windows-based True lmagerM software for your
386/486 computer provides sub-angstrom image reso-
lution and precise three dimensional measurements.
Proven system with over 150 in use by colleges,
universities, and high schools worldwide.
For more information and a demonstration video
call 7 1 6 / 924 -9355, FAX : 7 1 6 I 924-907 2 o r write :

ryl'Ursffi
f, :E corpt"te System <$15,ooo-
Beliable Precision
l:'i:.. .,..ii;!w I ncl
udes ContrOl eleotron iCS, STM
scanning head, sample set, sampling kit, Burleigh lnstruments lnc.
instruction manual, and student workbook. Burleigh Park, Fishers, NY 14453
.U.S. Prices 924-9355 . F AX'. 7 1 6 I 924-907 2
7 1 61

Circle No. 2 on Reader Service Card 31


KALEIDOSCOPE

]lol'sellies and IIyinU hol'se$ i


i
i
Questions of scale in the animal kingdom

by A, Zherdev

'i:.+

IZE TS ONE OF THE MOST then as now were delighted by *@=''


.'4
important characteristics of a Swift's "wondrous realism" as ex- \#.-1-'
living thing, but the difference pressed in the precision of his arith-
in size is so obvious that we of- metical calculations: Gulliver was
ten pay little attention to it. Every- 12 times larger than Lilliputians and
body knows that an elephant is big- was smaller than the Brobding-
ger than a mouse/ but rarely do we nagians by exactly the same factor,
think how much bigger: by a f.actor and everything in both countries
of 100,000. The smallest adult was scaled by the appropriate
shrew is a tenth the size of a mouse, amount.
which makes it l/1,000,000 the Real life is not so simple, how-
mass of an elephant. The difference ever, and the first person who real-
is even more striking when we com- izedit was none other than Galileo
pare animals of different phyla-say, Galilei. He wrote: "It is not pos-
aprotozoar and a whale. sible to decrease in equal mea- i
Does a size difference lead to a sure the surface and the weight ,i
qualitative change? For a long time of a body and preserve similar-
both scientists and lay persons be- ity of form. It is absolutely i
lieved that it doesn't and thought clear that the decrease in i
that all the characteristics of living weight is proportional to the :
creatures change in proportion to decrease in volume , and
their size. The classic example of therefore every time the vol-
such an assumption is the world cre- ume decreases more than the )

ated by |onathan Swift in his elassic surface (while preserving


satire Gulliver' s Tr avels. Readers

71
--k**
w>
ffilan
o
l< s=24
-C
a
'tr
v= 8
p
C
o
c)
J
_o
s/v :3
wv* l))-
s/v=5
Figure 1

t2 llllAY/Jl|lrr rg04
weight will also de-
crease more than the sur-
face. But geometry teaches
that the ratio of volumes
of similar bodies is larger
than the ratio of their sur-
faces. ., . Therefore, it is
impossible to construct
ships, palaces, and churches
of enormous size such that
their oars, masts/ beams,
iron clamps-in a word, all Figure 2
their parts-hold together.
On the other hand, Nature whose structure and arrangement
herself cannot produce gi- are basically the same. Still, a dis-
gantic trees because their tinct differentiation exists: the
I branches would ultimately smaller the animal, the greater the
break under their own weight. mass (relative to its own) it can lift.
Likewise, it is impossible to How can this be explained?
imagine the skeleton of an im- 2. Why do animals with approxi-
possibly huge human being, mately the same shape (a grasshop-
horse, or other living creature per and a locust or a kangaroo rat
that cansupport the body as it is and a kangaroo) iump to the same
meant to. Animals can attain ex- height, regardless of their size?
traordinary sizes only i{ their 3. In the treatise cited above,
bones change, increasing in thick- Galileo introduces a drawing that
ness by a corresponding amount." shows that alarge bone is dispropor-
(See figure 1.) tionately thicker than a small bone
As an animal increases in size, the fis.2lr. He made a small arithmetical
parameters of its various physiologi- error, though. What was it?
cal processes increase in di{ferent 4. A scientific expedition discov-
ways: some linearly proportional, ered a new creature: a one-{ooted
others proportional to the squares or mammal (fig.3). Its dimensions are
cubes of these values. And so animals given in retems-the unit of length
of different sizes must have different
shapes. An entire branch of biology
is devoted to analyzing the rela-
tionship between size and shape,
and researchers in this area
have obtained a number of
interesting results. Work
through the questions
'. presented below and
you'll become ac-
quainted with some
. of them.

Questions
t. On the mi-
croscopic level
the muscles of the
most various kinds
of animals do not
differ all that much
in structure. Muscu-
lar contraction is
caused by intermolec-
ular chemical complexes Figure 3

0llAtlT ll It/[il.il [ 0 $c0 Pt 33


used by the natives. Estimate the Take a good look around. . . . . . a few years ago the newspa-
mass and height of the "monoped" Compare the height and thick- pers trumpeted the achievement of
using human data (average measure- ness of nearby trees and stalks of tall a cettain seven-year-old who set a
ments or your own dimensions). De- grass. What {ormula describes the re- record for doing push-ups (about
termine the length of 1 retem in lationship between these two pa- 5,000). The boy grows up and prob-
meters. rameters? What factors are at play ably takes up g),'rnnastics, but he has
5. A baby was born weighing 8 here? no chance of beating his own rec,ord.
lbs. It tripled its weight in one We hope by now you understand
year, weighing in at 25 lbs. Con- It's interesting that . . . why.
tinuing to grow at the same rate, . . . there are so many large ani- . . . the biggest land animal was
by the end of its second year it mals around. If it's so disadvanta- the brachiosaurus/ which was up to
would weigh 25 x 3: 75 lbs; after geous to be large, why didn't evolu, 20 m long and weighed 80 tons.
three years , 75 x 3 = 225 lbs; after tion produce a prevalence of small Speculation about how such a huge
four, 225 x 3 = 675lbs; and after animals? The reason is that large animal could exist led paleontolo-
five years, the baby would weigh animals are stronger than smaller gists to the hypothesis that this kind
67 5 x 3 :2,025lbs. The daily food species in absolute terms/ although of dinosaur lived in tidal areas, so
intake of this one-ton " toddler," at the smaller animals are relatively that its entire body (except the head)
I 15 to I 17 of its body mass (as is nor- stronger. This gives them the advan- was immersed in water (fig. 4). Oth-
mai for growing children), comes to tage in head-to-head competition erwise, in the opinion of these au-
300 lbs. From this estimate the fa- and also allows them to occupy new thors, the bones of the brachiosaurus
mous Po1ish teacher |anuszKorczak ecologic4l niches. But bigness is could not have borne the enormous
drew the conclusion that one unfavorable in other respects. In or- load. However, this mode of living
should not force little children to der to find enough food, a horse creates certain problems. In particu-
eat against their will! What do you must cover more terrain than a lar, it would be impossible to
think the daily food intake of such mouse does. Since the food supply is breathe, because the lung muscles
a three-year-old baby should be? usualiy limited, a mouse may have would not be able to continually
How much do you weigh, and how the upper hand over the horse be- overcome the pressure of the water.
much food do you eat each day? Do cause it needs less food. A hectare of And so biologists had to go back to
your figures iibe with Korczak's cal- meadowland can support a huge the blackboard and recalculate the
culations? population of mice, but no more strength of the bones. They came to
6. (a) Insects don't try tobtzz or than one or two horses. Appreciable the conclusion that the bones could
drone-it just happens. The sound populations of large animals can support such a huge load.
comes from the flapping of their gather only on great expanses of . . . comparing different mam-
wings. The force of the flapping land. Thus, both smallness and big- mals, biologists came up with a
(pushing against the air) must com- ness have their own ecological ad-
pensate for the insect's tendency to vantages. CONTINUED ON PAGE 37
fall due to gravity. So why do gnats
buzzwhlle bees drone? What is the
relationship between the insect's -.-:._Zr'- z- - .t:ilt' ,r,
-^-'-.
tone and its size? (b) Our friend -
Gulliver complained about the dron-
ing of Brobdingnagian flies: at
dinnertime these insects didn't give
him a moment's rest. He may have
been bothered by them, but did they
really drone?
7. The higher the body tempera- -i+\-\. I

ture of birds and mammals, the ---_ \-r- ,:':t\\


smaller the animal is. Why?
//2 -- - \\ -\i-,,.
B. Parents who are in a hurry waik . .< .-- -.., \\\ r'il,-<:---ll
quickiy, and if they happen to be INr I - )',. r{l ,-i^-. ;-lr- ---
holding their child by the hand, the
child must run to keep up. Why do
adults and chiidren achieve the
same speed in such different ways? =ZET /*"4+<<\. I
9. Which desert animals are able
to live without water for a longer
\fJ={;--1.-.-{
time-smalI ones or big ones? Figure 4

3 4 l'tlY/Jtlrrr lss4
AT THE
BLACKBOARD I

A litlle lens tillk


Then let's go to the movies

by Alexander Zilberman

HE PHENOMENON OF RE- the light beam is almost parallel (for Admittedly, the simplest calcula-
fraction of light at the bound- example, the beam from a search- tions of optical systems that can be
ary of transparent media (say, light). done within the framework of high
air and glass) can be used for a In many cases of practicalimpor- school physics will not meet the
number of purposes in various opti- tarrce, the optical system consists of practical demands of actual devices:
cal devices, including parallel plates, several lenses. For example, the ob- even the lenses for high-quality eye-
prisms, and lenses. jective of a camera can consist of glasses (not to mention contact
Most often lenses are used to form more than ten different lenses-con- lensesl) are often calculatedby com-
images of luminous (or illuminated) cave and convex/ thin and thick, puter, and not because o{ a surplus
objects. A lens makes it possible to made of different and special kinds of computers.
produce an image at the right place of glass. One can even have lenses Nonetheless, even a simple
(on the film in a cafiierat on the screen with a more complicated geometry theory can come in handy, because
in a movie theater) or at a distance than the usual spherical surfaces. approximate calculations will often
that is comfortable
for viewing (eye-
glasses, a magnify-
ing glass, contact
lenses). We can ob-
tain an image of the
object that is either
greatly increased (in
a microscope orfilm
projector) or de-
creased (in a tele-
scope or binocu-
lars-yes, decreased
by afactor of thou-
sands, but brought
nearerby afactor of
hundreds of thou-
sands, whichmakes
it possible to ex- o'
amine the object a)
in all its details). o
(o
We can also obtain o
an image of a lumi- E
OJ
nous point at infin- x
ity-in this case =
=

OllAltlIUltl/AT TIII BTACI(BOARI I


be good enough. Let's look at the tween the principal optical axis of
paths of beams falling on a very
tance Rf (n - 1) from the lens. In
the lens and radius drawn to the other words, we have calculated the
simple, plano-convex lens with a point of incidence of the beam. The focal length of our lens:
spherical convex sur{ace of radius R. beam then exits the lens at an angle
I've chosen this lens because the of ,R
paths of the beams will be simplest 'r i-. n-l
in this case. Let the lens be made of
glass and placed in air.
Let's imaginethat aparallel beam
[;.4"-o=s+(n-r)B. In the same way/ after more
lengthy calculations we can obtain
of light falls on the plane surface bf From now on we'llneed the thin- an analogous formula for the focal
our lens. We know that after refrac- lens condition: we'll consider that iength of a lens formed by two
tion in a convergent lens it must the points of entry of the beam into spherical surfaces with radii R, and
converge at a point lying in the fo- the lens and exit from it are at the Rr. It's more convenient to present
cal plane. Let's show how. First we same distanceRB from the principal this formul a in a slightly different
need to formulate the problem more optical axis. At a distance I to the form:
accurately. We'll assume that we're right of the lens the beam is shifted
using a thin lens (in due course I'11 vertically relative to the exit point
explain more rigorously what the by Lfln- 1)F * ol, and the distance to !=.,-l)fr*a).
f ' '[R, Rr)
term "thin lens" means and what the principal optical axis is
we should neglect), and we'll take The radii of the lens can be both
the angle of incidence to be small H = Ll(n - 1)F * 0l- RB.
positive (biconvex lens), both nega-
(this keeps the error small when we Two different rays from the origi- tive {biconcave), or they can have
replace the functions sin o and tan o, nai incident beam intersect to the opposite signs. (In our case of a
with the angle cx itself, which right of the lens (fig.2l. Equating the plano-convex lens, one of the radii
greatly simplifies the calculations). is infinitely large.) If the resulting
These are reasonable conditions focal length turns out to be positive
that correspond in general to the ac- (the focus of the lens is real)-that
tual situation in simple optic,al ex- is, the refracted beams indeed con-
periments. verge at a point-the lens is called
Let's draw the lens and the path converging (or positive). On the
of one of the incident beams (fig. 1- other hand, if the {ocal length is
here the lens is thick and the angles negative (the focus is imaginary)-
arelarge for the sake of legibility). that is, the parallel rays diverge af-
Figure 2 ter refraction-the lens is ca11ed di-
verging (or negative).
distances H for both rays, we find There is simple relationship
the distance I to the point of inter- known as the lens formula (it can be
31 {":1)B _
section: deduced geometrically) that links
I[(r-1)B,+o]-RB, the distance s between the source
and the lens, the distance s'between
= Llb - 1)B, + o) - RFz,
the lens and the image, and the fo-
from which we get ca1 length / of the lens:
Figure 1

r_ R
1-t_1
The chosen beam strikes the lens at n-l s s' ,f'
an angle a to the principal optical
axis, and after refuaction at the plane We see that the distance obtained Thus, alter re[raction by the con-
surface the angle decreases by a f.ac- doesn't depend on the angle B-it,s verging lens, the beam of parallel
tor of n, where n is the refractive the same for all the rays from our rays converges at a point in the fo-
index of glass. We can see this by beam. Thus, we have proved that cal plane. This makes it easy to de-
using Snell's law the refracted beams converge at one termine the path of any beam after
point, and we've found the distance refraction. We need only draw the
sin c,: n sin Q
from this point to the lens. Notice auxllrary ray parallel to it that passes
and approximating these sines by that ali the points of intersection through the center of the lens (this
the angles to get Q aln. The beam (corresponding to various angles of ray is not refracted) and find the
=
falls on the spherical boundary be- incidence) lie in the plane perpen- point of intersection of this ray with
tween the glass and air at an angle of dicular to the principal optical axis the focal plane-the original ray
uf n + p, where B is the angle be- of the lens and are located at a dis- must pass through this same point

38 ]itlAY/JUI'lt 1004
the first case, draw the focal plane -KALEIDOSCOPE"
on the other side of the lens. How-
ever, both focal planes must be at
CONTINUED FROM PAGE 34
the same distance from the lens. curious relation (Bergmann's rule): as
Now let's retum to "real" lenses. we move from the poles to the equa-
Usually it's assumed that the point tor, the size of warm-blooded animals
source emits light evenly in all di- of the same or similar species de-
rections. Wherever the eye is 1o- creases. Why? One of the important
Figure 3 cated, it receives the divergingbeam factors determining the energy bal-
of light and we perceive the iumi- ance in an organism is heat loss, so
after refraction (fig. 3). The possibil- nous point. It's quite another matter animals in polar regions "try" to de-
ity of sketching the path of an arbi- if we want to see the image of this crease it by any means possible. Be-
trary ray helps greatly when we need point in a lens. Let figure 5 show a cause mammals are warm-blooded
to form an image in a complicated "!eal" lens of a cettain size. In this and their temperatures don't change,
optical system consisting of a dozen they can't decrease heat losses by
t5
lenses. There is no need to obtain lowering their body temperatures.

--il
o
intermediary images after each lens. However, increasing the body's di-
I should point out that with this mensions lets them decrease the
method o[ drawing we can use any relative (that is, per unit mass) sur-
rays-even those at large angles to face that transfers heat. Since heat
the principal axis. If necessary we transfer is higher in a cold climate,
can increase the diameter of the lens a larger size is more important for
so that the rays land on it. The point animals in polar regions, accounting
Figure 5
is, this method of drawing corre- for Bergmann's rule.
sponds to a "paper lens"-that is, case the image of a point source is . . . hummingbirds with a mass of
not to a lens but to the lens formula. formed by a beam of rays whose 3-5 g are the smallest birds, and
For a real lens such a result corre- marginal r^ys are limited by the di- their heat losses are particularly
sponds to " eorrect" rays only-that ameter of the iens. These marginal high. To keep their body tempera-
is, to rays at small angles to the prin- rays also restrict the beam of rays ture constant at night, the hum-
cipal axis-and this is exactly what emerging from the lens. We can see mingbird must store up fat or glyco-
we need. the image only if we are within the gen during the day. However, this
Let's use the method described to solid angle formed by the rays that would be extremely inconvenient
solve a simple problem. Figure 4 passed through the point of intersec- for the bird, since it would increase
shows a converging lens, its princi- tion after passing through the lens. its body mass in the daytime and
pal axis, and the path of one ray be- It's interesting that there ire lead to higher energy expenditures
fore and afterrefractionby the lens. points from which neither the when flying. In addition, the conver-
source nor its image can be ob- sion of the original carbohydrates to
served. For example/ say we were at stored, energy-rich substances also
the point B: the source is covered by requires energy. Also, the humming-
the lens, and we're outside the solid bird's method of feeding-hovering
angle from which the image can be over a flower-consumes quite a bit
seen. In order to expand the region of energy. Lr the course of evolution,
where the real image can be observed, the hummingbird was faced with
we can use a screen. If we place a two options: get bigger, or reduce
screen where the refracted rays nocturnal heat losses. As a result,
Figure 4 emerging from the lens intersect, the their record (among birds) high body
rays reflected from the screen image temperature of 43-45"C is main-
We need to find the position of the will travel in every direction. tained only during the day. At night
focal plane. Let's draw the auxiliary This is how a movie is shown in it drops all the way down to
ray parallel to the incident ray such atheater. If we try to watch a movie 10-20'C. This explains why hum-
that it passes through the center of by looking into the objective of the mingbirds live in the tropics-other-
the lens-this ray doesn't refract, projector, or by using a mirror in- wise they would experience large
but its point of intersection with the stead of the screen, we'll see at best heat losses around the clock and
refracted ray lies in the focal plane. only a small fragment of the overall couldn't have the normal metabo-
By the way/ we can "reverse" the picture. Different parts of it wiil be lism of warm-blooded animals. O
rays-taking the incident ray to be visible from different places-not
the refracted one and the refracted exactly what the director had in ANSWERS, HINTS & SOLUTIONS
ON PAGE 59
ray as the incident ray-and, as in mindl o

0[Ailrlllll/AT TIr Br.ICr(80[R0 r $7


PHYSICS
CONTEST

Lasel' leuilaliolt
"Suspend here and everywhere, eternal float of solution!"
Whitman, Leaves of Grass
-Walt
by Arthur Eisenkraft and Larry D Kirkpatrick

OW CAN SOMEONE LEVI. where R is the number of pellets suspended is so smal1 that its weight
tate an object? Magicians do it hitting the box each second. is of the order of 10-10 newtons? If the
all the time. Can physicists do We can get a feeling for the prob- it can be levi-
object is transparent,
it as well? The easiest tech- lem by solving it with some appro- tatedbya laserbeam! How to do this
nique is to attach a string to the ob- priate values. If the pellet gun shoots was one of three theoretical problems
ject and secure the string to the ceil- 5 pellets per second, and each of that were given to students who par-
ing. The weight of the object is bal- these 2-g pellets hits the box with a ticipated in the XXfV International
anced by the tension in the string. If speed of 50 m/s and rebounds with Physics O\nnpiad, which was hosted
the suspended object is a magnet, the same speed, what is the heaviest in the United States inluly L993.
then a second magnet can keep it in box that can remain suspended? This theoretical problem was created
place. A third technique is to shoot Let's work it through: by Charles Holbrow of Colgate Uni-
pellets at the object so that the {orce versity. We have adapteditfor Quan-
of the pellets balances the weight of
poeret = mv =
' {210-3 kg)(50 m/s)
tum readers.r
= 0.1 kg m/s,
the object. By means of refraction a strong
Let's assume that the object we Apperet = 0.2 kg m/s, laser beam can exert appreciable
wish to suspend is a rectangular box forces on smal1 transparent objects.
oriented so that its bottom is hori-
F: (5 pellets/s)(0.2 kg m/s) : 1 N.
To see that this is so, consider a
zontal.If we shoot pellets vertically Therefore, a 0.l-kg box can be sus- small glass triangular prism with an
upward at the box, the pellets just pended with these high-speed pel- apex angle A = n - 2a, abase o{
provide an average force on the box lets. length 2h, andawidthw. Theprism
that is equal to its weight. If the pel- A. The problem becomes more has an index of refraction n and a
lets rebound from the box down- challenging to solve if the pellets hit mass density p.
ward with the same speed, then the the box at an angle. Assume that the Assume that the prism is placed
momentum change of each pellet is pellets are identical to those in the in a laser beam aimed horizontally
given by example, but that they hit the box at in the x-direction. (Throughout this
:2mvs' an angle of 53'from the vertical. problem assume that the prism does
APP"11"I
Once again, the pellets rebound at not rotate-that is, its apex aiways
where vo is the initial speed of the the same speed (50 m/s) and at the points opposite to the direction of
pellets and m is the mass of each same angle. (Assume that the pellets the laser beam, its triangular faces l<
pel1et. The impulse-momentum hit at random orientations about the C
f
theorem and Newton's third law tell vertical so that there is no horizon- lThe entire XXIV International 00
a
us that the beam of pellets exerts a tal component of the net force.) Physics Olympiad Examination has (U
E
force on the box equal to What is the heaviest box that can been published in Physics Today o
(November 1993) in an article by F
remain suspended? Anthony P. French, chair of the _o
F66,.: RAppellet,
What do we do if the object to be examination committee. t

38 l'tAY/JUltr 1s84
the net force exerted on the prism by law tells us that
the laser light when the apex of the
" n-Q
prism is displaced a distanceyo from
the x-axis, where h s yo ( 3ft. If we
--o^ -
-r
want the prism to be suspended,
should the prism be placed above or where Qo is the total charge en-
below the axis of the laser beam? closed by the surface and the minus
D. PIot graphs of the values of the sign is included because the electric
horizontal and vertical components field is directed into the sphere. Be-
of force as functions of the vertical cause
displacement yo.
Q, = ooA,
Figure 1
E. Suppose that the laser beam is
1 mm wide in thez-direction and 80 where oo is the Earth's surface
are parallel to
the xy-plane, and its pm thick (in the y-direction). The charge density, we can solve for ei-
base is parallel to the yz-plane, as prism has the following characteris- ther the charge density or the total
shown in figure 1.) Take the index of tics: cr = 30, h: 10 pm, n = 1.5, w : charge. Let's find the charge density:
refraction of the surrounding air to 1 mm, andp = 2.5 glcms.Howmany
be n",, = 1. Assume that the faces of watts of power would be required to 06 = -€oEo
the prism are coated with an balance this prism against the pullof
antireflective coating so that no re- c2
= )(t.oN)
gravity when the apex of the prism =[-r.ru.1o-I2N'm'/\
flection occurs. The momentum of is at a distance yo = 2h : 20 trtrr,? I c)
a photon is given by p = Elc. This problem is certainly difficuit
The laser beam has an intensity enough. Olympiad students from4Z = -1.33.10-e +,
fiao
that is uniform across its width in countries took the problem one step
the z-direction but falls off linearly further and solved parts C, D, and E where we have replaced the units
with the vertical distanceyfrom the for prism positions where y6 < hl V/mbyN/C. Theminus signtells us
x-axis such that it has a maximum And some of them correctly com- that the charge on the Earth is nega-
value 1o at y = 0 and falls to zero aty pleted this analysis within the allo- tive, which we also know from the
= 4h (fis.2). cated time of 100 minutes! direction of the electric field. We can
Please send your solutions to now find the total charge on the
Quantum, 1840 Wilson Boulevard, Earth:
Arlington YA222OI within a month
after receipt of this issue. The best Qo = o# = oo4nRz = -6.85 . 105 C.
solutions will receive special certifi- Part B required our readers to cal-
cates from Quantum. culate the average net charge per cu-
bic meter of the atmosphere given the
_=_+ Elednidly in lfie ah electric field at a height of 100 m.
_--+ In the November/December issue Many students at the International
__+ of. Quantumwe asked our readers to Physics Olympiad solved this part of
use Gauss's law to examine the elec- the problem by considering the
tric field near the Earth's surface. We gaussian surface to consist of two
--+
Figure 2 will follow the solution provided at concentric spheres, one with a ra-
the International Physics Olympiad dius R and the second with a radius
B. Write equations from which held in the United States in |uly R + h with h = 100 m. However,
the angle 0 (see figure 3) may be de- 1993. since R << h, the Earth's surface is
termined in terms of tx and n for the Part A of our problem was to find relatively fiat on the scale of the
case when the laserbeam strikes the the total charge and charge density problem. Therefore, it's simpler to
upper face of the prism. on the Earth's surface given the elec-
C. Express, in terms of.Io,0, h, w, tric field near the surface. We begin
and yo the x- and y-components of by assuming that we have a spheri-
ca1 gaussian surfac,e that is only
slightly above the Earth's surface.
Therefore, the radius of this surface
is R. Because the electric field points
radralLy, the total electric flux
through this surface is just the prod-
uct o{ the surface area of. the sphere
Figure 3 A and the electric field Eo. Gauss's Figure 4

40 Ii|AY/JUlllI IgO4
consider a cylinder with a cross-sec- upward. Therefore, only the positive
tional area S and a height h sitting ions can contribute to the cancella-
just above the Earth's surface, as tion of the surface charge density. Yes, yuu calt UEt
shown in figure 4. The current per unit area i is grven by
The walls of the cylinder do not hack issues ol
i = n*ev = (1.44 . r}-t+lE,
contribute to the electric {lux, be- t1t; i.i,.r.
.; tl ll '
,1t,, ,il, ,
cause the electric field is parallel to
the walls. Therefore, Gauss's law
where we have used the values and
relationship given in the statement
'r!, . ,.i, r;, .,,,,r',i 'i. r "r ', 1,, ,,,,:l ", I
tells us of the problem. Note that the con- Quantum began publica-
stant must have units of A/V.m. tion with the ]anuary
s(Eo-E,oo)=?=f, Now, I is the rate of change of the 1990 pilot issue. All back
surface charge density ltolttl, andf, issues of. Quantum [ex-
= -oleofrom part A. Therefore, cept September/October
where p is the average charge den- 1990, which is out of
sity inside the cylinder and the con- 4=-1.68.10-3o=- 1o. print) are avatlable for
tribution of Eroo to the flux is nega- ar 613
purchase. For more infor-
tive. Using the data in the problem, mation and prices, call
this yields This is the same relationship that
we encounter in radioactive decay.
Therefore, its solution is an expo- 1 800 777-4643
p = - E oo) = 4.42' to-''
?(Eo ,rq. nential decrease of o with time: or send your order to
o(t) = oos-tLc'
Quantum Back Issues
Notice that the charge density is
positive. with t : 613 s. Putting oltl: ool2 Springer-Verlag
To solve part C, we first note that gives PO Box 2485
under the infiuence of the electric Secaucus NJ 07094
field, the positive ions move down-
t:tin2=425s=7min.
ward and the negative ions move o

wHrqH GAME FTRST, TrrE SHIqKEN OR THE EGG?


Invite Stephen Hawking into your classroom to explore the nature of the universe,
black holes, and the mystery of time with the "A Brief History of Timerr Educator's Kit!

TTTIs NEw KIT INqLUDES:


. of the critically acclaimed film, "A Brief History of Time"
a video
o the book, 'rA Brief History of Time: A Reader's Companion,"
edited by Stephen Hawking (retail price-$25.00)
. a teacher's guide with classroom exercises, and discussion questions
o the rrBrief History of Time" quiz
THE ENTTRE KtT CAN BE YOURS FOR $39.95!
$-
tem # orderecl Unit qrice Total F
ORDER the "Brief History of Time" Educator's Kit by
I
z
A BRIEF HISTORY OF TIME EDUCATOR'S KIT $ 19.95 o
FEBRUARY 1, 1994 AND RICBIVB A FRXE Sales tu for Illinois residets,T.5Vo
IIBRIEF HISTORY OF TIME'I POSTER ] Shipping and handling, $4.)0 per item
l
hletbod of payneut
\THILE SUPPLIES LAST. SHTPPING INFORMATION J N{l check or moner order is enckrsecl.
Ship to: f Visa E Nlastercard J American Exprcss

ORDER DIRECT BY CALLING I AOO 433 9833


School Erpirirlon Drre

Or mail the attached order form to: Add.*


Triton Pictures, P.O. Box 57 6, kasca, lL 60 1 43 -0 57 6
G,)/&*/24, Billing Addrcss of Cr..tit Cail

Pleav allaw 2-3 wuks far shippirig Davrine Pho.€:

OUAIIIUllll/PllYSICS CIlllIISI 4l
QUANTUM
SM I LES

A lnalltolnalical handhoolr
urith lto IiUul'E$
A little book that gives endless "dolgics"
of reading pleasure to children of all ages

by Yuly Danilov

EGEND HAS IT THAT WHEN credulously open the book and interrupting one another, recite
the famous American ptzzle- promptly lose yourself in a world of their favorite problems from Oster's
meister Sam Loyd invented curious characters struggling with little book. And it's easy to under-
Taquin (or "15"1, it was greeted unmathematical problems: captai.n stand why. How would youreactto
with almost disastrous enthusiasm. F1int, boatswain Fedya, sad uncle "problems" like these (the first ones
Farmers stopped farming, shopkeep- Borya, little baby Ktzya, the she- in the Problem Bookl?
ers forgot to open their shops in the goat Lucy, the scientist Innokenty of 1. In the tiny hold of a pirate ship
morning, government bureaucrats world renown/ a criminal, the Captain Flint and boatswain Fedya
stood all night long under street criminal's grandma, a cannibal, kit- divided one and the same dividend
lamps, trying in vain to solve a tens, ducklings, an elephant, per- by different divisors: Captain Flint,
puzzle that looked simple but was sons from other planets/ an octopus/ with a dark smirk, divided by 153,
really unsolvable: to make two Bryaka, Mryaka, Slyunik, Hryam- and boatswain Fedya, with a pleas-
small squares numbered 14 and 15 zik, ar-.d many others. It's an en- ant smile, divided by B. Boatswain
change piaces without taking them chanting blend of fairy tale, thriller, Fedya got a quotient of 612. What
out of the frame. adventure novel, and probiem book! quotient did Flint get?
Something like that happened, Most people catt't tear themselves 2. The personal paffot of Captain
and continues to happen, with those away from the Problem Book until Flint learned I,567 swear words in
lucky enough to get their hands on they've read it cover to sover. different langu ages: 27 | swear words
a copy of a little book by the poet The book has led to untold dra- in English, 352 in French, andI27 in
Crigory Oster with the seemingly mas, and even tragedies. People are Spanish. The rest of the swear words
innocent, but actually subtly ironic, Iate for work, a son argues with his the parrot got from the great and
sarcastic, and even insidious title {ather about who will read the book powerful Russian language. How
Prcblem Book. Without paying at- first, but while they struggle on, the many swear words did Captain
tention to the subtitle-"A Math- boy's mother picks up the Problem Flint's personal parrot get from the
ematical Handbook with No Fig- Book and won't let go until she's fin- Russian language?
ures," which to some extent should ished. Hoary scholars greet each 3. Ten pirates divided among
warn of impending danger, 1 you other like the courtiers in Hans themselves in equal shares L29 cap- o
-:<
c
lThe subtitle-Henarnsruroe noco6ue Christian Andersen's fairy tale "The tured maidens, and the rest were put o
.C
rro MareMarr{Ke-contains a rather
Emperor and the Nightingale"2 and, in a boat and sent home to their par- o
-C
outrageous pun. Harnxgnrni means ents. How many captured maidens o
(U

"visual" (something you can easily 2For those who haven't reread were sent back to their parents?
understand by looking at it); while Andersen for a while, I'11 explain:
nenarnsm*trT-with the negative prefix =
when three persons would meet, the third "-gale"-ss great was their
ue added-usually means "beloved', instead of "Good day" the first would admiration for the mechanical _o
(someone you can't see enough of)! say "Nigh-," the second "-tir:.-," arrd nightingale presented to the emperor.

42 trtAY/JUrrE tss4
ft {-/
&
((. s
d ,:,:r'l
r
S
{
q/
r
rd

s d
'J.J

-(
.L
'i
t
,t .{
Jl' $
c{ x*
p
L
t'
L
t t
$ r
J
1
d l,r
dr 1
-(
Ll{. f
0
o
tn
{( "f
f-'""td" 7
> 0
,J
-1. r F

rtl o
,{ tr
J o
,
ri
Y ,+*
$
rr '\
rfg
.{g
,'0
\ \
r.'
c
d .)\
1'.
'1'
.}-

,Y ;,

AJ
.b
ry'

s G $ fiP 3 E) {:} P,J

srrar:y -r, ?$
4. To find a hidden treasure, you as a matter of. fact. All of them are with the same marfoofochka on her
need to start from the old oak and go solvable and help cement the mate- whatever. The question is, how many
12 steps to the north, 5 steps to the rial studied in the classroom. But the times was the poor marfoofochka
south, 4 more steps to the north, and main aim of theProblem Bookisn't tail and kryacked on
grabbed by the
11 more steps to the south. Find to cement any material, and the somebody's whatever?
where the hidden treasure is. problems have nothing in common 4. Bryaka hid 3 poosics under a
5. The 14 best friends of Captain with what is called 'recreational coolyuk, shoved 5 poosics in a
Flint, after various pirating exploits, mathematics.'I don't think these mliiechka, and buried 12 poosics in
were left with one leg each, and Cap- problems will eiicit any professional a gryazinuce. Mryaka went out to
tain Flint was left with two legs'. interest from math olympiad cham- look for Bryaka's poosics, found l7
How many legs, not including pions. These problems are for those of them, and droosed them into
wooden ones, could be counted un- who don't like mathematics, who hroonichkas. Where did Bryaka
der the table when all t5 men sat find it tedious and enervating to most likely find his undroosed
down to eat breakfast? solve problems. Let them have their poosics?
So, do you like them? So do I. doubts about itl" 5. Mryaka and Bryaka came to a
Many people have tried to imitate For kids: "Dear kids, this book is meadow and started to jump.
Oster, but all have fa11en short. The calledProblem Book on purpose. It's Mryaka jumped on 7 lygs and
Problem Book even got an official so you can read it on your lap behind Bryaka iumped on 8. How many lygs
Ministry of Edu-
seal of approval: the your desk. And if your teacher gets remained uncrushed if 39 lygs had
cation of Russia (no joke!) recom- upset/ just say: 'I don't understand. been sitting in the grass, softly sing-
mended the "mathematical hand- This book has been approved by the ing their pensive song?
book with no figures" as a textbook Ministry of Education."' 6. Mryal<a and Bryaka found a
for schools ! Apparently bureaucrats Needless to say, the spoofing, chalochka that was 9 tyatoosics
in the Ministry of Education actu- high spirits, and inventiveness dis- 1ong. Mryaka nibbied 4 tyatoosics
ally have a sense of humor and ap- piayed by Oster are enjoyed greatly and gave the rest to Bryaka. How
preciate a good joke. not only by those who don't like long in dlinnics was the piece of
And when, after they had read the math but also by olympiad champi- chalochka that Bryaka got? (Keep in
Problem Book over and over, from ons. And their teachers, and the mind that there are 7 dlinnics in a
cover to cover/ and had learned their teachers of their teachers-profes- tyatoosic.)
favorite problems by heart, charmed sors of mathematics-were filled 7. Mryaka, Bryaka, Slyunik, and
readers turned at last to the foreword with the warmest feelings (and the Hryamzik walked and walked and
(because a real reader reads a {ore- darkest envy) toward the author, walked, covering 200 dlinnics in 5
word only after diving deep into the because they didn't come up with dolgics. How many dolgics will it
book itself), they found that the au- the idea of creating such wonderful take for them to cover 360 dlinnics
thor had no intention of misleading problems. if they walk and walk and walk with
them. He had honestly warned ev- Now I think it's about time I of- the same velocity?
eryone, children and adults alike, fered those who haven't seen S.ItHryarnzik is cailed a sl1.unik,
about his intentions. He even went Grigory Oster's Problem Book a he starts to butt and doesn't stop un-
the extra mile and wrote two (or few more problems, taken almost til he has butted the offender 5 times
possible three, depending on how at random. with each horn. One day Bryaka
you count) forewords. Here they are. Problems called Hryamzik that very thing, and
1. Mryaka drooses poosics. To Hryamzik butted Bryaka 35 times.
tol'ewords droose one poosic it takes Mryaka a How many horns does Hryamzik
"Would you like to hear a sadis- half-dolgic. How many dolgics will have?
tic joke? One day a children's author Mryaka spend droosing 8 poosics?3 9. Every time they go out for a
comes to his readers and says: 'Dear 2. Mryaka and Bryaka droosed a walk, Mryaka puts on 3 foofiras,
children! I have written a new book poosic. Mryaka took for herself 2 while Bryaka puts on only 2. Both of
for you-a mathematical problem farics, and Bryaka took 1. How many them always return home buck na-
book!'That's like getting a bowl of hroonechkas does Mryaka have, and ked. How many foofiras did Mryaka
oatmeal instead of a pretty cake on how many does Bryaka have? and Bryaka lose in one summer i-f it's
your birthday. But to telI you the 3.Bryaka and Mryaka quarreled. known that Mryaka went for a walk
truth, the book you're looking at Mryaka kryacked Bryaka 7 times 150 times and Bryaka 180 times this
isn't exactly a problem book." with a marfoofochka on his whatever, summer?
Here the text splits: one foreword and Bryaka kryacked Mryaka 9 times I0. If Slyunik is teased, she begins
for kids, one foreword for grown-ups. to kick and doesn't settle down un-
sThere are 3 farics in a poosic, 5
For gtown-ups: "Don't wotry, til she kicks the teaser 3 times with
blyakas rnafaic, and 2 hroonichkas
don't worry, these are realproblems. in a blyaka. (You'Il need this for the each leg. One day Mryaka called
For second, third, and fourth grades, next problem.) Slyunik ahryarrrzik, and Slyunik

44 l,tAY/Jtttrtr I os4
kicked Mryaka 27 times.How many to do with "dolgiki"? [onna7 ("dolgy")
legs does Slyunik have? also means long-but "tirrre" this time!
11. One day two numbers-S and English readers can perhaps imagine a
3-came to a place where a lot of similar work written by Lewis Carroll or
different di{ferences were scattered Ogden Nash, but we hope they experi-
about, and they started iooking for ence some of the giddy silliness that
their own. Find the difference of Russians feel when thev read Oster's
these numbers. exuberant coinages. o Figure 2
12. Once upon a time there lived
two numbers-S and 3. They had a tual inductance of the rings, the
sack of arerage size that they took ,,HOW DO dispiacements of the rings while
with them wherever they went. YOU FIGUREI" field is turned off, and ar,y fuic-
When they came across something CONTINUED FROM PAGE 19 tional effects. (V. Mozhayev)
dangerous, they would quickly jump
into the sack, close it from inside, P1 14 Pl 15
and press against each other so Rings on the move. Two identical Image vs. reality. A point source of
tightly that sometimes they became wire rings of radius rt each of mass light moves parallel to the principal
one number. And then the sack mt are located in a homogeneous axis of a converging lens with focal
would contain their sum. Find the magnetic {ield B directed perpen- length F. Determine the distance of
sum of the numbers 5 and 3 in the dicular to the plane of the rings and the source from the lens when the
sack. into the page (fig. 2). The rings make absolute value of the velocity of its
Editor's note: We have retained the electrical contact at the points of in- image is equal to that of the source.
Russian flavor of the nonsense words in tersectionA andC.What is the ve- The distance from the source to the
these problems. A true English transla- locity that each ring gains when principal axis is H = F14.lA.Zllber-
tion might turn "dlinniki" into the magnetic field is switched off ? man)
"longies," for instance, since [nltrrrufi The eiectrical resistance of each
("dlinny") means long, as in "long dis- ring is R and the angle o : m/3. Ne- ANSWERS, H/NIS & SOLUTIONS
tarrce.'t But what is the poor translator glect the self-inductance and mu- ON PAGE 54

Only $ZO per set!


RJ/,IIPI Catalog #MC-05

Available in Electronic Format


for the First Time
This electronic database indexes both
lhe American Journal of Physics
(1972-92) and The Physics Teacher
(1976-92), indexing every word/name in
every article title and author's address,
and every word of author supplied
abstracts tor AJP. Computer
requirements: DOS compatible machine
with 12 megabytes of hard disk space
and 640K memory. Available in both 1.44
meg S.s-inch disks (3 disks per set) or
1.2 meg 5.25-inch disks (3 disks per set).
Please specify disk size when ordering.

AAPT welcomes your written


comments concerning this database, but
Send order to: at this time is not planning any future
American Association of Physics development on this project. AAPT does
Teachers, One Physics EIIipse, not offer phone suppon for this program,
College Park, MD 20740-3845 but will give a full refund if the user is
Phone: 301 -209-3300 unable to access the database.

OlJAllJIU]i4/OUA[IIUllll S]i{IIIS 45
AT THE
BLACKBOARD II

Illine solulionslo olte pl'ohlem


And integer angles galore

by Constantine Knop

HE PROBLEM THAT I'M GO. find IFGE: 180' - 50' - 80" = 400, from a problem book, uses two ad-
ing to discuss has appeared re- and zEFG:40" lsay, from triangle ditionally constructed points, and
peatedly in geometry problem BGF). This reveals another isosceles five triangles are under consider-
books. Although its statement triangle, FEG (FE : EGl.Finally, by ation. It couldn't be called too com-
seems simple, it's very difficult to the SSS property, triangles EFD and plex or too long, really, and yet it
solve. This might be why most of EGD are congruent/ and so didn't seem very elegant or beauti-
the books give more or less the same ful to me. So when, after thinking
solution, and not even the best one, 60o long and hard, and unsuccessfully, I
IEDB = Z.EDF= = 3go.
in my view. But let's start with the 2 finally found another solution, I was
statement. hrppy. The only thing that dis-
Problem. In an isosceles tiangle I want to point out one fact men- tressed me was that this solution
ABC, AB : AC and the angle BAC tioned in this proof that will be re- was analytical (trigonometric) rather
measures 20o. Points D and E are peatedly used in what follows: the than geometric.
taken on the sides AC and AB, re- trianfl,eBCE is isosceles-that is, BE The second solution. Let x be the
spectively, such that angle ECB is = BC. measure of the unknown angIeEDB.
50" and angle DBC is 60'. Find the Well, this solution, borrowed Then ZBED : 160' -x. By the Sine
angle EDB.
A Law, from triangle BED we find
Before you go on/ try to solve the BD : BE = sin (160" - x) : sin x, and
problem on your own. Give yourself from triangleBCD, BD : BC: sin 80'
a few hours (or maybe minutes) to : sin 40o : 2 cos 40'(since IBDC =
think it over. You'llfind real pleasure 180'- 50" - 80"1.
. . . if you manage to find the answer. Using the aforementioned equal-
The first solution. Draw segment ity BE : BC, we get the equation
DF parallel to BC with F on AB
(fig. 1), draw CF,label as G the in- sin(150"-x)
\ /
tersection of BD and CF, and draw =2cos40o.
sinx
GE. Clearly the triangleBGC is isos-
celes (withBG = GCI and, therefore, Let's rework and solve it:
equilateral (since ZDBC : 60").
Then the triangle GDF is equilateral sin (20' + xl :2 cos 40o . sin x
as well. Further, we notice that = 2 cos (60'- 20') sin x; ll
C
ZBEC : 180' _ ZBCE _ ZCBE= lBO' sin 20' cos x + cos 20o sin x cOc)
- 50'- B0' = 50o (ICBE can be found 20' + rE sin 20') sin x;
= (cos ca
(d
from the given isosceles triangle sin 20o cos x : "uE sin 20o sin x;
ABCI, so IBEC : ZBCE, which f
1
means that triangle BGEis also isos- tanx=-z
t;'
celes (BE : BC = BGI withEBG = 80' !o !

-50'= 20" and/.EGB = B0'. Nowwe Figure 1 x = 30o.

4B ttIY/JUirr rso4
Trigonometry is a powerful and and zGCD: 80'- 200, so
500 :
universal tool. But does our problem ZHBE = IGCD.
realiy have no other geometric solu- Therefore, the triangles BEH and
tions? CGD are congruent by the SAS
Fortunately, such a solution does property; consequently, ZBHE =
exist-in fact, there are a number of ICDG:40 :Yz Z-BCD: YzlBHD.
them. I created the next two in sev- It follows that HE bisects IBHD,
eral hours of leisure time. and at the same trme, BE bisects
The third solution. As with the ZHBD (since ZHBE : ZDBE :20"1.
first solution, I'11 again try to prove Therefore, E is the center of the
that DE is the bisector of zeOP incircle of triangle BDH, and DE is
(fig. 1). To this end, I'11 create atri- the bisector of z.non.
angle, one of whose angles will be This was a different solution, but
BDF, with its incenter at E. Draw it's hardly simpler than the first one.
DH andBHparallelto CB andCD, The next solution seems more at-
respectively, to obtain a parallelo- tractive to me.
gramBCDH lfi1.2l. Draw CG as we The fourth solution. Mark point
did in figure I (to make an equilat- K on AC such that IKBC = 20 , and
eral triangle BCGI. Now we have join it to B andE (see figure 3 cin the
lll BH = CD lbv a property of the next page).
parallelogram); Then . . . On second thought, why
(21BE=BC=CG, don't you try to finish this proof
(31ZHBE = ZHBA = ZBAC =20", Figure 2 yourself?

: t-
L, .1. -,
$._ '
5i*i,.,,..
ts*r:

tll
,

:,iij
:::8:-

'
,
auxiliary construction based on other hand/ as we/ve already seen,
drawing the bisector of the angle B BD bisects angle EBT, so D is equi-
of the given triangle ABC. All three distant from the lines BA, BT, and
solutions use the f.act that this line ET, and, therefore, ED is the exterior
is also the perpendicular bisector of bisector of triangle BET at E.
the segment CE (sinceBC = BEl,b* Note, by the way, that D is the
other than that they are surprisingly excenter (center of the escribed circle)
different. of this triangle. Now we tirrd ZBED
The fifth solution (Maria Gel- : IBET + ZTED = I.BET +lVzlBO' -
band). Let M be the reflection of E IBETI:90 +Yz ZBET = 130o, because
across AC (fig.4). Then CE = CM ZBET : ZBCT: B0'. Finally, from
and zECtw = L.ECD = 60", so the triangle BED we get what we want:
triangle CEM is equilateral, CM = IEDB = 180'- 130'- 20" = 30".
EM, and therefore M lies on the bi- In the last part of this proof we've
sector mentioned above. Now we actually proved the following prop-
notice that the point D is the inter- erty of the excenter D of an arbitrary
section of the perpendicular bisector triangleEBT that lies inside its angle
of EM and BD, the bisector of the EBT: ZEDB = YIIETB (which is in-
angle EBM o{ the triang,le BEM dependent of specific values of the
(.EBD = 20o = y2 BMl.It follows that angles).
D lies on the circumcircl e of BEM llf The seventh solution (Alexey
Figure 3 D, is the intersection o{ the circum- Borodin). Consider the circumcenter
circle with the perpendicular bisec- O of the triangle EDC. Since EO =
Exercise 1. Prove that the marked tor of EM, then ED, = DtM, and the OC, the line BO is just the bisector
segments in figure 3 are congruent angles EBD , and MBD, inscribed in of the angle EBC used in the two
and use them to find the unknown this circle, are subtended by congru- previous proofs.
ang1e. ent chords ED, and DMr; so these Exercise 2. Finish this proof using
After I found these solutions, I angles are equal, which means that figure 5. (Hint:prove the congruence
was obsessed with the idea of offer- Dr= Dl. Thus, the angles EDB and of triangles BED ard BOD.I
ing this problem to students at some EMB are inscribed in this circum- Perhaps one of the most natural
serious math competition. These circle and subtended by the same ways to tackle our problem is to
clever kids might discover some- chord BE. So IEDB : IEMB = notice that the measure of ZA of the
thing newl Unexpectedly, about a YILEMC = 30o, and we're done. given triangle is 2O"-that is, one
year ago my dream came true: the The sixth solution (Sergey Sapri- third of 60'-and try to make use of
problem was proposed to the candi- kin). Let the bisector of IABC inter- this observation. This idea is imple-
dates for Ukraine's International sect AC at T (fis,.5). Then ZETB = mented in the next solution.
Mathematics Olympiad team. And ZBTC (why?). But IBTC = lB0" - The eighth solution (Alexander
my collection of solutions grew by 40" - B0'= 60o, so ZETD = 60o, and Kornienko) ({ig.7l. Reflect the given
four new items. It's interesting that 7D is the bisector of the exterior triangle aboutAB (to getABC,)and
three of these solutions involve an angle of triangle BET at 7. On the AC (to get ACB1\. Then ZACrE :

Figure 4 Figure 5 Figure 6

40 ]i4AY/JlJlrr 1gg4
\

Figure 7
ZACE = 30o, ar,d lACrB, : 60o
(since triangle ABrCr is equilat-
erul-ACr= AB1, ZCIAB. = 60"), so
CrE bisects ZACrB' which means
that CrE is the perpendicular bisec-
tor of ABr. On the other hand, AD : MATHEMATICS
BD (because ZABD: IBAD:20'1,
a:ad BD = BrD (by construction). So CONTESTS
D is equidistant {romA and B, and, A Guide for
therefore, lies on line CE. Now the lnrmlving Students ond
angle EDB can be found from tri-
Schooh
angle CrBD, in which IBC.D = B0
-30': 5O andZC,BD:80'+ 20o = Frederick 0. Flener
100'. We find that Z.EDB: 100' -
50"-80'=30".
(In fact, the idea underlying this
proofis to considerfigure 7 apartof
the regular 18-gon centered at A arrd
segments CrB, BC, andCBras three
consecutive sides. Segments CrD book you can use right away!
and BD turn out to be parts of its
Written by an experienced mathematics
two diagonals-which has to be
proved, of course, but this enables us contest coach, question writer, and
to quickly find the unknown angle.) consultant. An invaluable collection of 100s
I hope you liked these eight solu- of contest problems. Answers included. Also
tions, full of many clever construc- contains ideas for coaching and tips on how
tions and useful properties of tri- to start a mathematics contest. 1 15 pp.,
angles. "But where's the ninth one? "
you ask. Why, I've left it for you to lsBN 0-87353-2821, #4O7, $13.*
find!
I'11leave you with another prob- To order call tol!-free 1-800-235-7566
lem about the same triangle (pro- or fax your order today.
posed by a ninth-grader, Sergey *20% discount to NCTM members.
Yurin).
Exercise 3. In an isosceles triangle National Council of

ABC, AB : AC, an.d ZA :2}".Point


Teachers of Mathematics
1906 Association Drive
P is taken on the side AC such that Reston, VA 22091
Fax: (703) 476-2970
AP = BC. Find the ingle PBC. O

ANSWERS, HINTS & SOLUTIONS


ON PAGE 60

OlJAlllIUll,l/AI TIII BI.ICI(OOAfl[ II 4g


HAPPENINGS

A IalB ol one cily


It was the best of times . . .
Problems from the lnternational Mathematics
Tournament of Towns in Beloretsk, Russia

by Andy Liu

HE ROAD NETWORK OF A solving Workshop conducted in carefully constructed. It is divided


certain city consists of a con- Beloretsk, in the Bashkirian Repub- into many questions, leading the
tinuous chain of circles. At the lic of Russia, from August 1 to Au- solvers step-by-step to the main one.
point of tangency of two adja- gust 9, 1993. The participants were It also raises other related questions
cent circles, the roads cross over as mostly from the former Soviet bloc, along the way. The problem we fea-
shown in figure 1, which illustrates but included representatives from ture here is proposed by S. Loktev
the case with four circles. England, Austria, Canada, Cer- and M. Viaiyj, based on a problem of
A ring road is constructed around many, and Colombia. They were Prof. V. Arnold.
the city, and is integrated with the high school students invited on the We begin our investigation by
inner chain at various points. At strength of their performances in the studying some small integrated net-
International Mathematics Tourna- works.

:XXX) ment of Towns.l


Each of the five problems is very
Question 1. Which of the inte-
grated networks in figure 3 are regu-
Iarl
It may be observed that each of
0 the irregular networks in figure 3

Figure 1
:XXX) 0
consists of two components. Is this
true in general? In other words, how
bad can an irregular network be?
Question 2. Is there an integrated
network that consists of three or
each "integration point," the ring more components?
road is crossed over with the inner Some of the integrated networks
chain as shown in figure 2, which il- in figure 3 have, at the end of the
lustrates the case with two integra- inner chains, circles without inte-
tion points. gration points. Clearly, the deletion
Note that in figure 2 the integrated of such circles does not affect regu-
network consists of two mutually larity. From now on/ we shall as-
inaccessible components. We call an sume that they have been deleted. If
integrated network regular if it con- the inner chain consists only of
Figure 2
sists of only one component/ and ir- such circles/ it will be reduced to
regular otherwise. We wish to find a the empty set. However, the inte-
lFor more on the Tournament of
necessary and sufficient condition {or grated network is stil1 considered to
an integrated network to be regular. Towns, see the articles in the
Happenings department in the |anuary
be irregular, although the ring road
This is the main question in one 1990 and November/December 1990 now constitutes the only compo-
of five problems posed in a Problem- issues of Quantum. nent.

50 l,tIY/JUirr lss4
/TT\ /fi-l 3b, and 3k.

@@
Question 6. From a binary se-

t99l t9?J quence/ we delete all subsequences


of the forms 00 and 111. We also re-
place all subsequences of the form
101 by 0. Prove that the reduced
a f k p

/TT\ sequence represents a regular net-


work if and only if the original one

@@@ r?gl
does.
It turns out that it is possible to
determine whether a binary se-
quence represents a regular network
b
m g I q
or not without act.ually carrying out
the reduction process in cluestion 6.

@ @ t99' Question 3. Consider the sequence as blocks of


Prove that nei- consecutive I's separated by singie
ther of the fol- 0's. For example, 111001101 con-
lowing opera- sists of four blocks of consecutive
h m

6)
C tions affect the 1 's, with 3 , 0, 2 and 1 of them in the

regularity of an respective blocks. The altemate sum

\_iJ @ @
integrated net- of these numbers is 3 - 0 + 2 - I = 4,
work: (a)adding and the altemate sum of any binary
two integration sequence can be defined in the same
points to a circle way.
d i n in the inner Question 7. Prove that a binary

4-T\ chain; (b)remov-


ing two integra-
sequence represents a regular net-
work if and only if its altemate sum

@ @ t99'
e j o
tion points from
a circle in the
inner chain.
A simple but
useful corollary
of question 3 is
is not divisible by 3.
Amazingly, what we have gone
through so far constitutes only part
of the problem of Loktev-Vialyi. A
more general setting replaces the
city with a metropolis, where the
Figure 3 the following. It road network is not a chain but may
makes figures look like figure 4. There are also
As we are interested only in the 3d, 3f, 3i, 31,3n, 3p, and 3q irrel- other questions one can ask about
regularity of integrated networks, evant. integrated networks.
we can simplify them in many Question 4. Prove that in an inte- The top prize winner for this prob-
ways. The following is also moti- grated network, we can move all in- lem at the workshop was Clemens
vated by observations of the simple tegration points in each circle of the Heuberger, a graduating high school
cases in figure 3. inner chain to the same side of the student frorrr Graz, Austria. Other
ring road. winners were M. Alekhnovich and
A more important corollary of M. Ostrovsky (joint effort), and I.
question 3 is that we may assume Nykonov. All three were high school
that each circle of the inner chain students from Moscow, Russia.
has either one integration point or The principal organizer of the
no integration points. We say that workshop was Prof. Nikoiay Kon-
such an integrated network is nor- stantinov of the Independent Uni-
malized, and represent it by a se- versity of Moscow, a recent winner
quence of 0's and 1's. of the prestigious Erd6s Award from
Question 5. Which of the follow- the World Federation of National
ing normalized networks are regu- Mathematics Competitions. The as-
lar? (al101 (b) 111 (c) 1001 (d) 1011 sembly was honored by the pres-
(e) 1111 (f) 10001 (g) 10011 (h) 10101 ence of Prof. Nikolay Vasilyev,
(i) 10111 (i) i1011 (k) 11111. chairman of the problem commit-
The next result allows us to re- tee of the Tournament of Towns,
duce every normalized network to and Prof. A. A. Yegorov, an editor
Figure 4 one of three in figure 3-namely,3a, of Kvant, the sister journal of

OtlAiIIUil/IIAPPINIiIES 5 I
Quantum. Both posed problems for Prove that
the workshop.2
Most of the group assembled at a = b (mod pl * aP : bP (rnod pzl.
Moscow on fuly 30, 1993, and took (blABCD is aparalleloeram (fig. 5).
a 35-hour train ride across European The circumcircle of triangle BAD,
Russia to Beloretsk. Five problems with center O, cuts the extensions of
were distributed on board to whet BC, AC, DC, andAO atK, L, M, and
the participants' appetites. We con- N, respectively. (1) Prove that N is
clude with one of them. the circumcenter of triangle KCM.
Training Problem. (a)Let a andb (2) Express the length of.LC interms
be integers and p a prime number. of a: KL andb = LM. O
2See
"A Mathematical |oumey" by
A. Liu in Crux Mathematicorum, ANSWERS, HINTS & SOLUTIONS
Figure 5 IN THE NEXT /SSUE
lanuary 199a (pp. 1-5).

Bulletin Boal'd
Budapest Semestet's in lulatflematius rangements can be made for taking head to head in a friendly math-
Initiated by Paul Erd5s, Llszl6 the Putnam Examination in Buda- ematics competition in Moscow in
Lovlsz, and Vera T. S6s in 1984, the pest and/or for taking the Graduate the summer of 1995. The event will
Budapest Semesters in Mathematics Record Examination in Belgrade or be sponsored by American Univer-
program offers a unique opportunity Vienna, both about four hours by sity in Moscow and administered by
to North American undergraduates train from Budapest. the two magazines. Participants will
for a semester oryear of study abroad, During the past ten years, hun- be selected on the basis of answers
in one of the most advanced math- dreds of North American students, to several rounds of questions pub-
ematical centers of the world. representing more than 120 univer- lished here and irKvant. According
Through this program, mathematics sities, took advantage of Hungary's to Edward Lozansky, Quantum's
and computer science majors in their long tradition of excellence in math- international consultant, a team of
junior/senior years can take a v ariety ematical education and in creative five US high school students will be
of courses in al1 areas of mathematics problem solving through this unique chosen for the one-week, a1l-expense-
under the tutelage of eminent Hun- program. Many of them have stayed paid trip to Moscow.
garian scholar-teachers, most of in close contact with one another Watch this space in the months
whom have had years of teaching ex- and the factltyf organizers of the ahead for further details.
perience in North America. The program.
classes are small, all of the courses are To leam more about the Budapest Dul'acell Scholat'ship tltirlter$
conducted in English, and the credits Semesters in Mathematics, please Seventeen-year-old Tr acy Phillips
are transferable to the students'home contact its North American Direc- of Long Beach, New York, was the
institutions. tor, Professor Paul D. Humke, at first-place winner in the Duracell/
The classes are held on the Inter- Saint Olaf College (telephone: 800 NSTA Scholarship Competition.
national College Campus of the 277-0434 or 507 545-3113; e-mail: She invented Money Talks, an elec-
Technical University of Budapest, humke@stolaf.edu). You can also ob- tronic device neatly built into awal-
which is near the historic city cen- tain a copy of the application mate- 1et that helps blind people distinguish
ter. The accommodations are excel- rials and a brochure describing the between different values of paper
lent, the living costs are modest, and program via anonymous ftp money. After a bill is placed in the
the tuition is most reasonable. The (ftp.stolaf .edu). The registration wallet, the device "talks," giving the
fall semester usually begins during deadline for the fall semester is denomination. How does it know
the first week of September and ends April30; early applications are en- what to say? It uses an infrared light
before Christmas, while the spring couraged, but late ones are some- emitter/detector that lets varying
semester starts in early February and times accepted. The size of each amounts of light pass through the
ends in late May. There is a brief class is usually around 30. printed patterns of a bili, identifying
orientation program prior to the Berzsenyi key points that distinguish the bill's
start of semesters/ and one can also -George denomination. The amount of light
take part in an optional two-week 0uantum tts.lfialtt passing through each point is
language program prior to the begin- Student readers of Quantum and
ning of the regular program. Ar- its sister magazine Kvant will go CONT/NUED ON PAGE 61

52 ilAY/Junr tos4
xcr0$$scEllGE by David R. Martin
vi,
,?
E
1 2 3 4 5 7 8 9 10 t1 t2 13

c5 t4

t7
15

18
16

l9

20 21 22 23

24 25 ).6

27 28 29 t0 31 32 33 34

35 36 37 38 39

40 4l 42 43 14

45 4t5 47 48 49

50 51 52 JN 54
o
(o
JC 56 57 @
--l
J
58 59 60 6t 62 63 54 o
3
D
o'
65 66 67 68 69 o
o
70 71 7Z
o
=
o-
.E
73 74 75 cN
\r
o

49 douloureux 8 Devoured 55 Compounds 51 River near Mexico


Acl'oss
50 -Elementary particle 9 Dog's cries containing City
1 Group of legislators Dawn goddess 10 Plasma confining 62 Concept
52 -cH:c(oH)-
5 Common electric 53 Scandinavian device 57 Ancient 63 Lowest tide
circuit element 55 Sways ll 1949 Physiology Mesopotamian 64 TalLs
10 WiId duck 57 Understand Nobelist Moniz, region 67
14 OI aircrak 58 Am. inventor Antonio _ 58 God of love --&-dah
58 Dutch town
15 Decrease 61 Fluid acceleration 12 Chemical prefix 59 Dirt
16 Stare flirtatiously SOLUTION IN THE
machine 13 Sly smile 60 Unloaded engine
17 Birds NEXT ISSUE
65 Fashion designer 21 Halley's speed
18 Wooden pin
19 Hardy cabbage
Gernreich 23 Highly radioactive -
66 -
Disulphuric(vi) acid 25 Graceful tree
20 Semiconductor SOLUTION TO THE
69 First garden 26 Parts
MARCH/APRIL PUZZLE
22 Rotating vector 70 European capital 27 Yaporizedwater
24 Negative prefix 71 Scoop 28 Corridor S T A R H A L A S G L A D
25 Type o{ salt 72 Peruse 29 (blue rock)
27 Hawthorne's home
o R L E A P A C E L I M A
73 British gun -lazdj.anthropolo-
30 French
L A I D L E V E R U N I T
30 1051 74 Deforming {orce gist _ Mauss
31 1958 Physiology 75 Plant fluids 32 Flavor I C E S I R E o C T A N E
Nobelist Edward L. 33 Untied D E N S I T Y C L o E
34 U.S. dancer o N E B R o wN I A N
3S Aunt in Spain Cunningham-
Dout. B E N N E C R A G S T S o
36 Tooth covering 37 I and 101, e.g.: abbr.
39 "Crtizen _"
1 Sheep cries E V o E P L I N Y F E E S
2 symbol 38 Roman spirit
40 Intuitive letters 43 Monochromatic
T A R P L A N E S E A M Y
City on the Oka
3 --Civita
4l Crag radiation source A N A P H A S E S E E
4 Trigonometric
42 4.19 foules: abbr. 46 Twisting deforma- R o T S S T A T I C S
{unction
44 Orre who transmits: tion
5 Radioactive gas C A L I T E S T E M M A P
abbr. 48 Angeles
45 Landed
5 Black A L E S L o I R E C A D E

47 Eye's outer coat


7 Fundamental -
51 Hydrocarbon prefix
R E A M E L L U L o C R E
physical truth 54 Magnetic flux units
T E D S T E E M S S E L D

0tlAI'lIU lll/tfl l$SCR0$S SCI tilCt 53


ANSWERS,
HINTS &
SOLUTIONS

on the same side otBD, because the


(n+3)! ., segment PQ intersects BD. Then
3n the angles DQK and DLK are in-
tUlalh scribed in the same circle, subtend
--o:-1 . 3!r 2. 4lr n(n+Z\r.
... r ____l___________1 the same chord DK, and have their
Ml11 3323n vertices on the same side of the
(a) The equality can be proved by
- - chord, so ZDQK = ZDLK. On the
induction. It's obviously true for n = other hand, since DC ll AB, ZDQK
Similarly, for any positive integer d,
t: lr .2lll2 = 3tl2 -Z = L. Assuming = ZKPB. Therefore,
we have
that it's already been proved {orn = k, ZKPB + ZKLB
let's make sure it's true for n = k + 1. (l<+ d)l
_ (k + d-1)! _ /<(k+d-l)! = LDQK + (180'- IDLKI
Using the inductive assumption, we dk dk-r 6* :1800,
get
from which the following generali-
which means that points B, P, K, L
are concyclic-that is, I lies on the
zation of statements (a) and (b) can
t.2t 2.3! be derived:
ckcle KBP.
22)
-*"' dt, 2(r+ d)t,, n(n+ d-t)t Ml13
, k(k + I)! , (ft + I)(k + 2)! r.
- 2k - zk-'t d - d, -"'- trn The answerisyes.If thefust player
names any three different integers
_(k+2)! T-, (k+t)(l<+z)l _(n+ d)l )t whose sum is zero (say, l, -3, 21, then
-- ^
2R zk+r dn
regardless of the order chosen by the
second player, the resulting equation
Q.+k+ 1)(l<+z)! o
2k +t
(N. Vasilyev, V. Zhokha) a* + bx+ c : 0 will have a rootxl = 1
(because a. 12 + b . + c = 0) and a
I
3)!
(/< +
= 2k +I -)^
Ml12 different second toot x2= cla * L
Let I be the point of intersection (since the product xlxz is always
of the diagonal BD and the circle eqtalto cf al.
KDQ ffig. 1). We have to prove that If you liked this problem, here is
(b) The sum equals (n + 3l! l3 - 5. I lies on the second circle KBP. For a much more elaborate extension.
This can also be proved by induc- the sake of definiteness, we'Il as- Two players create an equation of
tion, as in part (a), but we'll give a sume that points K and Q are on dif- the form f + **+ xx+ * = 0. The
somewhat modified argument with ferent sides of BD; then K and P are first one names a number, the sec-
"telescoping" sums. Note that any ond writes it in place of any of the
term of the given sum can be writ- asterisks; then the first player names
ten as the difference a second number and the other
player inserts it in place of any of the
two remaining asterisks; {inally, the
(k+ 3)! _ (k+2)! _ (k+2)!(l<+ 3- 3) first player replaces the last asterisk
3ft 3L-1 3k with some number. Can the first
player ensure that the resulting
_ k(l< + 2)! equation has three distinct integer
3J<
roots?

Writing out all these differences M114


for k = 1,2, ..., n and adding them The specific problems (a) and (b)
up we get, after canceling out have specific solutions-based, for
terms of opposite sign, Figure 1
instance, on the Fractional Parts

54 l,tAY/JUirr rso4
Of course, this statement is true
for the transiation by B?as well.
Now let's prove that for q27 arry
q = 8 (even) circle contains an attainable point
inside it. Let e be the radius of the
circle. Take any two attainable points
A andB and construct C as speci{ied
in statement ( 1 ) (fig. 41. Then BC I AB
= k = }sin {180'/q) < 2 sin 30' : 1.
Applying the same construction to
B and C instead of A andB, we'llget
a pair of attainable points C and D
such that CD = kBC = kzAB (fig. a).
Then we repeat the construction
with C and D, and so on, until we
the initial one/ so only an even num- get attainable points X and Y such
ber of rollings restores the initial ori- thatXY :k"AB < e and also the next
Theorem from "Ones Up front in entation of the polygon-see figure point Z in the sequence A, B, C, D,
Powers of T.,r,o" rn the November/ 2.) The points connected to O will ..., X, ...(such that XZ : XY and
December 1993 issue oi Quantum. be called attainable. Clearly any IZXY = 360lql. Repeatedly apply-
But we'11 consider the general prob- two attainable points A arrd B are ing statement (2) to points X, Y, O
lem (c) right au-a1-. The solution be- connected to each other (we can and X, Z, O, we see that any num-
low involr.es some ideas from the construct a track from A to O to Bl ber of translations bv the vectors
solution to M100 in thar same issue. conversely, any point B connected xY- ,-xY' :Yi , or+VZ-.thatis,
First let's introduce a number of to an attainable point A is attain- a translation by vector nXY +
convenient notations and terms. Let able itself (there is a track from O mXZ with any integer m and n-
Q be the fixed initial position of the to A to Bl. takes O into an attainable point.
given regular q-gon and O its center. Next we prove two important These points make up a grid of
Denote by Q r the q-gon (with cen- properties of attainable points. rhombi with side length less than e
ter A) obtained from Q.under lll If points A and B are attain- (fis. 5).
translation b1- r'ector OA' . Two able and C is obtained from B un- The center of the given circle falls
points A and B are said to be con- der a rotation r through 360'/q into one of the rhombi, and it's easy
nected ii the polygon Qo can be about A, then C is attainable. to see that its distance from one of the
rolled into the polygon Qr. The se- Indeed, the rotation z takes Qo vertices of this rhombus is smaller
quence of its successive positions in into itself and any track AB (which than e. So this vertex lies inside the
this series of roils will be called the exists because A and B are con- circle, whichproves the statement of
track AB. {Notice that if q is odd, nected) into a trackAC (fig. 3). So C the problem f.or q> 7, because a1lthe
then an,v uacl< AB necessarily con- is connected to an attainable point nodes of the grid are attainable.
sists of an odd number of polygons A. For q = 5 we can take two attain-
(including Qo and Q6), because in 2l A A, B, and C are attainable, able points A and B, rotate B about
this case a single rolling yields a then the translation t by vector AB A through 3. 36l- 1 q = 216' to get an
polygon turned 180' with respect to takes C into an attainable point D. attainable point P, and translate A
To prove this, it suffices to con- bv FT. This yields an attainable
struct a track AC and notice that our
translation takes Q, and Q" into Q,
and Qa, and the constructed track
into a track BD. So D is connected
to an attainable point B.

Figure 3 Figure 5

OUAIITU[l/AII$lIERS, lliltTS & S0r.Uil0ilS 55


wherep = (x + ul 12, q = (x - ull2, and The total shortening of the spring
so on. In this calculation we've used is equal to the sum of the changes in
the equality each turn's length:

pq=
x+u x-u N
22 A/.=tL,--t
n1 =Ms =L
-. - l-L
2k z',
=v*Y.L-Y
2;Ql where
N
= IS,
I(l-1)=N(N-D=M.
Figure 6 whichisequivalenttox2 -u2 :* -f i=r
and so to equation (1).
point C (fig. 61, which can alterna- Let abe the greatest common di- Therefore, we get Mglk = L.
tively be constructed by rotating B visor of p and r. Then p = ab, r = act Let's now consider the case with
about A through 2L6 - 180' : 35' : where b and c are relatively prime. the water at a heightI, f2.If there are
350110. So in this case we can ap- Substituting into equation (2) yields n turns underwater, the other (N-n)
ply the above argument for q: 10. abq: acs, or bq = cs. It follows that will be above the surface. Let's find
In the cases 4 = 3, q = 4, and c1 = $ 4 is divisible by c-thatis, q = s,fl, the value of n/N. We begin by find-
the statement is obviously wrong and so bcd = cs, or s = bd. Now we ing the load for each immersed
(the rolling produces a hexagonal, have winding and the change in its
square/ or triangular grid of centers/ length. We then sum the changes
respectively). Thus, the answer is 4
m: azbz + c2& + a2c2 + b2&
and equate the result to LnlN - Ll2
:5 or q>7.(Y. Dubrovsky) = (o' * d2l(b2 + czl.
(this is the difference between the
In our particular case this reasoning lengths of n tums in the relaxed and
Ml15 results in the factorization compressed states|.
The given number is equal to 1,000,009 = (172 +221(72 + 5B2l
The weight of the immersed
1,000,009 = 32 + 1,0002. Let's prove windings is reduced due to the
= 293 . 3,4t3.
the following general f.act: If an in- buoancy of the water. The effective
teger m of the fprm 4k + 1 (with a A stronger statement can be weight of each turn is
positive integer k) can be repre- proved: anumber m=4k +lis
sented as the sum of two squares in prime if and only if it's uniqueiy rep- Ms(p-po)
two or more di{ferent ways/ then m resentable as the sum of two -a"Ms
is a composite number. squares. For details, see the article PN N,
Suppose that by V. Tikhomirov in this issue. (N. where o, = (p - po)/p. So the force com-
Vasilyev, D. Fomin) pressing the (i + l)th tum (counting
m=*+5P=u2+tfl. (1)
down from the surface) is
Since m is odd, we can assume that
x and u are odd, x ) u, and y and v Ms(^/- n) , atvtgi
are even/ y <v, sox*u andv+y are
Physics "_
ti-l/Trr
positive even numbers. Thenm can
be written as Pl 11 and the sum of the changes in the
Let the spring consist of N turns lengths of the immersed windings
(x+u ,-r\2 (by the statement of the problem, N is
m-l_+
t, 11
I
>> 1). Consider first the compression
of the spring due to its own weight +4 Mgn(N-n),attrtgnz
-
(v+v _ ,-.rr\2
+t, 'l in the absence of water. Turn num-
?*kN- kN, zkN,
\2 2) ber I (counting from above) must
support the upper (i - 1) turns. The
t:2r'2,:2
(x+ul /x-u\ lv+v\ change in its length is determined by
=L(L-!\.
tN 2)
=[ , J.[, J.l', ) the weight of these tums Mg(l - 1 )/N,
r:2 whereMis the spring's mass, andby Settingn/N = x and usingMg/k =
,lv -y
Tl- \ the spring constant for a single turn, I, we obtain the equation
[2
|

) which is N times that for the entire


,i\Ax2 I
=p2+rt+P+s2,
spring k. Therefore, x(I-x)* Z =r-r,
twg(i -t) I
^, _
'NNk
50 [lAY/JUlllI I gg4
from which we get x = ll1T4. Now we can describe the trajec-
m6m R7
Now the entire length of the tory of the particle using Kepler's
spring can easily be obtained by iaws. It can be considered a yety !o 7S PS
(1 I S S
3p
summing the length of the im- elongated ellipse with semimajor
mersed part Llz and that of the de- axis a = Ll2 andsemiminor axis b << = 150 s.
formed (N - n) windings above the a (the foci are at O and the initial po- In reality, however, the evaporation
surface: sition of our point particle). requires more time; thus, we have
Because the period of an elliptical obtained a lower bound for the
orbit with semimajor axis r is the evaporation time.
L*=L+Lfi-r1-r(1-x)2 same as that for a circular orbit of
2"2 As the ice evaporates the accelera-
radius r, let's find the period of rota- tion of the astronaut is a = P"S|LM,
=/r-4)
\ 2l
tion ? of the particle about the mass
M:
and she will cover a distance d =
a'c2f2 = 90 m. Since this is compa-
4n2 GmM rable to the distance given in the
or 771-* f = problem, the rescue will work!
l
Ti 12 Recalling the approximate nature
r* _ L(3-2a) - of our computations/ we can say
" -'zn"i-
^11
z(z- a) l^ that the astronaut will return to her
vGM
spaceship in a time t 100 s.
It's clear that the time necess ary for =
P1 12 the particle to reach the plane is P1 14
When a point charge is placed equal to the half the rotation period: When the extemal magnetic field
rlear a conductingplane, charges are is switched off, the value of the mag-
f
1.-
'l 1r 'LB
netic field drops from the initial value
induced in the plane that attract the
particle. Their effect i.s equivalent to
-
22 -
2Gtll B to zero. The changing magnetic
the action of an image charge -Q _"1 E4
-alzl' field induces an electric current in
located at the same distance on the each ring. Let's determine this cur-
other side of the plane (see figure 7). rent at time t after the magnetic field
The resultingforce is obtained from P1 13 is tumed off.
Coulomb's law: Consider the closed loop AfCbA,
In a closed vessel the number of
molecules striking the surface of the
which coincides with the left ring
'- k@
t-=-!;.=-
kQ2
ice per unit time is equal to the
(fig. 8). According to Lenz's law, the
current flows clockwise. Let the
(2*)' 4x2' number of molecules sublimating
from the surface (dynamic equilib- current in the AfC section be 1r(r),
and let the current in the CbA sec-
Because the force of gravity also rium). These are the conditions un-
tion belr(t). The electromotive force
depends on the inverse square of the der which the saturated vapor pres-
(emf) induced in this loop is
separation, let's replace the electri- sure P, is measured. Both fluxes of
cal force with an equivalent gravi- mass are equal to psn Sf 6, where p.
tational force. We can do this by as- : P,/RTls the saturated vapor den-
@ _ , tB(t)
6ind = -n*i.
suming that we have a mass M at O sity, S is the surface area of the ice,
with p is the molecular mass of water,
amd v
According to Ohm's law, for a
= .flRfff. is the mean mo- closed circuit we have
kQ2 lecular velocity. The factor ll5 (or
M-"' - 4mC
u1f1 strictly speaking, 1/4) accounts for
the choice of a particular direction
among six possible directions. If the
Eind = rrft)
*Urc+ rrft)filoo,
vessel is open, the flow of evaporat-
ing molecules remains the same, but
or, taking into account that the
there is no return traflici now the
pressure is P"12.
Let's estimate the time necessary
for complete evaporation by assum-
ing an initial mass of ice m - 0.2 kg
and a cross-sectional area for the glass
S - 30 cm2 and using p = 18 g/moI:

Figure 7 Figure B

IllAI[Tllttll/AIISTIIRS, llll'lIS & S0LUTl0lllS n7


lengths of the arcs 1o," and 1"oo are 9"l5rB number of days worked. So the
ru13 and Sruf 3, respeitively, we ob- Y =:-'-fiz. number of days worked is 1/5 of
tain 1OmR
30, the total number of days.

=-ry Pl 15 81 12
t(r)+s/2(r)
RLt^BA Denoting the distance between
Since every two statements con-
tradict each other, only one of them
the source and the lens as d and the
In the same way we write down can be true. All of them can't be
distance between the image and the
Ohm's law for the loop AfCdA: wrong, because in this case the hun-
lens as /, we write the lens formula
dredth one would be true. So there
as follows:
,,(r)\= (zn-zJi)rz
1,
rc1t)
111
are exactly one true and 99 wrong
2R Lt statements. This means that the
df F only true statement is the ninety-
Inserting this value of 1,(t) into the ninth.
equation above gives us In a small time period lt the dis-
tance between the source and the 81 13
r /+\_ (ton+aJa)2'z ar(1; lens decreases byAd =voLt, and the Hot water puts out a fire quicker
tc\L ) -
10R Ar
distance between the lens and the than cold water, because it evapo-
image increases by Lf : u cos cx,At. rates more rapidly and the vapor
Each element of a ring A1 carrying Then (see figure 9) impedes the access of air feeding the
electric curent {t) experiences the flames.
Amperean force AF = IltlLl . B(tl, 1 11
_-a-,
which is directed along the radius of d-voLt /+ucoscr.At df 81 14
the ring. Due to the bilateral sym- The answer is 384. Suppose we
metry of these forces relative to the or start to lace the shoe by passing
horizontal axis connecting the cen- the lace through the top right hole.
ters of the rings, the resulting force YE ucoscr.At Then it may go out through any
d2
acting vertically on each of the rings other hole except the top left
is zero. The absence of symmetry of one-that is, in eight ways. Next
the forces relative to the vertical we have to pass the lace through
axis passing through the center of the "parallel" hole on the other
the left ring (1r(tl * tr(tll results in a side and let it out through any of.
horizontal force. This force is equal the six holes. This makes B . 6
to the difference between the {orces ways. Proceeding in the same
acting on the arc AfC and the sym- manner/ we'll get four choices on
metrical arc on the opposite side of the next step, then two choices,
the ring: The image's velocityu equals that of and, finally, we'll pass the lace
Fr- r, = Ir(tllocB(tl the source vo when f | = drJcosct'. through the top left hole. Al1in all
F= - Taking into account that cos o = we have 8 - 6 . 4 . 2 : S94ways of
Ir(tllo"B(tl,
FIJP + rI' , we obtain lacing the shoe.
where 1r" is the chord subtending
thearcAfC (1o":2r sino/2 =r). Thp 111 81 15
final expression for the force F is dt d,Jcoso
-T-------:_-, F See figure 10: pointA is the mid-
point of the arc; the two pieces can
r(r)4I(r). be made to coincide by rotation
F = -9,Fsrs
5R Lt d,=Fj**l={,
\
r*41 about O through 45'.
^/coso 7
pz
1
The action of this force duringa small
time period t results in a change in
=p(r*@)
the ring's momentum: I z)
mLv = Ft = -9{3-" B(r)ar(r)
5R Bnaintea$er$
= -'f:'
1OR
o[s'(,)],
L 8111
Six days. The number of days
and so the ring will gain a velocity off must be 48 + L2=4timesthe Figure 10

5I illY/Jtlirr r ss4
= 9 (rather than 31's = 5.21. In {another factor of nzl. Although a
lhleido$coIE Galileo's defense, it should be decrease in size makes an animal
pointed out that all the calculations relatively stronger (see problem I ), it
1. The molecular force is propor- in the text of his book are coffect. doesn't help with "rowing" types of
tional not to the muscle's mass but 4. As in the previous problem, we movement. In order to hang in the
to its cross-sectional area. So if all assume that the pressure exerted on air, a smaller animal has to increase
the linear dimensions of an animal the foot's cross section in both a the number of strokes. The pitch of
are decreased by a faetor of n, its human and in a "monoped" is the the sound emitted by the wings in-
mass will be reduced by a factor of same. The body density for all mam- creases correspondingly. As for the
n3, and the force will decrease only mals is about 1 g/cm3. The mass of flies that bothered Gulliver, their
by a factor of n2. Thus, the relative the foot itself can be neglected in flight should be more or less silent
force (that is, the force per unit such approximations. In humans (to the human ear), just as the flight
mass) increases as the animal's size this pressure is about of birds is relatively quiet. The
decreases. Of course, the geometry is sounds that we hear when birds fly
not precisely the same in different 700 N are produced by other types of wing
=
1 N/cm2
animals, but the influence of the 2.3.r4.(ts cm)z f + movement.
dimensional factors far outweighs There is an even more energy-
the role of specific features, which (two feet!). Let I retem be z meters. consuming mode of flight based
allows us to establish a clear rela- Then the weight of the monoped upon the rotation of a propeller. In
tionship between the relative {orce willbe your leisure time try to estimate
and the animal's size. how much jam Karlsson would have
2. The force produced by a muscle 9'8 to eat so as not to lose weight in
B.l4z3.1,ooo. = 4,ooor3 N
is proportional to its cross-sectional B flight. {Karlsson is a character in the
area (see the previous problem), and stories of A. Lindgren.)
the distance the muscle contracts is (estimate the contribution of the 7. Warm-blooded animals expend
proportional to its initial length. foot's mass on your own), and the a significant amount of energy keep-
Because the mechanical energy out- cross-sectional area of the foot is ing their temperature constant. This
put of a single contraction is the is a particularly challenging problem
product of force and distance, it is for a small animal with a relatively
proportional to the cube of the ,10.(o'22'loo)t =goz2 c,. .
large surface atea. Its surface area is
4
organism's linear size (and corre- inversely proportional to the square
spondingiy to its mass). The same This gives us of its linear dimensions, while its
amount of muscular energy per unit mass is inversely proportional to the
502: l,
mass corresponds to the same poten- cube of its linear dimensions. In or-
z = 0.02.
tial energy at the top of the jump. der to achieve equilibrium between
Thus, geometrically similar animals It should be noted, however, that heat production and heat loss, small
should be able to jump to the same comparing the monoped to an ante- animals maintain higher body tem-
height. lope instead of a human would dras- peratures.
3. If the linear dimensions of an ticaLly change the result. 8. When we walk, our center of
animal are increased by a factor of n, 5. Of course,30401bs of food per mass-which is located in the lower
its body mass increases by a factor of day is an absolutely fantastic figure part of the body, just below the na-
n3. Suppose that, when this happens, both for a three-year-old child and vel-moves along the arc of. a circle
the thickness of a bone is increased for our readers. (Don't forget that whose radius is approximately
by a lactor of m. To the extent that water requirements weren't in- equal to the length of our legs. It's
we assume (in accordance with ac- cluded in our estimates.) Little chil- known that a body moving with
tual conditions) that the composi- dren eat gteatet amounts of food velocity v along a circle of radius I
tion of the bone doesn't change, the because of their faster metabolisms has an acceleration * I 1 directed to-
pressure on the bony tissue (per unit and perhaps even more because of ward the center of the circle. Two
cross section) must be preserved- the higher heat losses typical of forces act on a person when walk-
thatis, n3 lmz = t.Inotherwords,m smaller mammals. Another in- ing: the force of gravity and the sup-
is proportional to nl s. As one can stance of this factor can be seen in porting force. The resultant of these
see from the figure, Galileo in- problem 9. forces-the centripetal force-
creased the linear dimensions by a 6. The weight of an insect (that is, clearly cannot be more than the
factor of 3. So the animal's mass in- the force of gravity acting upon it) is force o{ gravity lmtPil does not ex-
creased by a factor of 33 : 27. It proportional to n3, and the pressure ceedmg, wherem is the mass of the
seems that for the sake of clarity on the air created by each stroke of body). So the maximum walking
Galileo increased the bone's thick- its wing is proportional to the wing's velocity equals u = nE|, which for a
ness in the figure by a factor oI27 l3 area lnzl and to the muscular torce human comes to about 3 m/s (a

0UlilTUl'l/AllSlItRS, lllilIS & S0LUTl0llS 5g


reasonable value). Children have a b k > 1, then (k - lla is divisible by p,
shorter legs, so th-ey have to run to \/ which is impossible since both fac-
keep up with their parents, but their ) tors k - 1 > 0 arrd a areless than p,
running is actually a succession of *,,....,1l.ll:* / andp is prime). None of these prod-
jumps and not rotations of the body ucts is divisible byp, so the remain-
7l i),:r .,r:1.,C,
about the axis of the leg. ders take each of the values I,2, ...,
9. As the problem deals with (_ \ p - I once. for the given value of
So
desert animals, we can assume that d L a thereis a unique b, I < b < p - l,
water loss is caused by evaporation such that ab = I (mod p). For a = 1
from the body's surface. Since the and a = p - | the corresponding
areaof. this surface is proportional to Figure 11 number b is equal to a. For any
the square of the animal's linear di- other a (2 < a < p - 2l1, b * a,because
mensions (nzl and the amount of from the black squares to the white a2 = 1 (mod p) impiies that a2 - | =
stored water is proportional to the squares equals the number of cubes (a - ll(a+ 1) is divisible by b only for
body's volume lnsl,Iaryer animals that changed from white to black. In a - | :0 or a + I =p. This proves the
can survive longer after drinking particular, the total number of cubes first statement. The proof of
their fill of water. And yet, desert that changed their underlying color Wilson's lemma is now completed
animals vary widely in size. How do is always even. So the answer to (a) iust as was done for p : 13 in the
we explain this? It turns out that our is no (because a 90o turn is possible article:
reasoning is applicable to animals only with a change of color). The
that are closely related taxonomi- answer to (b) is no/ too/ because ali (p-z)t=2.3.....(p-z)
cally (for instance, ierboa and corner squares in a 3 x 3 board are
camel), for which a decrease in size
is not accompanied by a reduction in
the same color, and so we have to
make only two like color changes =[, s4) ['-' (p-z\1')
2 ) 12
the water permeability their tissues. without any opposite changes.
\
It makes no sense to compare We'll use the shortened notations =t(modp),
beetles, lizards, and mice according a, b, c, d for the four-roll cyclic
to the similarity principle because moves about points A, B, C, D rn fig- afi(p-Lll:p - 1=-1 (modp).
their tissues are fundamentally dif- ure 11 (as we did in the article).
ferent. When verifying the number of single
rolls, take cancellations into ac-
count.
tlline $oluliolt$
Toy Slnre 6. (dcbalsdc. i. In figure 3 in the article, the
7. azbzc2dcd2c-rd-t. angles of triangle CBK are lC = 80
l.Ifr,1, u, d are the numbers of B. ab-t ad-r cd-r ab-| ad-r c2 ab-I - (by the condition), ZB = 20" (by con-
the R, L, U, D moves/ respectively, c-Ib-Ldcbada2d. struction), zK : l9o - 80'- 20' =
required to get from (0, Ol to (m, nl, 9. a-r dcbz d-l a-2 d-I czb d-r c-l d-r _
B0o, so BC : BK. We saw in the first
thenz - I = m, u - d = n, andthe to- a-1c2. solution that BC = BE, so BE = BKj
tal number of moves equals t + I + bz a-| (b-r alsb aa D the vacancy
10. ( and, since IKBE = B0'- 20" = 60",
u+d=m+n+2(1+dl. is shifted; the direction of rotation the triangle BEK is equilateral. Fur-
2. Required short sequences to here is the opposite of that in the ther, ZKBD = 50' - 20o = 40o =
reach a colored square lm, nl fuorr, previous solution). ZBDK (the last equality was proved
(0, 0) marked face up are, for in- Ll. a-rbaz. in the third solution), so KD : KB:
stance: UR^D 6or m > l, n = O; KE. To finish the solution, we can
URfu-LURDR form t2,n:1; simi- notice, for instance, that K is the
lar sequences for (l,nl; and center of the circle that passes
(0, n) and
RULURD f.or (m, n) = (1, 1).
[Ull. Fenmat-tulel' through B, E, and D; it follows that
3. The three possible half-turns l.If a2 + b2 is an odd prime, then ZEDB:I/IZEKB = 50o +2.
are given by RIJLLDR, (JRLDLL, one of the scluares-say, a2-is even. 2. In figure 6 in the article, ZEOD
Anozt. Therefore, a = 2r7, and the other one = 2IECD: 50o, because ZEOD is a
4. H(a-|, b) generates the half-turn is odd, so b = 2k + L.Tt,en a2 + b2 : central angle in the circumcircle of
huof the central cube, and H(a,b-rl 4nz + 4k2 + 4k + l:4m
+ 1, where triangle CED, and IECD is an in-
generates hr; both operations can be rrl=r72+k2+k. scribed angle subtended by the same
reduced to 38 moves. 2.For anyinteger a, l<aap-1, chord ED. We deduce that DEO is
5. Consider the chessboard color- consider the products I . a,2 . a, ..., an equilateral triangle andED = DO.
ing of the squares. If the vacancy (p - lla. The remainders of all these Thus, D lies on the perpendicular
comes back to its initial place, then products when divided by p are all bisector p of the segment EOi also,
the number of cubes that moved different (because If ka : la lmod pl, we know that BD bisects the angle

80 il[Y/JUrr roo4
EBO.If p arLd BD were different
lines, we could apply the argument .BULLETII{ BOARD" CONTINUED FROM PACE 52
from the fifth solution to show that
D lies on the arc EO of the circum- measured by converting light energy in a house to sound when only one
circle of triangle BEO. Bttt in this to electrical energy. The signals are is activated by smoke or fumes.
case we'd have IEDO + ZEBO = sent to a digital logic circuit, which Slaughterbeck devised the Rx-
180o, whereas in fact this
sum matches codes and identifies the Locker, a timed, internally locked
equals 50o + 40o = 100'. So BD is the bill. The logic circuit activates a pill dispenser designed to prevent
perpendicular bisector of E O, which voice chip. The device is poweredby overdoses. (He was inspired by the
means that triangles BDE andBD'O four AAA batteries. accidental overdose of a family
are congruent, and IEDB = zBDo Phillips said she wanted to com- friend.)
=50.-2:30'. bine her interest in electronics with The first- and second-place win-
3. Apply the construction with her desire to do something for blind ners, their parents, and their science
two reflections from the eighth so- people. (Tracy's brother is blind.) For teachers were guests of Duracell at
lution (fig. 12) (compare figure 7 in her scientific skill, Phillips will re- an awards ceremony in Anaheim,
the article). Then AP = CB1, AB : ceive a $20,000 savings bond from California, on March 30. The young
Duracell. inventors demonstrated their de-
Second-place winners were Chris vices for a luncheon audience and
Hyun Cho, 15, of East Setauket, exhibited them for thousands of sci-
New York; Seth Frankel, 17, of ence teachers at the 42nd annual
Demarest, New |ersey; Eric convention of the National Science
Magnuson, 18, of Uniontown, Ohio; Teachers Association (NSTA).
David Monson, 15, of Boise, Idaho; Ten students were also awarded
and Robbie Lynn Slaughterbeck, 17, $1,000 third-place savings bonds; 25
of OklahomaCity, Oklahoma. Each students won $200 fourth-place
of these student inventors received bonds; and 58 finalists were selected
a $10,000 savings bond. for $100 bonds.
Cho was inspired to create the ,To enter the Duracell/NSTA
Automated Page-Replacing Con- Scholarship Competition, students
BC trivance because/ as a member of his in grades 9 through 12 design and
school's chamber orchestra, he build a device that is educational,
Figure 12 found quick page turning to be a useful, or entertaining and is pow-
problem. Frankel created Safe-T- ered by one or more Duracell@ bat-
C,8,. Using the fact that points C, Eyes, a device that protects a power teries. Entries are judged on energy
B, C, and B, all lie on a circle with tool operator from injury by requir- efficiency, practicality, and inven-
center A and applying the Inscribed ing a face shield to be in place before tiveness. Every student who enters
Angle Theorem, we get lcptc = the tool can be turned on. Magnuson receives a gift from Duracell and a
V2ZCIC = IBAC (and, therefore, developed the Safe Distance Brake certificate of participation from
LCPp is congruentto LBAP), ard System, which shows, in different NSTA. Proposals for entries are due
tBpp = V2ZBtAC : 10'. By the colors, the amount of pressure a at NSTA each )anuary. For more in-
congruence of the triangles men- driver is applying to the brakes. formation, write to Eric Crossley,
tioned above, ZABP = lB \C rC = Monson invented the RF Intercon- NSTA, 1840 Wilson Blvd., Arlington
10o, so ZPBC = 80'- 10' = 70'. lV. nectable Smoke Alarm, a wireless VA2220l-3000, or phone 703 243-
Dubrovsky) unit that causes every smoke alarm 7100.

lb! Iltaffi ttot hotlt it 0oes! lndex ol Aduel'tisel's


The author of "The Good O1d
Pythagorean Theorem" in the AAPT 45
|anuary,febn:an-issue is V. N.
Berezin inor ''Be11'ozit\," as Burleigh Instruments hrc. 31
printed). Likerr-ise, the author of
NCTM 49
"Flexible in the Face of Adver-
sity" in the September,,October NSTA Special Publications L3, 14
1990 issue is A. P. Veselov (not
Sharp Cover 4
"Vesyolov"). Our apologies ior
mangling these names. Triton Pictures 4t

0lJAilrl|lll/ilrsr{tRs, lililTs & s0LllIt0ils 0r


TOY STORE

The nollinU cuhes


Can you roll your way into the record books?

by Vladimir Dubrovsky

HIS ARTICLE COMPLETES grid in such a way that the marked after 3 + 4k moves for arly k. Thus,
(for the time being) the discus- tace appears on the top again, and we can ro11 the cube to the square
sion of different kinds of rolling- figure out the possible number of l-, rl : {41, Ll in m + n : 4l + |
block puzzles in the September/ moves (rolls) this procedure will moyes. Denoting the right, left, up,
October and November/December take. and down moves as R, L, U, and D,
issues. In most of the puzzles we Well, this problem isn't all that we can write this sequence of rolls
considered earlier you had to ro11 difficult. The answer is given in fig- as R(JRal- I (Rk denotesJ< successive
pyramid-shaped blocks. This time ure 1: if we number the horizontal R moves).
we'll be rolling cubes around. and vertical lines of the grid 0,1,2, Similarly (and symmetrically) we
..., thus linking a pafu lm, nl with n
can get to the square l*, nl : ft, 41lr
Iumlleweed retli$ited each grid square (m and n are the m + n : 4l + lmoves: tlRu{l-t. The
We'll begin with one of the sim- numbers of the column and the simple trick used here-rolling the
plest problems posed in connection row/ respectively, that contain cube with its marked face on a side
with the "Tumbleweed" game in this square), then the minimal along this face-enables us to solve
the November/December issue. number N of rolls sufficient to the problem in m + n moves for any
Slightly generalized, it reads as fol- reach the square l-, ,l marked square (m, n)with m>2, n> 2.For-
lows. face up is equal to m + n for the rnally, the sequence of moves that
Consider a unit-square grid in the white squares/ m + n + 2 for the blue carries the cube over to this square
first quadrant (fig. 1) and a unit cube ones, andm + n + 4 = 5 torthe single marked face up is RtF-2RUR^-L[J.
sitting on the corner square of this yellow square (1, l). (See the example tor (m, nl = (7, 5l n
grid. Suppose five faces of the cube To see that this is indeed true, we figure 1-rolling along the dotted
are white, and one {ace-the top note first that after four rolls in the lines in this figure doesn't change the
one-is colored (we'11 call this the same direction the cube restores its relative position of the marked face.)
marked face). We have to ro11 the initial orientation (it makes a full Next, you can easily make sure
cube over to a given square of the turn). Therefore, for (-, ,l : (41,01or that m + n rolls aren't enough to get
10, 411we have N = rri + 17 = 41. Obvi- to the colored (blue and yellow)
ously it's impossible to get to these squares in figure 1, and that to get to
squares in a smaller number of the yellow square (1, 1) you havd to
moves/ and it's just as obvious that make even more than 4: m + n + 2
we can't get to any square (m, nl in moves (there are only a few conceiv-
7
Iessthanm+nmoves. ableroutes oflengthm +n from (0,0)
Now, imagine you roll the cube to lm, nl with m or n no greater than
from the initial position once to the l, and you can check all of them).
5 l
right. Then the marked face appears Problem 1. Prove that if you can
t ) :)
on the right side and will stay on reach the square (*, nl in k moves,
J this side after arry number of subse- then k - (- * n) is a nonnegative
2 quent rolls in the vertical direction. even number.
In particular, if you make only one According to this problem, the
0 O "ttp" roll and then continue roiling blue squares require no less than m
to the right, the cube will assume + n + 2 moves, while the yellow
the initial orientation after three one-(m, nl = (1,1)-requires at least
Figure 1 additional right moves, as well as m+n+4=6moves.

02 [lAY/JUilI 1SS4
Problem 2. Find the solutions for
the yellow and all the blue squares in b any closed path/ it
ends up on the initial
6 and m + n = Zmoves/ respectively.
If you compare these results with
the numbers given in the Novem-
ber/December issue for the Tum-
bleweed game/ you'Il see at once
Ly d,
square rotated "even-
ly." But can we actu-
ally obtain all 12 even
rotations in such a
way? The simplest
that these numbers are the mini- Figure 3 closed path consists
mum possible (that is, m + n for the of four moves around
white squares/ m + n +2 for the biue want the line on the cube's bottom a2x2 square (fig. a). We can see im-
ones, and 6 for the yellow one). to fit the line on the square, we'll be mediately that this "rolling tour"
left with only two ways to turn the results in a 120" rotation about the
Botalions and rolling touns cube on the bottom, which amounts diagonal passing through the center
More interesting questions come to 6.2 = l2 positions. of the square (point A in figure 4al,
up when we have a cube whose faces Now imagine that we've rolled as shown in figure 4b. The sense of
are all colored differently. Can we the cube somewhere and then slid it
roll a cube from the square 10, 0) to back without turning it (that is, by
it ends up
(m, n\ tn such a way that a parallel translation). Then the final
in exactll- the same position as it position of the cube can be obtained
started, or, more generall,v, in a cer- from the initial one by a certain ro-
tain given positronl tation. It's a very good exercise to
It turns out that possible final po- find all the24 rotations of the cube, ao
sitions depend on the coordinates and the 12 thatpreserve our pattern
Figure 4
lm, n) of the iinal square-more ex- of lines on the faces. Try to do it
actly, on the parltv oi m + n. This yourself. the rotation depends on the sense of
becomes clear rrhen 1'ou look at fig- And here's the answer to verify the rolling tour. In our notation the
ure 2. We drarr a line on each face of your investigation: there are three tour in figure 4a is written as LURD.
the cube as shorrn in this figure (the rotations (by 90", 180', and 270') Since the moves R and D are the
lines on the iaces we can't see are about each o{ the three axes perpen- inverses of I and U, we can rewrite
parallel to the hnes on thefu opposite dicular to the cube's faces (fig. 3a); it in the ftorrn L(JL-lU-l. Visitors to
faces), and drau' lines on all grid two rotations (120' and24O'l about our Toy Store may remember that
each of the four diagonals (fig. 3b); operations of this form (in general,
and the (least obvious) rotations, or XYX-IY-|, rendered in shorthand as
half-turns, about each of the six axes lX, Y1l are called commutatorc (of X
through the midpoints of the cube's and Y) and are often useful with trans-
opposite edges (fig.3c). This makes formational puzzles. (See, for in-
3 . 3 + 2 . 4 + 6 = 23 different rota- stance, the lan:uary lFebruary issue. )
tions; the one missing is the identity The commutator lL, Ul together
transformation. Our pattern of lines with the seven other commutators
Figure 2 is preserved by the three half-turns of L, (J, and their inverses-[I, U-1]
about the " face axes," eight diagonal : LU-rL-rU = LDRU,[U, L] = ULDR,
squares altemating their directions in turns/ and, of course/ the identity and so on-yield all eight diagonal
chessboard order. Put the cube on a transformation. rotations of the cube.
grid square so that the line on its bot- Since every single roll changes the Problem 3. Find the three 6-move
tom coincides with the line on the direction of the line on the cube's rolling tours that turn the cube 180"
square. Now ro11 the cube. The lines bottom, the rotations that preserve in its place about each of the three
on the cube and on the plane will fit the pattem of lines emerge after any face axes (shown in figure 4a).
each other again, so they will coin- even number of rolls, and we can call By taking the cube on rolling
cide a{ter a second roll and, in general, them even rotations. Twelve other tours/ we can obtain a1l 12 of its
after any number of rol1s in any direc- rotations will be calledodd. By prob- even rotations and return it to its
tion. This means that only a half of all lem l, the number of rol1s required to initial location.
possible positions of the cube on a get from the square (0, 0) to (m, nlhas
given square are accessible by rolling the same parity as m + n, so the rota- flollinUinacroM
it from another given square. Indeed, tions that can emerge as the result of Now we're ready to investigate
a cube can rest on any of its six faces rolling the cube from (0, 0) to lm, nl puzzles with many rolling cubes and
and we can tum it four different ways are even or odd depending on whether only one empty space. One version of
on this face. So there are 6 . 4 = 24 m + n is even or odd. such apuzzle is shown in figure 5 (on
positions on a given square. But iI we In particular, when a cube traces the next page). It consists of eight

OlJII{TUIil/TOY $TORI 03
inverses a-r = RDLU and b-r :
DLUR. This is easy to verify, so I'11
give only one illustration: RDLLUR
= RDLUDLUP = rL-r6-t-the succes-
sivemoves U andD intheinterme-
diate expression cancel out. Below
we'll use this shorthand notation-
Figure 5 that is, the cycles a and b-rather
identical 6-color cubes in a squarebox corners and on the top at the edges. than the notation for single rolls R,
with an empty space in the middle. According to the instructions, "if L, U, D'
Initially all the cubes are oriented in you find a solution in no more than So, in cyclic notation, operations
the same way so that each face of the 35 moves, you're a genius; if not, of the first type are commutators
square "rirrg" is the same color. You you fall just a little short." (deia vu!) of the triple cyclic moves
have to ro11 the cubes in such a way A natural way to tackle the prob- a3, b3, and their inverses. More ex-
that they all again become oriented lems we described, and others like actly, under la\, ba) = a3b3a-3b-a the
alike but not as they were initially. them, would be to try to find se- four corner cubes stay in place,
It's not hard to see that the final col- quences of rolls that result in rotating while the central cube makes a half-
oring in this problem can be chosen a single cube in its place and leave all firnhrabout the axis parallel to the
in 23 different ways, which is the the other cubes unaltered. This won't long sides of the box; then, loa, ba)
number of non-identical rotations of be economical; but the advantage of : a-3b3a\b-3 performs a half-turn h,
the cube. this method is universality: if we of the central cube about the axis
This particular problem was pro- learn how to turn a single cube in paraliel to the short sides of the box;
posedbypuzzle designers A. Dryom- every possible way, we'll be able to andlas, b-3] rotates the central cube
ov and G. Shevtsova, whom we've obtain the required arrangement by by 180" about the vertical axis (per-
aheady mentioned in connection rotating the cubes one by one. Notice pendicular to the box). Notice that
with other rolling-block puzzles. It's that this approach didn't work with the last half-turn fr, is produced by
interesting how they managed to pre- the rollingpyramids (see the Septem- 4 . 3 . 4 = 48 single rolls, while the
vent the cubes from slipping: the ber/October Toy Store), because a acfitalnumber of rolls inft, andh, is
cubes they use have grooves along pyramid always retums to its initial 46 (we've seen that in the combina-
their edges lh1.5l, and the bottom of location in the same orientation that tion a-tb-l two rolls-U ar'd D-
the box has a3 x 3 square grid of nar- it had at the start. cancel out; the same happens with
row laths glued to iq the iaths fit into I'll describe two types of operations bal.
the grooves, so when you tilt a cube that rotate a single cube. Both can be It's interesting to look into why
to ro11 it to an adjacent square, it performed in a "small" box measur- these operations behave as they do.
"stumbles" orr a ing 3 x 2; for definiteness, we'l1 as- The 4-ro11 move d cycles the three
lath and lands ex- sume that the empty square is at the cubes about A clockwise by one po-
actly where you middle of the longer side of the box, sition. Repeated three times, it
want. Of course, as shown in figure 7. Any sequence of brings the cubes to where they
other antislipping rolls in this box that brings the empty started after rolling one full circuit
devices exist, and space back to its initial location can about A-that is, a3 twists these
other problems, Figure 6
be represented as a combination of cubes 120" about their diagonals
too. the four 4-ro11 cyclic moves about the drawn from their vertices at A (com-
For instance, the set of rolling centers A and B in figure 7: DRUL pare figure 4). The operation b3 be-
cubes manufactured in Poland some (the clockwise cycle about A, which haves similarly about point B in-
time ago comprised eight identical will be denoted by ol, LDRU lthe stead of A. What happens under
cubes that had only one colored face ciockwise cycleb about B), and their a\bza-3b-\| The two left corner
each. It was supplied with instruc- cubes are twisted by a3, left intact by
tions saying that the puzzle could be d b3, untwiste dby a-3, and left intact
used for evaluating the IQ of the again by b-3. So eventually these
player. The maximum score was
given for the following "Royal Prob-
lem." Initially the four corner cubes
.1 \ ..-
-\ cubes, as well as the two right cubes,
stay put. At the same time, the cen-
tral cube is successively subjected to
are set in the box, their colored faces
up, and the remaining four cubes at
the edges are turned so that their
z '- aA
l{i 'i,rs:
120" rotations about its diagonals
frorn A and B and their inverses,
which results in the half-turn speci-
colored faces are down. You have to
ro11 the cubes so that the colored
16/ fied above.
Operations of the second type
faces appear on the bottom in the Figure 7 were presented by |ohn Harris in one

04 ilAY/JUrrr rss4
of the first publications on rolling cubes in its place. For instance, the
cubes. They can be written in the top left cube is rotated by an opera- Beat lhese !'E[ord$
foliowing general form: tion of thefotnara-l, wherer is any Although the expianations above
: xyxyxytr'f'r'y-', of the rotations of the central cube al1ow us to solve any (solvable) ro11-
H(r, yl
considered above: a rolls the top left ing-cube problem, they don't help
where x and y stand for a, b, or their cube to the central location, r rotates much if you have to find a solution in
inverses, and they also result in rota- it, a-1 rolls it back. a small enough (or the smallest)num-
tions of a single (central) cube. In par- Now we can do any rolling-cube ber of moves. I count such problems
ticular, taking (r, yl = la, bl, we gey arr problem in which the "target" posi- among the most intriguing and chal-
operation H(a, bl : a(bal2{b*1 a-Llzb-l tion differs from the initial one by an lenging transformational puzzles.
that performs the diagon aL rctation d, even rotation on each square turning Here are some of them/ together with
shown in figure 7; H(b, a) gives the cubes in their places one by one. As the lengths of the best solutions I
inverse rotation d;t; H(a-|,6-11 and for problems with odd rotations, I'11 know of. The empty square must re-
H(b-r, a-1) produce rotations do and illustrate them with one example: main in its place in all the problems
d o-t, respectively. Although these turn all the cubes in a box through below except one (figure out which).
operations are defined as sequences of 90o about their vertical axes. Problems
ten 4-ro11 cyclic moves/ they contain Imagine that the squares of the box The first three problems are bor-
a number of mutually annihilating are colored black and white in chess- rowed from |ohn Harris's article in
successive ro1ls (in the combinations board order so that the empty square the lournal of Recreational Math-
ba and a-1b-1 ,1, so after cancellations is white. To be "oddly rotated," a ematics,Yol.T, No.3.
they can be reduced to 32 rather than cube must make an odd number of 6. In a3 x 3 box with an empty
40 single rolls. The four other opera- rolls; therefore, it must moYe to a space in the middle each of the eight
tions of this forrn-H(a-I,b1, H(a,b-11, square of the other color (because one cubes has only one colored face. Ini-
and their respective inverses H(b, a-l ) roll changes the color of the cube's tially it's on the left side of each
and H(b-t,a)-don't add much to underlying square). But initially our cube. You must move it onto the
what we already have: they only re- cubes occupied three black and two right side. (30 moves).
produce some of the rotations gen- white squares. One white square for 7. Starting with the initial posi-
erated by the commutators above, the "b1ack cubes" is lacking, and the tion of the previous problem, turn
though in fewer moves. problem seems to be unsolvable! the colored faces onto the front sides
Problem 4. Exactly what rota- However, a solution does exist. Here of the cubes. (44 moves)
tions are generated by the operations it is a2b2a2b2a2D.It wasn't stipu- (In the next problem the coloring
H(a-t, bl ato.d Hla, b-r), and how lated that the vacant place remain is different.)
many rolls do they comprise after the same, and in the last move it's 8. Initially all the external faces of
possible cancellations ? shifted to a black square, while an the cubes arered, and allthe hidden
So, it tums out we have compara- extra "black cube" changes its un- faces are white. You must hide all
tively short operations for seven of derlying color. the red faces. (84 movesr)
the eleven even non-identity rota- Tricks aside, this solution i11us- The remainingproblems are for 5-
tions of the central cube on a 3 x 2 ttates a general rule for solving ro11- color cubes. The record solutions for
board: the three half-tums h1, h", hu, ing-cube przzles. First, using the the first two are heid by A. Pante-
and four 120o rotations about two of chessboard coloring, we determine Ieyev, a Moscow mathematician.
the diagonals. As to the remaining which cubes must change their un- 9. Turn all the cubes in the 3 x 3
four rotations about the other two derlying colors. Then we ro11 the box through 90o about the axis par-
diagonals, I don'tknow atice, elegant cubes so as to change colors as re- allel to the lower edge of the board.
way to obtain them, but they can be quired, without paying much atten- (60 moves)
represented as combinations of d, tion to the cubes' orientations. After 10. Do the same as in the previous
and d r. For instancg the rotation d in that, the required final orientations problem in the 3 x 2 box (45 moves.)
figare 7 results when do is followed become attainable by even rotations I 1. You're given five cubes in the
by drr is, d = d od r-t . A shorter of the cubes in their new places, 3 x 2 box having the same orienta-
way to -that
perform d is (i) tum the cen- which can be done by using the op- tion. Roll them so as to obtain five
tral cube (and maybe some other erations we described above. different colors other than the initial
cubes) so that the axis of d fits onto Problem 5. In a 3 x 3 box with an one on their top faces. (The author,
the axis of, say, dp; l2l perforrn dr; empty space in the center, all the cubes V. Rybinsky, cafl do this in 14
(3) "undo" operation l-that is, d = are colored the same and have the same moves.) O
a3dra-3, or a-3do--r a3. Here d is repre- initial orientation.Is it possible to ob-
ANSWERS, HINTS & SOLUTIONS
sented as a coniugate ol d, (or d or )- tain a position in which (a) one cube
ON PAGE 60
see the lanuarylFebruary Toy Store. is rotated 90", (b) two comer cubes are
Conjugation can also be used to rotated 90', while all the other cubes lAccordlng to Martin Gardner,
rotate separately any of the corner retain the initial orientation? Harris later found a 74-movc solutior.r
[shcrrp EL-E3OO Teaching Colculotortr
It's o new kind of colculotor. For o
new \floy of teoching. Doing whol no

other colculotor hos ever done


before. Using Shorp's Equotion Editor

to disploy entire equoiions on screen-

iust os you'd write them. Exploining


in words on the screen- how to solve
froction problems. And moking stutis-

tics eosy- since there sre no olher


intimidoting functions. So if's eosier

thon ever for 4th-8th grode students


to leorn moth-even when you're
not there. And, there's o l0-unit
a Classroom Pock thot comes with o
a
a versofile colculctor opplicction guide.
a
a The Shorp Et-E300 Teoching
': i t Colculotor. So convenient, it mokes
I
onyproce, your workploce. For o
SHAFIP ri-r:oo
brochure ond the nome of your
a
neorest educolionol distribulor, coll

il;i..3t:L
jr1,::l:.;-i:'::,,''t
].ii:.

it::::ii::.:ii:.:Ji:.:,:'.

Potrebbero piacerti anche